Renal :)

Pataasin ang iyong marka sa homework at exams ngayon gamit ang Quizwiz!

1. The protein concentration of plasma entering the efferent arteriole will likely be lower than normal following: A. an increase in glomerular plasma flow without a change in glomerular capillary pressure (PGC) B. constriction of the efferent arteriole C. an increase in the filtration fraction D. a fall in the pressure in Bowman's space E. an increase in the filtration coefficient (Kf)

1. Answer: A A. True. Since the fluid filtered through the glomerular filtration apparatus is removed from a larger than normal volume (flow is higher), the rate of rise of protein concentration in the glomerular capillary will be less than normal. B. False. Constriction of the efferent arteriole increases (PGC) while decreasing flow. This increases the filtration fraction and concentrates plasma proteins more than normal. C. False. See B. D. False. A fall in PBC leads to a larger filtration pressure gradient (PGC - PBC), increases the volume filtered and raises oncotic pressure in the plasma left behind. E. False. An increase in the filtration coefficient increases the rate of filtration for any driving pressure. It increases the concentration of proteins remaining in the glomerular capillaries.

A 55 year-old woman with uncontrolled type 2 diabetes and sustained hyperglycemia (plasma glucose > 500 mg/dL) presents with urine glucose concentration of 100 mM. Under these conditions, any further increase in plasma glucose concentration will result in: A. an increase in glucose reabsorption. B. no further change in the excretion rate of glucose. C. a decrease in the clearance of glucose. D. a proportional increase in the filtered load of glucose. E. an increase in the fractional reabsorption of water by the thin descending limb.

1. Answer: D A. False. An individual spilling glucose in the urine has exceeded the transport maximum for glucose reabsorption. Increasing the filtered load of glucose (GFR x PGlu) delivers more glucose to the tubule each minute but cannot lead to further glucose reabsorption since the glucose transport mechanism is already saturated. B. False. The amount of glucose excreted each minute will equal the filtered load minus the rate of glucose reabsorption. Since the latter is equal to TmGlu in this case and the filtered load (GFR x PGlu) increases linearly with PGlu, the amount excreted will increase with PGlu. C. False. Since glucose is freely filtered, reabsorbed (although partially in this case), the clearance of glucose can approach but never be greater than the plasma volume filtered per minute (GFR or CIn). D. Correct. See A. E. False. The presence of glucose in the thin descending limb decreases the osmotic gradient for water reabsorption

What are the 5 categories of circulatory shock

1. Hypovolemic shock: hemmorrhage/diarrhea/vomiting leads to loss in blood volume 2. Cardiogenic shock from MI acute organ impairment 3. Vasogenic Shocks (physiological response varries but pattern of hemorrhage response is the same) Septic shock: toxins -> vasodilation -> drop in blood pressure/CO Anaphylactic shock: allergy -> vasodilation -> drop in blood pressure/CO Neurogenic shock: ANS disrupt -> vasodilation/hypotension/syncope

The Urinary Bladder

1. Relaxed. Examine the section of the relaxed urinary bladder (nonhuman primate), slide 472. Observe that (1) the mucosa is thrown into thick, irregular folds; (2) the transitional epithelium is 6-8 cell layers thick; (3) there is a very thin muscularis mucosae; (4) the muscular coat is well-developed with many interlacing bundles of smooth muscle fibers (as a result the muscle fascicles are seen sectioned at various angles) but not sharply demarcated into layers; and (5) the peritoneal serosa has an elastic lamina within the submesothelial connective tissue. The serosal surface is limited to the superior surface of the urinary bladder (and part of the posterior surface in men); the rest of the surface, where it attaches to the pelvic wall, is an adventitia. 2. Distended. Slide 475 is a tissue section of the wall of a distended (stretched) urinary bladder. This tissue section looks stretched, compared to the relaxed bladder slide. Observe that the mucosa is not folded and the transitional epithelium has been stretched to only a few layers. The surface cells are relatively squamous in shape when stretched. The wall of this tissue section is not so well fixed as that of the previous tissue section, causing an artifactual shrinkage of the smooth muscle of the muscularis. You are not responsible for identifying the distended urinary bladder without the whole wall being present in the tissue section for context.

Components of the nephron

1. Renal Corpuscle. Blood enters through the afferent arteriole, circulates through the tuft of capillaries and exits through the efferent arteriole. While in the capillaries, the blood is filtered, so that particulate objects (like cells) are retained within the capillary, but water, ions, and a minute amount of proteins smaller than albumin (68 kD) pass into the urinary space. Many of these components of the ultrafiltrate will be selectively reabsorbed as the filtrate is modified into urine as it passes through the various tubules of the cortex and medulla. Glomerular capillaries have a fenestrated endothelium and the outside surface of the capillaries is covered with cells called podocytes. These cells constitute the visceral layer of Bowman's capsule. The podocyte basal lamina (80-300 nm thick) abuts the endothelial basal lamina, together comprising an important filtration barrier, selective for both size and charge of blood-borne molecules. The podocytes have cellular extensions that in turn subdivide into many finger-like projections (pedicels).

B. Cellular Details of the Nephron The cellular detail of the kidney is best observed in slide 451 and slide 460. Take time to orient yourself to this tissue section: where you are in the tissue section determines what you are likely to see.

1. Renal Corpuscle. Examine the cortical region and observe the numerous glomeruli, each partially encircled by a clear area. The renal corpuscle consists of two parts: the vascular tuft of capillaries, the glomerulus, surrounded by an epithelial sac, Bowman's capsule. Renal corpuscles are found only in the cortex. The glomerulus arises from an afferent arteriole that enters the corpuscle at its vascular pole, and branches into the glomerulus. An efferent arteriole, formed of the reuniting capillary plexus, exits at the same vascular pole. Search several glomeruli to find a plane of section that shows an arteriole entering or leaving. You are not expected to differentiate between the afferent and efferent arterioles, although the lumen of the efferent arteriole is smaller than that of the afferent arteriole. Within the glomerulus, identify endothelial cells forming the capillary walls. Their endothelial cell nuclei are typically small, dark, and flattened.

10. The kidney is unique in that an immune response may be specific to the glomerulus or to the cortical interstitium, despite the fact that both compartments are connected. The cells that are best situated to provide a barrier to the transit between these two compartments of cells of the immune system such as lymphocytes or polys are the: A. juxtaglomerular cells. B. cells of the macula densa. C. endothelial cells. D. parietal layer cells of Bowman's capsule. E. extraglomerular mesangial cells. F. intraglomerular mesangial cells.

10. Answer: E This question is a test of how accurately you've imagined the relationship of the glomerulus to its surrounding structures. The intraglomerular connective tissue space is largely filled by mesangial cells. The extraglomerular interstitium has cells, fibers, and vessels of the PTCP. Between these two CT compartments lie the extraglomerular mesangial cells. A. The juxtaglomerular cells are a single layer bound to the endothelium of the efferent and afferent arterioles. This layer (tunica media) separates the endothelium from the tunica adventitia B. Cells of the macula densa are part of the DT epithelium. They separate the DT lumen from the cortical interstitium C. Endothelial cells are also epithelial, separating the vascular lumen from whatever is their surrounding CT compartment. D. The parietal layer of Bowman's capsule is an epithelium separating the cortical interstitium from the urinary space. E. Extraglomerular cells are CT cells situated at the vascular pole. There, they sit between the glomerular CT and the CT of the cortical interstitium. F. Intraglomerular mesangial cells sit within the glomerular CT, where they are largely between the endothelial cells and the podocytes.

12. A 54-year-old man with small cell lung cancer presents with lethargy and confusion. Blood work shows an increase in plasma levels of ADH, possibly from an ectopic production, and hyposmolality. Patients with the syndrome of inappropriate antidiuretic hormone secretion (SIADH) are likely to exhibit an increase in: A. urine flow rate. B. medullary interstitial urea concentration. C. vasa recta blood flow. D. water permeability of the thin descending limb of Henle's loop. E. fractional water reabsorption by the proximal tubules

12. Answer B A. False. Urine flow rate decreases as plasma ADH is elevated and more water is reasobed by the CD. B. Correct. ADH enhances urea permeability by the IMCD leading to an increased concentration of urea in the medullary interstitium. C. False. ADH reduces vasa recta blood flow by vasoconstriction D. False. The tDLH water permeability is unaffected by ADH. The reabsorption of water by this segment, however, is dependent on the interstitial osmolarity and thus it increases when the interstitium is more concentrated, as it happens with high ADH level in plasma. E. False. Water reabsorption by the proximal tubule is unaffected by ADH.

2. A modest increase in constriction of the efferent arteriole in a euvolemic individual will result in: A. increased renal plasma flow B. decreased filtration fraction C. a lower than normal oncotic pressure in plasma delivered to the peritubular capillaries D. decreased organic solute delivery to the proximal tubule E. increased proximal tubule reabsorption of salt and water into peritubular capillaries

2. Answer: E A. False. A constriction of the efferent arteriole will increase resistance and decrease renal plasma flow. B. False. Efferent arteriole constriction will decrease flow, raise glomerular capillary hydrostatic pressure and, therefore, GFR. FF = GFR/RPF will therefore rise. C. False. Oncotic pressure increases with an increased FF. D. False. Increased GFR will increase the filtered load of organic solutes in the tubules. E. True. Constriction of the efferent arteriole will increase the FF, raise oncotic pressure, and by decreasing flow into the peritubular capillaries decrease PCAP. These two factors will promote reabsorption of solute and water into peritubular capillaries.

3. Which of the following statements is FALSE? A. Increasing the plasma inulin concentration will increase its clearance. B. Competition between substances secreted by the same carrier system might decrease both their renal clearances. C. The fluid reabsorbed in the proximal tubule is isosmotic. D. The fluid delivered to the descending loop of Henle is isosmotic. E. In an individual with a normal plasma volume, an increased rate of filtration leads to an increased rate of Na+ reabsorption by the proximal segments of the nephron.

3. Answer: A A. FALSE. Inulin is cleared only by filtration. Increasing plasma concentration increases the excretion rate, but the plasma volume/min cleared remains the GFR. B. TRUE. Competition between two solutes for the same carrier decreases the rate of transport of each. If the rate of transport (secretion) falls below that necessary to clear the plasma of the solutes, the clearances of the solutes will decrease. C. TRUE. Water reabsorption at the proximal tubule is secondary to solute reabsorption in such a way that the reabsorbate is isosmotic. D. TRUE. See above. The fluid remaining in the tubule and delivered to the loop of Henle must be also isosmotic. E. TRUE. As a consequence of GT balance.

As a consequence of Glomerulo-Tubular balance: A. renal blood flow is relatively constant in spite of variations in renal arterial pressure. B. less fluid is delivered to the distal nephron when GFR increases. C. GFR remains near constant when renal arterial pressure falls. D. tubular reabsorption increases if GFR rises. E. an increase in RBF leads to an increase in filtration fraction

4. Answer: D A. This is autoregulation of renal blood flow by the myogenic mechanism, not G-T balance. B. This is not G-T balance and it is a false statement: more distal fluid will be delivered if filtration increases. C. This is autoregulation of GFR involving myogenic response, effect of Ang II on efferent arteriole and tubulo-glomerular feedback, not G-T balance. D. True. GT balance means that reabsorption changes in parallel with changes in GFR. E. False. This is not G-T balance and, in general, an increase in RBF, although increasing GFR, decreases the FF.

Which of the following statements is correct? A. The osmolality of plasma in the ascending vasa recta is less than that in the descending vasa recta at the same level. B. In conditions of high excretion rate of solutes, the osmolarity of the urine may exceed that of the inner papillary interstitium C. Intravenous administration of hyperosmotic saline will increase collecting duct water permeability. D. The lumenal fluid at the end of the proximal tubule has an osmolality 2/3 that of plasma. E. The luminal fluid at the end of the thick ascending limb of the loop of Henle is hyperosmotic.

5. Answer: C A. False. The ascending vasa recta osmolality is larger as it drags away interstitial solutes from deeper zones in the medulla. B. False. The urine cannot be more concentrated that the interstitium. C. Correct. Hypertonicity of the plasma stimulates osmoreceptor firing and release of ADH that increases CD water permeability. D. False. The filtrate in the PT is always isosmotic. About 2/3 of salt and water is reabsorbed by the proximal tubule isosmotically. E. False. It is hyposmotic since salt is being reabsorbed along the ascending limb but water cannot follow.

6. Which of the following statements about glucose handling by the kidneys is FALSE? A. Below the Tm the rate of glucose reabsorption increases with increases in GFR. B. The glucose concentration in the arterial plasma of a healthy individual is higher than that in the venous blood draining the kidneys. C. The clearance of glucose may be raised by increases in plasma glucose. D. At constant GFR the filtered load of glucose is a linear function of the plasma glucose concentration for any plasma glucose level. E. Saturation of the proximal tubule reabsorptive mechanism for glucose results in glucose excretion.

6. Answer: B A. True. As the filtered load of glucose increases so does its reabsorption. B. False. Since all the glucose is normally reabsorbed its concentration in the renal venous blood is the same as in arterial blood. C. True. If the plasma glucose increases sufficiently to saturate the reabsorptive mechanisms, D. True. The filtered load is equal to plasma concentration times GFR. E. True. If the proximal tubule mechanism saturates glucose is excreted since there are no other mechanism for its reabsorption in the other tubular segments.

A common cause of kidney injury in older men is prostatic enlargement that constricts the urethra and can increase pressure in regions upstream of the obstruction. While transitional epithelia are resistant to pressure/stretch damage, simple epithelia are not. If the first simple epithelium upstream of the obstruction is selectively damaged, what would be the result? A. glucose lost in the urine B. edema due to water retention in peripheral tissues C. loss of functional responsiveness to ADH D. proteinuria E. blood in the urine

9. Answer: C Pressure caused by an enlarged prostate will be felt, in ascending order, in the prostatic urethra, the bladder, the ureters, the calyces, the collecting ducts. The collecting ducts are the first simple epithelium encountered - their chief function is ADH-sensitivity. A - glucose in the urine indicates damage to PCT, which is responsible for resorbing glucose. B - water control depends primarily on a good GFR across the glomerulus, good resorption through the PCT, good function in the thin tubules, in that order of importance. D - proteinuria results chiefly from GBM dysfunction and/or PCT damage. E - blood in the urine can result from a break in the filtration barrier, or damage to the epithelium of any of the tubules. CD, being among the thicker of the tubules is not a likely site for such an injury

On average a person excretes in the urine about 600 mosmoles of solutes every day. . C. What would you anticipate would be the effects of furosemide (a loop diuretic) on the: • interstitial osmolality • luminal osmolality • maximal concentrating ability • maximal diluting ability • osmolar clearance

A more intuitive interpretation of the free water clearance is possible by writing the equation: 𝑉̇ = COSM +CH2O The total urine volume can be viewed as having two virtual components. One component, COSM, contains all the urine solutes in a solution that has an osmolality equal to that of plasma. The second component, CH2O, is a volume of solute-free water that makes the urine dilute if it is excreted, or concentrated, if it is reabsorbed

20. When Na+ balance is normal, as a consequence of Glomerulo-Tubular balance: A. A constant fraction of the filtered salt and water is reabsorbed from the proximal tubule. B. The concentration of the filtered substances in the Bowman's space is the same as in plasma. C. The GFR is maintained constant in spite of variations in arterial pressure. D. The proximal tubule reabsorbate is isosmotic. E. A constant fraction of the renal plasma flow is filtered at the glomerulus.

A. A constant fraction of the filtered salt and water is reabsorbed from the proximal tubule.

65yo man MI 4month ago: Short breath + Swelling + Dist. Neck vein + Edema. Rapid + Rales. Pulse Rate 110b/m, BP: 110/70. A ECF volume is higher or lower? B. ECV higher or lower? C. How is renal Na handling? Plasma renin, NE/E, ADH?

A. ECF volume high: edema, vein distension, * rales, weight gain. B. ECV decreased bc damage to myocardium, BP, CO, tissue perfusion reduced. C. Kidney retain sodium due to aoritc/carotid sensor of low ECV = more sympathetic = more renin by Jxapparatus + reduced perfusion pressure. Also decreased delivery of NaCl to macula dense (due to reduced GFR due to sym const. of arterioles) Renin triggers AII that stimulates Aldosterone to retain sodium. ECF increases and expanded venous volume increased firing from cardio-pulm sensors to oppose (not override) response. Low ECV activates system for conserving circulatory volume but ECF contraction leads to both baroreceptors to reduce firing.

Volume of ECF/ICF after 1.5 L water: Effects with: A. Excessive loss of water (dehydration)

A. Excesive water loss means the total osmolality of the body fluids is increased and the volumes of the intracellular and extraceullar comparments decrease PROPORTIONATLY to original volume. Due to equilibrium of pOSM, water loss from EC fluids with elevation of EC osm leads to water movement out of cell until internal/external pOSM is equal. For desert or fever, H2O lost by sweating is insensible and comes from plasma that becomes hyperosmotic and shifts H2O to move into the plasma. This leads to flow of H2O into ECF.

48. All of the following will induce an increase of ADH secretion EXCEPT: A. Increase in right atrial pressure. B. A potent diuretic acting on the thick ascending limb. C. Severe hemorrhage. D. Ingestion of a salty meal without water. E. Severe diarrhea

A. Increase in right atrial pressure.

19. Isosmotic water reabsorption at the proximal tubules implies: A. The osmolality of the tubular fluid remains constant along the proximal tubule. B. The concentration of inulin along the proximal tubule remains constant. C. Active water transport must be responsible for the reabsorption of water. D. The fluid reabsorbed from the proximal tubule has the same protein concentration as plasma. E. The urine will be isosmotic in the absence of ADH

A. The osmolality of the tubular fluid remains constant along the proximal tubule.

60. A hemorrhage that results in a loss of 20% of blood volume would lead to: A. an increased fractional reabsorption of Na+ in the nephron B. movement of fluid from the capillaries into the interstitium C. movement of fluid from the extracellular to the intracellular space D. increased firing of cardiopulmonary receptors E. an increased GFR

A. True. A loss of blood volume decreases firing from both cardiopulmonary and arterial baroreceptors and triggers an increase in sympathetic tone, renin secretion, AII formation and a rise in ADH and aldosterone levels. This increases Na+ reabsorption in the proximal tubule (AII and symp. firing), loop of Henle (ADH) and distal nephron (aldosterone). B. False. The drop in blood pressure and arteriolar constriction both lower PCAP and lead to absorption of fluid from the interstitium into the capillary. C. False. With loss of circulating volume elevated epinephrine stimulates liver gluconeogenesis and glycogenolysis. Blood glucose levels rise and increase ECF osmolarity. This pulls fluid out of cells and adds to the ECF volume. D. False. See A/B above. E. False. GFR falls because of a drop of PCAP in the glomerular capillaries. This results not only from a fall in arterial pressure, but also from the baroreceptor reflex constriction of the afferent arteriole.

26. Which of the following would tend to increase free water clearance in a subject? A. a decrease in levels of circulating antidiuretic hormone B. a decrease in urine flow C. an increase in the water permeability of the collecting duct D. infusion of a hypertonic solution of saline E. a decrease in the osmolar clearance (Cosm)

A. a decrease in levels of circulating antidiuretic hormone

Impact of ACE inhibitors

ACE inhibitors prevent AII production (vasoconstrictor for raising BP in hypovolemia). ACE inhibitors thus cause body-wide dilation, increased renal blood flow, reducing vascular resistance. Dilates efferent arterioles more so so lower glomerular capillary pressure/ glomerular filtration. Blocking Angiotensin II prevents aldosterone release so increased sodium/water excretion. Blocking AII prevents Na reabsorption by the Na/H exchanger at the PCT.

Water reabsorption is controlled by ADH for permeability at CD. AII and reduced cardio/baro firing rates increase ADH and Na/Water retention to restore PV.

ANP by filled atrial granular cells promotes vasodilation/renal filtration/excretion of Na by kidney. ANP inhibits aldosterone. When ANP falls due to less volume = ADH can begin to act again.

Sodium and water reabsorption

About 2/3 of the salt and water filtered at the glomerulus is reabsorbed at the proximal tubule. Sodium is also reabsorbed at the thick ascending limb (TAL) of the loop of Henle, the distal tubule and the collecting duct. Sodium is the major ion reabsorbed and the transport of many other solutes and water is coupled to that of Na+ . Most of the O2 consumed by the kidneys is used in energizing Na+ transport. Due to the large amount of Na+ reabsorbed most of the O2 consumed by the kidneys is used in energizing Na+ transport. Although entry of Na+ into the tubular cells across the luminal membrane is energetically a downhill process the exit from the cell across the basolateral membrane is always uphill, energized by the Na+ -K+ ATPase in most cells. Water reabsorption at the proximal tubule is secondary to Na+ reabsorption and explained by the mechanism (Standing Osmotic Gradient Hypothesis) described for the small intestine: Continuous reabsorption of Na+ and Clcreates a small increase in the osmolality of the intercellular spaces that drives the reabsorption of water across the highly water permeable cell membrane and tight junctions. The descending limb of Henle's loop is also highly permeable to water. The driving force for water reabsorption at this segment and at the collecting duct is the high osmolality of the medullary interstitium (see Osmoregulation lecture). On the other hand, the ascending limb and distal tubule are impermeable to water, and no water reabsorption takes place at these segments. The permeability to water of the collecting ducts is regulated by the antidiuretic hormone (ADH). High level of plasma ADH increases the water permeability of the collecting duct promoting water reabsorption and the excretion of a small volume of concentrated urine.

Acid-Base Balance: Aldosterone will cause secretion of K when K is too high. Aldosterone causes secretion of K when K is too high

Acute acidosis reduces K secretion by inhibition the basolateral Na-K ATPase and lumenal K permeability (Alkalosis will increase these for K secretion). Inhibition of Na-K ATPase at the PCT by acidosis depresses salt and water reabsorption thus causing larger tubular flow to the distal nephron. More flow and Na delivery in the distal nephron adds K secretion after a few days. Chronic acidosis decreased ECF volume stimulates the R-A-A system so metabolic acidosis due to inorganic acids, plasma K increases and stimulates aldosterone secretion and K secretion thus offsetting the initial acidotic K decrease. Hard to know rate of K+ excretion in chronic acidosis.

What are the responses to Acidosis

Acute response to acidosis includes: insertion of NHE-3, rh-glycoproteins, and H+-ATPase transporters. Chronic response includes transcription/translation of genes/mRNA for transporters and enzymes needed in ammoniagensis. With renal compensation, there should be a greater rate of renal ammonia produced to allow daily excretion of 100s mEq H/NH4 and equal reabsorption of HCO4 until acid-base balance is restored. Endothelin-1 (ET-1) secreted by endothelial and PCT cells + cortisol from the adrenal cortex are stimulated by acidosis to increase expression of NHE-4/NBCe1. PTH secretion is stimulated by acidosis to inhibit phospohate reabsorption in the PCT so more is used at TA. Acidic urine also has a U HCO3 of zero.

Effector Mechanisms

Adjustments in sodium excretion by the kidneys can be accomplished by changes in both the filtered Na+ load, determined mainly by the GFR, and tubular Na+ reabsorption. These functions are regulated by intrinsic renal mechanisms and by external regulatory factors, e.g., autonomic nerves and circulating hormones. Since the renal processing of Clis usually coupled directly or indirectly to that of Na+, the following discussion on regulation of Na+ excretion applies equally to Cl The daily amount of Na+ filtered by the kidney in man is PNa∙ GFR = 140 mEq/L x 180 L/day = 25,200 mEq/day. Thus, the total amount of Na+ in the ECF is filtered more than 10 times per day. Only a small fraction (<1%) of the filtered load is excreted, and, as discussed previously the filtered load and tubular reabsorption must be precisely balanced (Glomerulo-Tubular Balance) to avoid excessive losses or gains of ECF Na+ . Tubular reabsorption may be considered in two broad categories. Proximal segments of the nephron (proximal tubule and thick ascending limb of Henle's loop) have a large capacity to reabsorb Na+ , and they have the ability to increase or decrease transport promptly in response to corresponding changes in Na+ delivery (Na+ load) to the segment. The more distal segments of the nephron (distal convoluted tubule and collecting duct) have relatively limited capacity to reabsorb Na+ (5-10 % of filtered Na+ ) but are able to "fine-tune" the reabsorption so that the final urinary Na+ ultimately matches intake Multiple regulatory mechanisms are involved in the control of Na+ excretion. Body Na+ influences key physiological functions such as ECF volume, cardiac output, blood pressure, blood flow to vital organs, etc. The presence of multiple regulatory mechanisms protects against major physiologic dysfunction, if any one mechanism becomes defective..

5. Collecting Tubules and Ducts.

Although the collecting tubule shares morphological features of the collecting duct, the profiles are of smaller caliber, they are intermixed with the other tubules in the cortex, and they are not radially oriented within the medullary rays as are the collecting ducts into which they drain. The collecting ducts are the main constituents of the medullary ray in the cortex. Their epithelial cells are noticeably more pale than those of the nephron tubules; they are cuboidal to columnar with a round nucleus, regularly spaced apart with domed apical surfaces; they have unusually distinct cell boundaries that are especially apparent in tangential sections through the wall of the duct. There are two types of cells forming the walls: principal cells, the more numerous ones, and intercalated cells, the less numerous ones with darker cytoplasm. As noted, the collecting ducts travel radially in bundles (medullary rays) from the cortex to the medulla. The height of the cells in the walls of the collecting ducts increases as they near the renal papilla. At the apex of the renal papilla, the successive union of collecting ducts results in the formation of the papillary ducts (of Bellini). The papillary ducts are straight terminal ducts about 200-300 µm in diameter. They are not visible on this slide. Urine flows from these openings into the calyx that caps the papilla. VERY IMPORTANT HINT: Knowing where you are in the kidney is of prime importance for identifying the cells and tubules. For example, if you are looking at tubular profiles in the medulla they cannot be distal convoluted tubules because that portion of the nephron is confined to the cortex. Likewise, if you are examining the cortex, there is a limited choice of possible identifications of the cell types that you might see. For example, there are no thin segments of loops of Henle in the cortex. You can always tell that you are in the cortex if you see glomeruli.

Regulation of Sodium Reabsorption by the Distal Segments The Renin-Angiotensin-Aldosterone System

Although there are differences in the precise functions of the late distal convoluted tubules and collecting ducts, Na+ reabsorption there is influenced by a number of factors that together "fine tune" reabsorption. Under euvolemic conditions, with small changes in Na+ intake, aldosterone is the primary regulator of Na+ reabsorption by its effects on distal segments of the nephron, where reabsorption of Na+ is adjusted to match its excretion to dietary intake. Aldosterone is a steroid hormone secreted by the glomerulosa cells of the adrenal cortex. The regulation of aldosterone secretion is summarized in Fig. 3. The rate limiting factor in the cascade of events is the level of plasma renin, a proteolytic enzyme secreted by the juxtaglomerular cells, which cleaves a circulating protein, angiotensinogen, produced by the liver, to yield a 10-amino acid peptide, angiotensin I. Angiotensin I has no known physiologic function and is further cleaved to an 8-amino acid peptide, angiotensin II, by the angiotensin converting enzyme (ACE), found on the surface of vascular endothelial cells, mainly pulmonary and renal capillaries. Angiotensin II has several important physiologic functions: stimulation of aldosterone secretion by the adrenal cortex, arteriolar vasoconstriction, stimulation of Na+ reabsorption by the proximal tubules, and stimulation of both ADH secretion and thirst. Aldosterone acts on the principal cells of the distal nephron by binding to a cytoplasmic receptor (mineralocorticoid receptor, MR) that upon activation translocates to the nucleus and regulates gene transcription. Early effects (after 30 min latency) of aldosterone involve the recruitment of the apical Na+ channel (ENaC) and the basolateral Na+ -K+ ATPase. Later (after 4-6 hrs), the synthesis of these transport proteins is stimulated. These events lead to an enhanced reabsorption of Na+. [Aldosterone secretion is also stimulated by a rise in plasma potassium concentration, and it enhances the secretion of K+ by the distal segments of the nephron (see K+ Balance lecture).]

Tight junction permeability

Although tight junctions are impermeable to macromolecules, they are permeable to ions and water in most epithelia. Thus, the paracellular (between cells) pathway is an important route for the absorption or secretion of small solutes. All transport trough the paracellular pathway is passive in nature, driven by concentration differences and, for ions, also the transepithelial potential difference. The permeability of the tight junction to solutes and water differ among epithelia. Epithelia that exhibit a high rate of transport usually have high junctional permeability and low electrical resistance (leaky epithelia), such as the renal proximal tubule and the proximal small intestine (duodenum and jejunum). On the other hand, epithelia that establish large chemical gradients, like the renal collecting duct or the colon, have junctions with low permeability and higher electrical resistance (tight epithelia). At least 27 different claudins are identified in mammals. The specific claudins expressed by the epithelial cells determine in part the permeability and selectivity of the paracellular pathway to solutes and water. In addition, the claudins are attached to scaffolding and cytoskeletal proteins (see Fig. 1) allowing regulation of junctional permeability by the cell metabolism. For example, it has been observed that the junctional permeability to glucose in the small intestine is increased when there is a high rate of transepithelial glucose transport. Also, the paracellular calcium absorption by the ileum is upregulated by vitamin D, a process that indicates alteration in the selectivity of the pores formed by adjoining claudin molecules. Thus, although transport through the paracellular pathway is passive, it exhibits selectivity and physiological regulation.

The glomerular filtration rate (GFR) provides the best assessment of the state of renal function. It is thus important to understand how GFR is measured and the limitations of the techniques. As explained in lecture, the best estimate of GFR is obtained using the clearance of inulin. The following example shows you how to calculate clearances, GFR, renal plasma flow, as well as the relation between filtered load and the excretion rate of certain substances. You should be able to understand the explanations below. Sample problem: Given the experimental data below: Plasma conc Urine conc. Px (mg/ml) Ux (mg/ml) Inulin .12 6.0 PAH .02 5.0 Urea .20 5.0 Fructose 3.00 10.0 Hematocrit = 45% Urine flow rate ( 𝑽𝑽̇ ) = 2.0 ml/min. Evaluate the following: a. The glomerular filtration rate. b. The effective renal plasma flow. c. The effective renal blood flow. d. The filtration fraction.

An

B. What will happen to the rate of tubular reabsorption if, as a consequence of an increase in filtration fraction, GFR rises? Explain the mechanisms responsible for the response.

An increase in the rate of filtration is matched by a proportionate increase in the rate of reabsorption. This is a consequence of Glomerulo-Tubular balance, which as explained in class (see session 7, p.6) involves two major mechanisms. One mechanism increasing proximal tubule reabsorption involves the delivery of a larger filtered load of solutes, including substances like glucose, amino acids, bicarbonate and phosphate that are co-transported with Na+. Since there is a high capacity for reabsorption of these co-transported solutes, there would be greater reabsorption of the solutes together with Na+ and water. The second factor involved in GT balance is the Starling forces in the peritubular capillaries. An increase in filtration fraction will amplify the increase in protein concentration and oncotic pressure in the glomerular and peritubular capillaries. This would facilitate movement of reabsorbate from the interstitial 2 space into the blood. G-T balance takes place mostly at the level of the proximal tubule, although reabsorption of salt at the TAL is also load dependent. As a result of G-T balance the percent reabsorption in the proximal tubule is kept relatively constant. For example, if the filtered load of Na+ is near 18 mEq/min, the proximal tubule reabsorbs about 12 mEq/min or 2/3 of the filtered load. The remaining 6 mEq/min are delivered to the loop of Henle. Let's say that the filtered load increases to 21 mEq/min (a very large increase, just for the sake of the argument). The mechanisms involved in G-T balance will maintain the reabsorption at near 2/3 of filtration and thus about 14 mEq/min of Na+ are now reabsorbed and 7 mEq/min are delivered to the loop of Henle. Thus the load increased by 1 mEq/min instead of 3 mEq/min if there were not G-T balance. Consequently, GT balance reduces but does not completely prevent the increase in the delivery of fluid and solutes to the distal nephron (distal tubules and collecting ducts), which has a limited capacity for reabsorption. GT balance minimizes the loss of fluid and solutes that would occur with unopposed increases in GFR.

Problem 2 The following table lists the rates of renal clearance of a number of drugs in relation to their concentrations in the plasma of a human subject. All values are corrected for binding to plasma protein. The last column represents the simultaneous clearance of PAH at a plasma concentration permitting extraction of virtually all of the PAH in one circulation through the kidney. For statements a-d, select the drug that best fits the conditions described. (Assume that these drugs are filtered freely at the glomerulus, and that the GFR is about 130 ml/min.) a. Partially reabsorbed by the renal tubules. b. Secreted by the renal tubules. c. Excreted without tubular reabsorption or secretion. d. Secreted by the tubules but limits its own excretion by decreasing renal blood flow.

Answers to Problem 2: a - drug 4. A drug which is filtered freely and partially reabsorbed would be expected to show a clearance lower than the GFR. b - drug 2. A drug which is filtered freely and also secreted will show a clearance greater than the GFR. c - drug 1. A drug which is filtered freely at the glomerulus without tubular reabsorption or secretion will have a clearance equal to the GFR. (This of course is the basis for the use of inulin for the evaluation of the GFR.) d - drug 3. A drug which is freely filtered and also secreted will show a clearance greater than the GFR. If high concentrations decrease renal plasma flow, as Pdrug increases the CPAH will fall. An alternative explanation is that this drug uses the same secretory transporter as PAH, and thus it competitively inhibits PAH secretion leading to a decreased PAH clearance.

A. Starting with a GFR of 150 ml/min, calculate the volumes of fluid reabsorbed by the tubular segments numbered in the figure below and excreted, during: a) antidiuresis, with maximal interstitial osmolality of 1200 mOsm b) diuresis, with maximal interstitial osmolality of 600 mOsm Assume: 1) plasma osmolality of 300 mOsm; 2) tubular fluid osmolality at the end of the loop of Henle of 100 mOsm; 3) water reabsorption does not significantly alter interstitial osmolality

Antidiuresis Reabsorbed Moving into next segment 1) Proximal Tubule 100 ml 50 ml 2) Descending limb 37.5 ml 12.5 ml 3) Distal tubule and CCD 8.3 ml 4.2 ml 4) Medullary Collecting Duct 3.2 ml 1 ml 5) Urine 1 ml Diuresis Reabsorbed Moving into next segment 1) Proximal Tubule 100 ml 50 ml 2) Descending limb 25 ml 25 ml 3) Distal tubule and CCD 0 25 ml 4) Medullary Collecting Duct 0 25 ml 5) Urine 25 ml Proximal tubule reabsorption was estimated as 2/3 of GFR in both conditions. In other segments, reabsorption was calculated assuming osmotic equilibration with the surrounding interstitium. Although this is an oversimplified model, it shows that: a) during antidiuresis there is actually more water being reabsorbed at the DT and CCD than at the MCD, although the interstitial osmolality is higher in the latter. Keep in mind that there is some water reabsorption in the CCD even in the absence of ADH, and thus the actual amount of urine excreted would be less than 25 ml (about 15 to 20 ml) in diuresis, b) the volume of water reabsorbed by the descending limb increases with high ADH (antidiuresis). This happens, not because ADH affects the permeability to water of this segment but because the osmotic driving force (interstitial osmolality) is larger in antidiuresis.

Free water clearance

As discussed above, the kidneys contribute to the stability of the plasma osmolality by excreting or reabsorbing water without solute. To measure the amount of solute-free water that the kidney can excrete per unit time, one can calculate the free-water clearance. This is an unfortunate name because this quantity is not defined as a clearance, but rather as CH2O = •V - COSM [1] where •V is the rate of urine flow and COSM is the osmolar clearance, i.e., the clearance of total solutes (osmoles) from plasma by the kidneys: COSM = UOSM ·V / POSM [2] or CH2O = •V (1- UOSM / POSM It can be seen that when the urine osmolality is equal to that of plasma (UOSM = POSM), the free water clearance, CH2O, is zero (and COSM = 𝑉𝑉̇). On the other hand, with hypoosmotic urine,, UOSM < POSM, CH2O is positive and water would have to be removed from the urine to make it isosmotic with plasma. With hyperosmotic urine, UOSM > POSM, CH2O is negative and it would be necessary to add water to make the urine isosmotic. Of course, the negative free-water clearance represents the free water reabsorbed by the tubules rather than excreted. Another interpretation of free-water clearance can be obtained by rearranging Eq.1 as V = COSM +CH2O The total urine volume can be viewed as having two virtual components. One component, COSM, contains all the urine solutes in a solution that has an osmolality equal to that of plasma. The second component, CH2O, is a volume of solute-free water that makes the urine dilute if it is excreted, or concentrated, if it is reabsorbed. The concept of free-water clearance provides a way to quantify the ability of the kidneys to generate solute-free water (a volume of water that is free of all solutes). When the urine is dilute, this solute-free water is excreted from the body (CH2O is positive). When the urine is concentrated, this solute-free water is returned to the systemic circulation (CH2O is negative).

Regulation of Filtered Sodium

As stated above, the filtered load of Na+ is principally a function of GFR, since plasma Na+ concentration varies little. Although hemodynamic mechanisms (RBF, glomerular capillary pressure) tend to maintain GFR, as previously reviewed, changes in body Na+ , and thereby ECF and plasma volume, lead to changes in GFR. Increases in Na+ tend to increase GFR, while loss of Na+/ECF volume tends to reduce GFR and the filtered load of Na . These effects are mediated by neurohumoral factors Sympathetic nerves innervate afferent arterioles. Enhanced sympathetic traffic due to decreased Na+ /ECF volume causes arteriolar vasoconstriction that tends to lower glomerular capillary pressure and, therefore, to reduce GFR. Angiotensin II constricts primarily the efferent arteriole and tends to maintain GFR in the face of low renal perfusion pressures. But when arteriolar constriction is more pronounced, renal perfusion drops to a value that decreases GFR. On the other hand, ANP and NO are released in response to increased ECF volume. ANP dilates the afferent arteriole, while constricting the efferent arteriole; thus, it triggers a potent mechanism to increase GFR. Nitric oxide (NO), produced locally by vascular endothelium, dilates renal vasculature. Although changes in GFR do occur with alterations in ECV, they are not required to maintain Na+ balance. Many patients with substantial reduction in GFR are able to maintain Na+ balance by decreasing the rate of tubular reabsorption, primarily because of GT balance. In general, the changes in tubular reabsorption constitute the main response to fluctuations in ECV.

Micturition (urination)

At rest urinary continence is maintained by sympathetic signals that relax the bladder wall and constrict the internal urethral sphincter, resulting in an inhibition of bladder emptying. As the bladder fills, stretch receptors in the bladder wall are sensed by GVA (parasympathetic) fibers in the pelvic splanchnic nerves. Parasympathetic motor (GVE) fibers in the same nerves then induce a reflex contraction of the detrusor muscle and relaxation of the internal urethral sphincter, which induces urination to occur. However bladder emptying can be prevented by the external urethral sphincter, which remains closed at rest. When the individual chooses to urinate, general somatic efferent signals from the pudendal nerve cause voluntary relaxation of the external urethral sphincter, allowing the bladder to void. When complete, the external urethral sphincter close and in males the bulbospongiosus muscles contract to expel the remaining urine from the urethra.

In both male and female: Internal urethral sphincter • Continuation of detrusor (bladder) muscle; consists of smooth and striated muscle • Under autonomic nervous control External urethral sphincter • Skeletal muscle • Under somatic nervous control (pudendal nerve)

At rest urinary continence is maintained by sympathetic signals that relax the bladder wall and constrict the internal urethral sphincter, resulting in an inhibition of bladder emptying As the bladder fills, stretch receptors in the bladder wall are sensed by GVA (parasympathetic) fibers in the pelvic splanchnic nerves. Parasympathetic motor (GVE) fibers in the same nerves then induce a reflex contraction of the detrusor muscle and relaxation of the internal urethral sphincter, which induces urination to occur. However bladder emptying can be prevented by the external urethral sphincter, which remains closed at rest. When the individual chooses to urinate, general somatic efferent signal via pudendal nerve cause voluntary relaxation of the external urethral sphincter, allowing the bladder to void Urinary system is retroperitoneal and is well protected by ribs • BUT retroperitoneal nature makes it difficult to visualize (esp ureters) so vulnerable to injury during surgical procedures • Kidney pain is typically perceived as flank pain, whereas ureteric pain radiates to groin

Natriuretic and Local Factors

Atrial Natriuretic Peptide (ANP): 28-amino acid peptide produced and stored by atrial myocytes; it relaxes vascular smooth muscle and promotes NaCl and water excretion by the kidneys. ANP is released with atrial stretch, as occurs with expansion of the ECF. The actions of ANP antagonize those of the renin-angiotensin-aldosterone system: o Increases GFR and Na+ filtered load o Inhibits renin secretion by juxtaglomerular cells o Inhibits aldosterone secretion by the adrenal cortex o Inhibits (directly) Na+ reabsorption by the collecting duct o Inhibits ADH secretion by the posterior pituitary • Other cardiac natriuretic peptides found more recently are brain natriuretic peptide (BNP), and C-type natriuretic peptide (CNP). The serum levels of BNP are elevated in patients with left ventricular dysfunction and appear to have prognostic value since they correlate with the severity of heart failure. • Urodilatin: 32- amino acid peptide secreted by distal tubules and collecting ducts. Acts locally inhibiting Na+ reabsorption by these segments. • Local factors: NO, prostanoids and kinins are produced in the kidney and inhibit Na+ reabsorption.

Neurohumoral Regulation of RBF and GFR

Autoregulation is likely most important in the day-to-day regulation of renal hemodynamics in normal subjects. In many patients, however, the renal artery pressure is mostly reduced because of a fall in the effective circulating volume (ECV), whether because of a true volume depletion or congestive heart failure. Recall from the CVS section that in those settings there is a marked stimulation of the vasoconstrictor sympathetic nervous and renin-angiotensin systems. A decrease in blood volume or blood pressure leads to sympathetic nerve activation. The released norepinephrine (either from the nerve endings or the adrenal medulla) interacts with α-1 receptors, and causes constriction, mostly of the afferent arterioles, and reduction in RBF and GFR. Sympathetic activation of β- 1 receptors in granular cells in afferent arterioles stimulates the release of renin. Decreased perfusion pressure and decreased stretch of afferent arterioles also lead to increased production of renin and resultant formation of AII, which increases the resistance in the efferent and to a lesser degree the afferent arteriole. The result is a decrease in RBF, but the effect of these changes in GFR varies with the degree of neurohumoral activation. Both norepinephrine and AII stimulate renal prostaglandin (mainly PGE2 and PGI2) production. The relaxing action of prostaglandins dampens the vasoconstrictor effects of the sympathetic and AII activities. This prevents the renal ischemia that may be induced by the vasoconstrictors, particularly AII, which may be at a much higher local concentration than in the systemic circulation. A clinical consequence of the protection by the prostaglandins is seen when nonsteroidal anti-inflammatory drugs (NSAIDs) are given to patients with high AII and norepinephrine levels (because of volume depletion). Under these conditions, blockade of prostaglandin synthesis by the NSAIDs leads to acute reductions in RBF and GFR because of the unopposed vasoconstriction. Another vasoconstrictor of afferent and efferent arterioles is endothelin, produced by endothelial and mesangial cells; although it may not play a role in regulating renal hemodynamics in normal subjects, it is elevated in some renal diseases. Nitric oxide, produced by endothelial and macula densa cells, is a vasodilator released tonically in the renal circulation. Expansion of the ECF volume and several circulating factors (ATP, bradykinin, histamine) stimulate the release of NO. The levels of dopamine, synthesized by proximal tubule cells from circulating L-dopa, and of atrial natriuretic peptide (ANP), are also increased with volume expansion. These substances are renal vasodilators that lead to increases in RBF and GFR. ANP appears to produce the unusual combination of dilating afferent and constricting efferent arterioles, raising GFR with less of a change in RBF.

Explain severe hemorrhage/decompensation (ie progressive shock) Severe Hemorrhage and Decompensation (Progressive Shock)

Blood loss over 30% for 3-4 hrs before transfusion is decompensation, progressive shock smeaning decline will occur. Gray skin: vasoconstriction/low hemoglobin in capillaries/venous plexus. Cold sweat/goose bumps: increased sympatheti activity Mouth dryness: vasoconstirction of salivary gland Rapid breathing: reflex of anoxic carotid/aortic bodies Thin pulse: Low SV due to Low VR and Rapid HR Low UO: retention of salt/water Prolonged hemorrhagic hypotension: vasoconstriction in all but skin reverses due to depletion of NE and pressor area depression. Long term hypotension causes vasodlation further dropping BP and limiting perfusion. In this decompensated stage of hypovolemic shock there is stall of circulation, fluid leakage into the interstitium due to restored capillary hydrostatic pressure from vasodilation, cardiotoxic shock factors.

The diagram below indicates how renal blood flow (RBF) and the glomerular capillary pressure (PGC) are affected by the degree of constriction of these arterioles. Constriction of either arteriole increases the resistance to flow and reduces RBF. The afferent resistance determines the degree to which the renal arterial pressure is transmitted to the glomerulus and thus afferent dilation (less resistance) increases PGC, while constriction leads to reduced PGC. Since the glomerular filtration rate increases with both the RBF and PGC, its response to changes in afferent resistance is straightforward: afferent dilation increases GFR while constriction decreases it. On the other hand, constriction of the efferent arteriole increases PGC. This leads to an increased GFR unless the efferent constriction reduces RBF to a level where filtration gets compromised

Both arterioles are innervated by sympathetic nerves (the kidney has a rich sympathetic innervation, particularly of the arteriolar smooth muscle and the tubular system, but not parasympathetic innervation).and have differential sensitivities to angiotensin II and other vasoactive substances, providing the mechanism of the physiological regulation of RBF and GFR, as described next.

Anatomy of the Urinary System

Both kidneys lie deep to ribs 11 and 12 Inferior pole of the right kidney is approximately 1-2 cm above the iliac crest. Right kidney is slightly more inferior than is the left kidney The ureters are muscular tubes that run inferiorly from the apex of the renal pelves at the hila of the kidneys, pass over the pelvic brim at the bifurcation of the common iliac arteries to run along the lateral wall of the pelvis and enter the urinary bladder. The entire path is retroperitoneal. The ureters are normally constricted to a variable degree in three places: Ureters 1. At the junction of the ureters and renal pelves, 2. Where the ureters cross the brim of the pelvic inlet, and 3. During their passage through the wall of the urinary bladder. These constricted areas are potential sites of obstruction by ureteric (kidney) stones Nephrolithiasis (kidney stones) is a common disease that refers to calculi in the kidneys. The caliculi can leave the kidney and pass through the ureter into the bladder, or get caught within the ureter

21. The concentrations of inulin and a freely filtered organic acid in the luminal fluid collected from the end of the proximal tubule of a normal experimental animal are 4 times and 3 times higher than in plasma, respectively. You can then safely conclude that: A. The clearance of the organic acid is less than the clearance of inulin. B. The proximal tubule reabsorbs 2/3 of the filtered water. C. The proximal tubule reabsorbs the organic acid. D. The clearance of the organic acid is more than the clearance of inulin. E. The proximal tubule secrets the organic acid.

C. The proximal tubule reabsorbs the organic acid.

54. In a person losing a significant amount of gastric fluid due to prolonged vomiting all of the following will happen EXCEPT: A. Plasma bicarbonate will be elevated B. ADH secretion will be stimulated C. The urine pH will be very low D. Hypoventilation E. Protons will leave cells in exchange for potassium.

C. The urine pH will be very low

6. Which of the following statements is correct? A. Antidiuretic hormone (ADH) induces incorporation of aquaporins in lumenal membranes of the proximal tubule. B. ADH increases the water permeability of ascending vasa recta. C. With extreme salt and water depletion ADH secretion promotes renal water reabsorption even though this may lower plasma osmolality below normal. D. ADH increases the solute permeability of ascending vasa recta. E. ADH reduces medullary collecting duct urea permeability

C. With extreme salt and water depletion ADH secretion promotes renal water reabsorption even

The clearance of PAH measures the renal plasma flow

Consider now a substance like para-aminohippuric acid (PAH). Like inulin PAH is freely filtered and not reabsorbed. However, unlike inulin, the PAH in the peritubular capillaries that escaped filtration is avidly secreted into the tubular fluid. Indeed, if the secretory process is not saturated, all of the PAH in the plasma delivered to the kidneys is removed and excreted into the urine in a single pass. The amount of PAH delivered into the kidneys per unit time is PPAH· RPF, where RPF is the renal plasma flow. The amount excreted per unit time is, as before, UPAH · V̇ . Then PPAH · RPF = UPAH · V and solving for RPF: RPF = UPAH · V̇ = CPAH [8] / PPAH That is, the clearance of PAH is the RPF. Low concentrations of PAH (<0.12 mg/ml) must be used so that the tubular maximum for PAH secretion is not exceeded and all the PAH arriving to the kidneys is excreted (see next lecture). Note that this clearance represents an effective renal plasma flow since a small amount (ca. 5%) of the renal blood perfuses extra-glomerular areas of the kidney.

Blood Supply

Consider the spatial organization of the blood vessels (angioarchitecture) in the kidney. Refer to Figures 2 and 3 and return to slide 453 and slide 455. These slides are not perfusion-fixed tissue sections, so blood cells remain in many of the blood vessels, facilitating their identification. The rodent kidney tissue section is especially good for visualizing the smaller vessels and extensive capillary networks In the kidney, the larger arteries and veins travel into the cortex together; arterial branches supply the capillary systems and veins drain the capillary beds. Location and orientation are key to the following identifications. These systemic vessels have the expected morphology of arteries and veins: the arteries have a distinct smooth muscle media and internal elastic lamina and the veins are thinwalled; in fact, they are unusually thin-walled in the cortex. First, observe large blood vessels in the renal sinus on either side of the renal papilla(ae). Vessels in this location are interlobar (between lobes) arteries and veins. In a multi-lobed kidney such as in humans, interlobar vessels are oriented radially and course between (not within) the medullary pyramids. These interlobar arteries give rise to smaller branches that arch along the interface between the medulla and cortex, the arcuate arteries. Observe such arcuate arteries and veins in these tissue sections: unlike the interlobar vessels the arcuate vessels are surrounded by cortex because they travel just beyond the interface between cortex and medulla and they are oriented relatively parallel to the kidney surface Figure 2: Simplified diagram of the blood vessels in three lobes of the kidney. The difference in the second capillary bed associated with a subcapsular glomerulus (left), a regular glomerulus (middle), and a juxtamedullary glomerulus (right) are illustrated. (DWV)

Capillary beds of the kidney

Cortical glomerulas (PTCP) is 75% of glomeruli, it sits far from the corticomedullary border, contacts a cortical nephron which only reaches the outer medulla, sends blood to the peritubular capillary plexus. The PTCP is the capillary bed of the cortex that can drain to the interlobular vein or stellate vein. Vasa recta is the capillary bed of the medulla which returns to the juxtamedullary region to drain interlobular and arcuate veins. Juxtamedullary Glomerulus (VR) (25%) sit neat the corticomedullary bored and contact the "juxtamedullary nephron" which goes into the inner medulla and contributes blood to the vasa recta. Nephrons are cortical or juxtamedullary based on glomeruli connecting to PTCP vs. VR.

B. A patient plasma creatinine concentration rose to 3.2 mg/dL due to the fall in GFR. The following additional values were obtained (Note that the attending physician lost the urine flow data): UNa 85 mEq/L PNa 140 mEq/L Ucreat 112 mg/dL What was the fraction of filtered water that was reabsorbed? Do you think that this result is consistent with the suspected tubular injury?

Creatinine and water are filtered at the same rate but only the water gets reabsorbed. Therefore, the extent to which creatinine gets concentrated in the urine with respect to its concentration in plasma is an indication of the fraction of filtered water that gets reabsorbed. The urine creatinine concentration is 112 mg/dL. The plasma concentration is 3.2mg/dL. Thus the U/P ratio for creatinine is 112/3.2 = 35. Since the filtered creatinine was concentrated 35 -fold (i.e., concentrated into 1/35th of the filtered volume), 34/35 (= 0.971 = 97.1%) of the filtered water must have been reabsorbed. A fractional reabsorption of 97 % for water is low, since typically more than 99% of the filtered water gets reabsorbed. It does indicate reduced tubular function.

8. A patient is found to have a very low serum Na+ concentration. Which of the following conclusions can be drawn? A. She has low total body Na+ content. B. She has low total body Cl - content. C. She has high total body water content. D. Her intracellular osmolality is likely to be low. E. All of the above inferences are likely to be true

D. Her intracellular osmolality is likely to be low.

10. Which of the following is likely to cause hypokalemia? A. Insulin deficiency B. Exercise C. Beta-adrenergic blockers D. Hyperaldosteronism E. Increased ADH secretion

D. Hyperaldosteronism

49. What is not true A. Because of autoregulation GFR is maintained near constant despite changes in arterial blood pressure between 90-180 mmHg. B. Because of autoregulation renal plasma flow is maintained near constant despite changes in arterial blood pressure between 90 and 180 mmHg. C. Tubulo-glomerular feedback contributes to autoregulation. D. Increases in afferent and efferent arteriolar diameter have similar effects on GFR. E. Increases in afferent and efferent arteriolar diameter have effects on renal plasma flow.

D. Increases in afferent and efferent arteriolar diameter have similar effects on GFR.

D. How does loop diuretic (salt transport in TAL to increase loss of Na/H2O) lead to hypokalemia?

D. Loop diuretics cause excretion of K by depresssing NaKCl2 cotransport out of the lumen of the TAL. The also depress transport that increases rate of Na and H2O delivery to distral nephron. Increases tubular blow increases K secretion via flushing so lumenal fuild K is still low enough to keep a K wash out gradient. Increased delviery of Na increases apical Na entry and NaK pump and increased basolateral K uptake to push out into tubular lumen. Enhanced lumenal Na influx depolarizes cell and increases electrical driving force of K secretion.

D. Over a period of several days the creatinine U/P ratio (i.e. urinary concentration/ plasma concentration) is observed to increase. Does this represent an increase in GFR? Explain.

D. Over a period of several days the creatinine U/P ratio (i.e. urinary concentration/ plasma concentration) is observed to increase. Does this represent an increase in GFR? If we ignore the small secretion of creatinine in humans, the (U/P)creat represents the extent to which the concentration of a filtered substance can be increased as a result of reabsorption of water. It tells us only about the fraction of filtered water reabsorbed by the tubules but it is not necessarily correlated with the GFR. (Note that we do not know what happens to the urine flow rate).

52. If the sodium intake of a person is abruptly reduced to near zero, all of the following will occur EXCEPT: A. The glomerular filtration rate will decrease. B. Aldosterone secretion will increase. C. Atrial natriuretic peptide secretion will decrease. D. Renal blood flow will increase. E. Body weight will decrease.

D. Renal blood flow will increase.

57. Which of the following statements is FALSE? Met acidosis assoc. w low plasma pH, low plasma HCO3, low plasma pCO2. Chronic acidosis increases urinary excretion of NH4 Hypoven incrases secretion of H. Pt w lower HCO3 in respirtory acidosis will have more ACUTE form. Urine pH 5.0, urinary NH4 is greater at 48 hr than 1 hr. A. Metabolic acidosis is associated with decreased plasma pH, decreased plasma [HCO3 and decreased plasma PCO2. B. Chronic acidosis promotes increased urinary excretion of NH4 +C. Hypoventilation promotes increased renal secretion of H+ D. Two patients suffering from respiratory acidosis have equal values of plasma PCO2. The patient with the lower plasma [HCO3 is likely to have the more chronic form of the disorder. E. A patient's urine pH was found to be 5.0 both 1 hour and 48 hours following ingestion of an acid. The rate of urinary NH4 + excretion was likely to be greater at 48 hours than at 1 hour.

D. Two patients suffering from respiratory acidosis have equal values of plasma PCO2. The patient with the lower plasma [HCO3 is likely to have the more chronic form of the disorder.

43. Which of the following statements is correct? A. The fractional water reabsorption in the proximal tubule exceeds that of solute reabsorption during dehydration. B. Lumenal solute concentration falls during passage of fluid along the descending limb of Henle's loop. C. During water diuresis the free water clearance CH2O exceeds the rate of urine flow D. With extreme depletion of salt and water ADH secretion promotes water reabsorption even though this lowers plasma osmolality below normal. E. Countercurrent exchange causes movement of solute from descending to ascending vasa recta.

D. With extreme depletion of salt and water ADH secretion promotes water reabsorption even though this lowers plasma osmolality below normal.

39. Which of the following most likely indicates the presence of disease in a human? A. a PAH clearance of 650 ml/min B. an inulin clearance of 130 ml/min C. a urinary osmolality of 1200 mOsm D. a glucose clearance of 15 ml/min E. a urinary pH of 5.5

D. a glucose clearance of 15 ml/min

What factors regulate K secretion into the collecting duct (aldosterone/plasma K) and how do they differe from the facotrs that change K secretion (lumenal flow, acid-base distrubance, anion delivery)?

DCT has 2 cells that change K secretion. The prinicpal/light cell is responsible for K secretion/Na reabsorption. The intercalated a-type cell controls K reabsorption. K secretion happens in the late distal tubule and cortical collecting tubules LDCT/CCD. Reabsorption in distal occurs in the medullary collecting duct MCD. To secrete into lumen, K is uptaken from the basolateral side via Na/K ATPase and diffuses to the apical lumen via ROMK/BK channels.Although K+ channels are also are in the basolateral memberane, there are more on the apical side and so K will prefer to diffuse to the apical side into tubular fluid. Princiaple cells (secreters) also have KCL cotransport in the apical side. K secretion through the apical KCL cotransport is modest but can increase if there is very low Cl in the tubular lumen due to poorly reabsorbed bicarb/phos anions in the LDCT and CD. 3 Factors Controll Rate of K+ Secretion: 1. Activity of Na-K ATPase 2. Electrochemical gradient for K movement into apical tubular lumen 3. Permeability of apical membrane K channels Changes in secretion of K can be caused by the following: 1. a-intercalated cells (reabsorbers) secrete H+ and uptake K using a H-K ATPase in the lumenal membrane. Similar pump to parietal cell of gastric mucosa for acid secretion. K exits into basolateral channel by diffusion in a-int cells. 2. Medullary collecting duct reabsorbs K passively through K-permeable tight-junctions when luminal K concentration rises due to high K+ intake or with high distal reabsorption of water (elevated plasma ADH). 3. Normal K intake of 90 mEq/day mens the secretion of K is in the DCT and rate of K excretion it 10% of filtration rate (630-900 mEq/day).. With high K intake, rate of excretion can exceed the rate of filtration. With K depletion, distal nephron reabsorbs K and excretion is reduced to 1% of the filtered load. Kidney cannot reduce K as low as Na (.2% of FL) thus hypokalemia is more likley for K def diet.

Regulation of ADH secretion

Dilution of body fluids (decreased osmolality) inhibits ADH secretion, whereas increased plasma osmolality or significant volume depletion of body fluids leads to increased secretion of ADH. ADH is secreted by certain hypothalamic neurons whose axons terminate in the posterior pituitary. They receive inputs from neighboring neurons that function as osmoreceptors (Fig. 8). These osmoreceptors are exquisitely sensitive to the plasma osmolality, increasing their firing rate when the osmolality increases and stimulating ADH secretion. Conversely, when plasma osmolality is low, the osmoreceptors' rate of firing decreases, and ADH secretion is reduced (Fig 10, left).

1. A subject is administered an intravenous solution containing low concentrations of inulin and PAH. In the steady state his plasma and urinary concentrations are reported to be: Pin = 1mg/100ml; PPAH = 1mg/100ml; Uin = 100mg/100ml; UPAH = 500mg/100ml. The record of the urine flow rate was lost! Which of the following is correct? A. From the data it is not possible to determine the fraction of filtered water that is reabsorbed. B. The data indicate that inulin is reabsorbed. C. The data show that the glomerular filtration rate is normal. D. The data indicate that PAH is not completely cleared in the kidney. E. The filtration fraction is approximately 1/5.

E. The filtration fraction is approximately 1/5.

E. What can you conclude about the tubular handling of a substance with a U/P ratio larger than that for creatinine?

E. What can you conclude about the tubular handling of a substance with a U/P ratio larger than that for creatinine? That substance must be secreted by the tubules. Since, as mentioned, the (U/P)creat represents the extent to which the concentration of a filtered substance can be increased as a result of reabsorption of water, a value of (U/P)X greater than (U/P)creat can be only as a consequence of secretion of X

37. Respiratory acidosis with partial renal compensation is characterized by: A. above normal pH, above normal plasma bicarbonate, above normal PCO2. B. normal pH, below normal bicarbonate, normal PCO2. C. below normal pH, above normal bicarbonate, below normal PCO2. D. below normal pH, normal bicarbonate, above normal PCO2. E. below normal pH, above normal bicarbonate, above normal PCO2.

E. below normal pH, above normal bicarbonate, above normal PCO2.

31. Which of the following will increase glomerular filtration rate? high ANP low symp output to kidney high arterial pressure low plasma protein concentration kidnye stones will decrease gfr A. an increase in the sympathetic output to the kidneys B. a fall in arterial pressure C. an increase in plasma protein concentration D. ureteral blockage by kidney stones E. elevated plasma atrial natriuretic peptide (ANP) concentration

E. elevated plasma atrial natriuretic peptide (ANP) concentration

34. In increasing order the plasma clearance for the following substances would normally be: A. creatinine, urea, inulin, sodium, glucose. B. sodium, inulin, creatinine, urea, glucose. C. glucose, sodium, inulin, creatinine, urea. D. glucose, urea, sodium, creatinine, inulin. E. glucose, sodium, urea, inulin, creatinine.

E. glucose, sodium, urea, inulin, creatinine.

Cellular mechanisms of sodium reabsorption

Entry of Na+ into the tubular cells across the luminal membrane is energetically a downhill process but the actual transporter varies along the tubule. Exit from the cell across the basolateral membrane is always uphill, energized by the Na+-K+ ATPase, and to a lesser extent through the 3HCO3/Na+ cotransporter (NBC) in early proximal tubule (PT) and TAL (see Fig 3). At the early proximal tubule, the preferential mode of entry of Na+is via cotransport with organic solutes (glucose, amino acids, lactate, phosphate, etc.). Na+ is also reabsorbed in exchange for H+ (NHE3 transporter). In the late proximal tubule, coupling of the Na+-H+ exchanger with a Cl- -OH-(or Base-) exchanger results in the net absorption of NaCl. About 2/3 of the salt filtered at the glomerulus is reabsorbed at the proximal tubule. Although the descending limb of the loop of Henle is rather impermeable to Na+ , the thin ascending limb of the loop of Henle is permeable to this ion. Passive reabsorption of Na+ takes place at the thin ascending limb, through a paracellular pathway (see Osmoregulation lecture). In the TAL Na+ is cotransported with K+ and Cl- by the NKCC2 transporter (sensitive to loop diuretics) in a stoichiometry of 1Na+ :1K+ 2Cl- ; the downhill entry of Na+ energizes the uptake of K+ and Cl-. Sodium is also transported in exchange for H+ (not shown in Fig 5). At the early distal tubule Na+ is cotransported with Cl- (NCC transporter, sensitive to thiazides), and at the late distal tubule and collecting duct Na+ entry is via an epithelial Na+ channel (ENaC), sensitive to amiloride and upregulated by aldosterone.

Describe extra-renal potassium homeostasis

Excretion of K is after ingestion or infusion and is slow, plasma K is regulated by the cellular uptake of K to prevent acute hyperkalemia. Ex. Average eaten 33mEq K in ECM would lead to a concentration of 2.4mEq/L increase that can cause fatal arrhythmias. It is uptaken via certain hormones.

Finally, whereas ADH is a prime factor in short-term regulation, other as yet incompletely characterized factors contribute importantly to long-term regulation of the concentrating mechanism, since extended periods of dehydration produce substantially higher urine osmolality than that following administration of a long-acting preparation of ADH.

Fig 10 Left: Plasma ADH as a function of plasma osmolality. Right: Plasma ADH as a function of percentage blood volume depletion. Note the high levels of plasma ADH when blood volume decreases by more than 10%. Although it is natural to focus on the mechanisms directly involved in creation of osmotic gradients, it is important to realize that there are quantitative limits on the capacities of these systems, and so it is essential that there be mechanisms for reabsorption of large volumes of isosmotic filtrate in the renal cortex in order to establish large gradients in the medulla. As you know, this occurs largely in the proximal tubules, where there are luminal and basolateral membrane water channels ("aquaporins") that are not regulated by ADH. The importance of proximal tubular water reabsorption is indicated by studies in "knockout" mice and in humans lacking the aquaporin AQP1. The mutants absorb less than normal quantities of glomerular filtrate and are unable to maintain water balance during dehydration, even when given an analogue of ADH. To reiterate: changes in plasma osmolality triggers body responses (thirst, urine output) that tend to restore the body fluid osmolality to normal. The renal response is dependent on the secretion of ADH. Although osmolality is the chief factor regulating the secretion of ADH, other factors are also significant.

The classic experiments of Verney (Fig. 2) demonstrated that plasma hyperosmolality results promptly in the secretion of a neurohypophyseal substance (ADH), with resultant decrease of urine volume (and increase of urine concentration).

Fig 2 Summary of results obtained on dogs by E.B. Verney (Proc. Roy. Soc. London, Series B. 135:25, 1947). Dotted line represents results on a dog prior to removal of the neuro- hypophysis; the solid line represents results on the same dog after neurohypophysectomy. Water was given by mouth; hyperosmotic saline and pituitary extract were added to the blood stream. The regulation of plasma osmolality and its main determinant, the plasma sodium concentration1 , is achieved by alterations in water intake and excretion. Slight hyperosmolality of body fluids stimulates the hypothalamic thirst center and normally leads to water ingestion, tending to return osmolality towards normal. Likewise, hyperosmolality stimulates the secretion by the neurohypophysis of a potent antidiuretic hormone (ADH) that modulates urine volume and concentration. These two mechanisms are needed to maintain water balance, since even with normal renal function, obligatory renal and non-renal ("insensible") water losses would lead to progressive negative water balance, if not for thirst.

Renal lobe. Each medullary pyramid is capped by cortex that arches over the base of the pyramid and extends inwardly between the pyramids towards the renal sinus as a renal column. The region of cortex close to the medulla is termed juxtamedullary cortex. One medullary pyramid and the associated cortex, including half of the adjacent renal columns, constitute a renal lobe.

Figure 1: Diagram illustrating the parts and regions of the human kidney as visible in a midsagittal section. The renal lobe and lobule are defined. The renal sinus is the connective-tissuefilled light space around the dark-shaded calyces in the interior of the organ. (DWV) Examine slide 455 for more histological details. This is a rodent kidney; the rodent has just one lobe per kidney. In this VM slide, the cortex wraps around a central medulla forming a 'U' whose open end is at the hilum. The medulla is the pale oval region in the center of this tissue section. As in the human, the medulla is a cone-shaped structure known as the medullary (renal) pyramid. As depicted in this slide, the renal papilla is capped by a clear crescent lumen whose outer wall is the calyx. In the renal sinus, beyond the calyx and between the extended cortical extensions, the ureter arises from the calyx in the manner illustrated by the lines drawn on the VM image.

There are two options for the second capillary bed: the one that arises from the efferent arteriole depends on the location of the glomerulus. The efferent arteriole arising from cortical glomeruli gives rise to a peritubular capillary plexus The efferent arteriole of glomeruli located near the medulla (known as juxtamedullary glomeruli) gives rise to the capillaries of the vasa recta that extend into the medulla in parallel with the straight portions of the nephron (the loop of Henle). Most of the vasa recta capillaries drain into the arcuate veins, whereas the capillaries of the peritubular capillary plexus drain into the interlobular veins. From these veins, the blood flows to the interlobar veins, the segmental veins, and out of the kidney.

Figure 9: The efferent arteriole of a glomerulus located in the mid to outer cortex, gives rise to the peritubular capillary plexus, which drains into the interlobular vein.

Tubular Potassium Transport PT reabsorption is paracellular

Filtered load: 630-900 mEq/day K Filtered Load= [K] * GFR = [3.5 - 5] * 180 PT = 80% K paracellular reabsorption secondary to water and idffusion Loop diuretics furosemide on thick ascending prevent 10% further reabsorption DT and CD = reabsorb (Principal cell) or secrete (intercalated cell) (1% to ++100%)

3. Which of the following statements is correct? A. The filtration coefficient Kf of glomerular capillaries is about the same as that of extra-renal capillaries. B. An increase of the filtration fraction of glomerular capillaries will lower both the hydrostatic pressure and the colloid osmotic pressure of plasma in peritubular capillaries. C. Constriction of the efferent arteriole will decrease the glomerular filtration rate and decrease renal plasma flow. D. Cortical nephrons have long descending and ascending loops of Henle running into the inner medullary zone. No only JxMed E. Hydrostatic pressure varies less along the course of glomerular capillaries than in extra-renal capillaries.

Filtration Fraction, FF = GFR/RPF Uin/Upah E. P Hydrostatic pressure varies less along the course of glomerular capillaries than in extra-renal capillaries.

The filtration barrier is fenestrated endo of glom cap, visceral epith of bowman podocyte, fused basal lamina, filtration slit of podocyte. 3 layer barrier: lamina rara externa near podocyte, lamina dense of 2 fused basal laminae, lamina interna of endothelial cells. Ultrafiltration 125ml/min protein under 30kD pass unless neg charge bc basement membrane is neg charge. Water, glucose, ions, amino acids, and urea pass freely into Bowman's space; serum albumin and the formed elements of blood are restricted. Thus the ultrafiltrate in Bowman's capsule = blood plasma.

Filtration barrier between lumen of glom cap and urinary space

Endogenous Acid Production

Fixed: Low O2 (ketoacid, lactic acid, uric acid), Low Insulin, High Protein excreted as HA + NH4+ (excreted for reabsorbed HCO3) up to 70mEq/day Volatile: CO2 + H2O 20M/day

Factors Affecting Potassium Excretion

Flow of tubular fluid - a rise in the flow of tubular fluid happens during diuretic treatment or expansion of ECF which increases K secretion quickly. A lower tubular flow due to ECF contraction/hemorrhage/vomiting/diarrhea reduces K secretion at the distal nephron. A high flow rate promotes K secretion because it deforms mechanosensitive cilium in principal cells to open Ca2+ intracellular thus opening Ca-sensitive K channels (BK channels). A high tubular flow rate favors a high gradient for K secretion by washing away screted K leaving lumenal K concentrations very low. Stimulation of K secretion is also affected by the Na content delivered to the distal nephron. With more flow in distal segments, a higher delivery of Na will cause increase reabsortion of Na. More reabsorption of Na causes more secretion of K due to activation of the basolateral Na-K ATPase drawing in more intracellular K + depolarizing the lumenal membrane to secrete K.

Calculation of filtration and excretion rates

For any given substance, X, the amount excreted by the kidney (per unit time) equals the amount filtered minus the amount reabsorbed plus the amount secreted (per unit time): EX = FX - RX + SX [2] Since the GFR is the volume of plasma filtered per unit time, the amount filtered per unit time (filtered load) of any freely filtered substance, X, is: FX = PX · GFR [3] [mass= concentration · volume or mass/time = concentration · (volume/time) ] where PX is the plasma concentration of X. Similarly, if the urine concentration of X is UX and the urine flow rate is 𝑉𝑉̇ (ml/min), the amount of X excreted per unit time is given by: EX = UX ∙ V̇ [4] Since the quantities in Equations [3] and [4] can be readily measured for any of the plasma components (see later for the measurement of GFR), we can compare the rate of filtration with the rate of excretion of any freely filtered substance. Thus, if EX<FX, we must conclude that there was net reabsorption of X by the tubules. On the other hand, if EX > FX, X was secreted by the tubules. Note that this computation tell us only about the net effect; it is possible that a substance X is both reabsorbed and secreted, but as long as the reabsorption exceeds secretion EX<FX.

The Kidney

General overview: human kidney. Examine slide 453, a midsagittal section through a newborn human kidney. Observe the indented hilum (hilus) in the middle of the top of the VM section; this is where the renal blood vessels enter and leave. The ureter originates in the hilum; in this tissue section, the ureter is visible in this region. The kidney cortex (outermost thick region; L., bark) is darkly stained and, as appropriate for an immature kidney, has a lobed surface. With advancing maturity the surface of the human kidney becomes smoother as cortical mass continues to develop and fill in the furrows between lobes. The kidney's medulla (interior; L., marrow) extends from the cortex towards the interior of the kidney and it appears less intensely stained than the surrounding cortex. With regard to its three-dimensional geometry, it is a series of cone-shaped structures, each known as a medullary (renal) pyramid whose tip is the renal papilla. The human kidney typically has 6 to 14 renal pyramids, with 8 being most common. Each renal papilla is capped by a structure that collects the urine and is attached to the parenchyma of the medullary pyramid. Observe that five of these renal papillae in this VM slide are bordered by a clear crescent lumen formed by a dense-appearing wall. This epithelium-lined dense connective tissue structure is a minor calyx (calyx, G. cup of a flower). It is shaped like a funnel that caps the top of the renal pyramid. Examine Figure 1 to see that several of these funnel-shaped minor calyces unite with their neighbors to form two or three larger chambers, the major calyces. The major calyces fuse to form the funnel-shaped renal pelvis, which traverses the hilum, tapers, and becomes the ureter. One ureter from each kidney carries urine to the urinary bladder. The kidney has a dense connective tissue capsule that lines the external surface of this organ. There is no surface mesothelium because the kidney is embedded in perirenal (perinephric) fat tissue. The region containing loose or adipose connective tissue within the kidney, but outside of the lobes, is termed the renal sinus. Notice that the renal sinus is physically separated from the medullae by the calyces. The renal pelvis, ureter, and major arteries and veins travel within the renal sinus before exiting at the hilus.

Glucose reabsorption

Glucose is reabsorbed in the proximal tubule by a Na+ -dependent mechanism. Two transporters (SGLT1 and SGLT2), located at the luminal cell membrane, bring glucose into the cell by using the energy released by the simultaneous downhill movement of Na+ . This is the same mechanism present in the small intestine. The two transporters differ in some aspects: SGLT2 is a low-affinity/high-capacity transporter with stoichiometry 1:1 located in the early proximal tubule. SGLT1 is a high-affinity/low capacity transporter, with a stoichiometry of 2Na+ : 1glucose, located in the late proximal tubule. Glucose exits the cell at the basolateral membrane by facilitated diffusion through two Na+ -independent transporters (GLUT2 in the early segment; GLUT1 in the late segment). With normal plasma glucose levels (100 mg/dL) all the filtered glucose [FG = PG·GFR ] is reabsorbed by the tubules. However, if plasma glucose increases to high values, as in diabetes mellitus, the filtered load exceeds the capacity of the tubules to reabsorb glucose. In other words, the transporters saturate, and the unreabsorbed glucose appears in the urine. This is shown in Fig. 5 (right). The maximum reabsorption rate (380 mg/min) is called the transport maximum (Tm). Many other substances are reabsorbed by a similar mechanism, and they are said to exhibit a Tm-limited mechanism of transport.

Bicarbonate reabsorption

H+ that is transported into the tubular lumen forms with HCO3 which is then reabsorbed back into blood. Filtered HCO3 in the lumen combines with H+ and makes CO2 + H2O so tubular lumen HCo3 disapears whereas the HCO3 formed in the tubular cell passes into the blood to promote HCO3 reabsorption without changing the luminal pH. Even with high plasma HCO3 at 28mM, most of it is reabsorbed and is labeled the bicarbonate threshold. The bicarbonate threshold varies during: -rate of HCO3 reabsoprtion due to level of pCO2 -Altered K and Na -Aldosterone production

Short and long term compensatory mechanisms are activated and responsible for restoring body water, plasma volume, and hematocrit

Hemorrhage: external, internal, slow or rapid. Hemorrhage has rapid short-term reflexes to shift internal fluid volume and long term neuroendocrine mechanisms to restore blood volume. -Graded hemorrhage on cardiac output and arterial pressure. Early in the bleeding, CO will fall but AP maintains. Withdrawal of 20% of blood leads to a fall in both AP and Co. There is a palteu after 30% due to central ischemic response due to sympathetic discharge triggered due to cv control center flow falling to cause ischemia. A reduction in blood volume leads to: reduction of venous pressure, reduced venous return, and reduced arterial pressures. Mean AP can be maintained between a 10-20% loss of BV but progressive loss in pulse pressure. Support of MAP is due to vasoconstriction and increased TPR , but CO decreases even with minute blood loss.

Explain the alkalotic state and bicarbonate secretion

In basic state, kidneys can excrete enough HCO3 to reach a urine pH of 8. HCO3 excretion to not accomplished by failing to reabsorb it, it must be secreted (this is based on studies of collectng tubules). 3 major cells in the distal nephron: Pricnipal/Granular Cell: Na reabsorption (no direct role in acid-base) Intercalated/Carbonic Anhydrase rich CA cells (reg acid-base) a- cells: secrete H into lumen (lumenal proton pumps, basolateral Cl-HCO3 exchangers) **b cells: secrete HCO3 (lumenal Cl-HCO4 exchangers, basolateral proton pumps) Both are regulated by transporter and enzyme expressiob. a-cells upregulated during acidosis (more H into lumen) b-cells upregulated during alkalosis (more HCO3 into lumen)

The mechanism for movement of water varies depending of the epithelium.

In many cases it is driven by osmotic pressure differences between the lumen and the interstitial space. Thus, for example, in certain renal tubules (descending limb of Henle's loop, collecting ducts) the higher osmolarity of the interstitium with respect to the luminal fluid drives the reabsorption of water. In other cases (proximal renal tubule, small intestine) a large absorption of fluid takes place in the absence of a significant difference in hydrostatic or osmotic pressure between the lumen and the blood side. A widely accepted explanation for the fluid absorption in these epithelia is the standing osmotic gradient hypothesis, a model for the coupling of water to the absorption of Na+. It is based on the observation that the absorption of fluid is proportional to the rate of solute absorption (Fig. 4, left) and that the fluid absorbed by the epithelia is normally isosmotic. Since most of the Na-K ATPases are localized at the lateral membranes, near the apical border, the continuing transport of Na+ into the narrow intercellular spaces, followed by Cl- results in an increased osmolality of the fluid in the intercellular spaces, near the apical end. Because of the large water permeability of the cell membranes and cellular junctions in these epithelia, small increases in osmolality (about 3-4 mOsm) is sufficient to produce large osmotic water flows from the lumen into the intercellular spaces. Fluid does not move in the opposite direction (from the capillaries and interstitial fluid into the intercellular spaces) because the reflection coefficient of the leaky membranes (basement membrane and endothelium) separating these compartments is so low that the effective osmotic pressure is near zero. The accumulation of fluid in the intercellular spaces raises its hydrostatic pressure and this pressure then drives the fluid (water and solutes) into the contiguous interstitium and capillaries, since the basement membrane and endothelium have a higher hydraulic permeability than the junctions and cell membranes (Fig.4).

Collecting Tubules and Collecting Ducts

In order for the filtrate to become fully modified into the urine that will exit the kidney, it must pass from the distal convoluted tubule through a short collecting tubule to a collecting duct. The collecting tubules can be distinguished from the distal convoluted tubules by the presence of interspersed dark cells (called intercalated cells) among the principal cells of the collecting tubule. These cells are dark because of abundant mitochondria throughout the cytoplasm; they play an important role in modulating acid-base balance in the body. At the base of the medulla the collecting ducts converge to form the ducts of Bellini that open into minor calyces.

Ureters path The ureters are muscular tubes that originate at each renal pelvis and descend through the abdomen into the pelvis to terminate at the urinary bladder. The anatomical location of the ureter is essential to understand, as it is difficult to visualize on imaging or during surgeries. The retroperitoneal descent of each ureter is characterized by the following anatomical relationships:

In the abdomen, the ureter passes anterior to the transverse processes of lumbar vertebrae and psoas muscle • The ureter is crossed anteriorly by the gonadal vessels • Crosses the pelvic brim approximately at the bifurcation of the common iliac artery • Descends along the lateral pelvic wall until it moves medially to the bladder. • In females the ureter passes immediately inferior to the uterine vessels as it passes lateral to the uterus • In males the ureter passes posterior and inferior to ductus deferens and anterior to seminal vesicle as it enters the bladder • The ureter obliquely enters the base of the bladder via a slit-like orifice that acts as a valve to prevent reflux of urine. In addition there are circular fibers in the intramural (intramural = within the wall of the bladder) part of the ureter that act as a sphincter.

Urea recycling

It has long been known that a low protein diet depresses renal concentrating ability markedly and that infusion of urea remedies this deficit within minutes. Although urea is a major solute of voided urine, a large fraction of the urea in the terminal inner medullary collecting duct (IMCD) undergoes medullary recycling. It is absorbed from the IMCD into the interstitium, from which it moves into the lumen of the thin ascending loop of Henle, and then is swept through the less urea permeable thick ascending limb, distal tubule and collecting ducts, until again it reaches the terminal IMCD. Urea transport across cell membranes entails diverse urea transporters, facilitating diffusion via protein "channels", and/or by secondary active transport in association with sodium. With a high rate of water reabsorption from the distal tubule and collecting ducts during antidiuresis a very high concentration of urea is presented to the terminal IMCD. ADH enhances IMCD urea permeability, leading to rapid diffusion of urea into the medullary interstitium. In antidiuresis (high plasma ADH), about 50% of the interstitial osmolality is due to urea, the remaining to NaCl. In diuresis, with low plasma ADH and low urea reabsorption from the IMCD, a significant fraction of the filtered urea is excreted, and its contribution to the interstitial osmolality is less important. (Note the different interstitial osmolalities shown in Figs 5 and 6). Despite intensive studies on the regulation of urea and other osmotically significant solutes, however, quantitative features of the terminal IMCD concentrating mechanism remain unclear. Finally, although it might be expected that renal medullary cells must shrink in the hyperosmotic renal medulla, shrinkage is opposed by the synthesis of intracellular osmotically active solutes.

Regulation of Potassium Secretion

K excretion changes by secretion in the distal nephron. Plasma K and aldosterone are regulators. Flow rate of tubular fluid and acid-base status changes K secretion in distal nephron. Hyperkalmeia causes K secretion in distal nephron via increasing basolateral Na-K ATPase so intracellular K rises enough to cause efflux to lumenal end. K permeability of lumenal membrane rises with hyperkalemia. Hyperkalemia effects are independent from aldosterone since they can occur in animals w/out aldosterone. Aldosterone will rise after K load and also cause rapid K secretion. Hypokalemia does the opposite. Aldosterone secretion is stimulated by high plasma AII and K. Aldosterone increases the synthesis of transport proteins (Na-K ATPase, Na channels, K channels) in the principal cell of the distal nephron to raise Na reabsorption and K secretion. High Na entry depolarizes the lumenal membrane to favor K secretion from cell to lumen. Aldosterone secretion is inhibited by hypokalemia and ANP.

What is the normal range of dietary K intake and major routes of K loss from the body? What is the role of extracellular K for maintaining nerve and muscle function?

K is the most concentrated cation in body. Total body content is 50mEg/kg BW (3500 mEq avg). 98% of K is inside the cell (150mEq/L). Most of ICM K is in muscle (2600 mEq) and 300 mEq in RBC, liver, bone. Only 65mEq 2% is in Shifts from eCF and ICF can cause large unpredictable changes in plasma K (3.5-5mEq/L)

The kidneys both produce hormones and respond to hormones

Kidney involved in ADH: Salt/Water, Aldosterone (adrenal cortex): reabsorb Na/secrete K, ANP: secrete Na, PTH: secrete PO4/absorb Ca, reabsorb Ca, Vit D Calcitriol active: increase Ca/PO4 gut absorp, regulated by PTH/prolactin, blood pressure, Erythropoietin: control erythroid hematopoiesis/made by cortex + outer medullar interstitial cells, Renin: release from JG due to low perfusion pressure or nervous sys, enzyme cleaves AI to AII in capilarries to increase blood pressure, Prostaglandin: medullary intersittial cell act as vasodepressor.

The kidneys filter blood to produce urine, which is then conveyed via the ureters to the bladder where it is stored until micturition (urination). The urine is then transported out of the body via the urethra. The entire urinary system is retroperitoneal with the exception of the urethra, which is part of the body wall anatomy.

Kidneys Anatomical location The superior parts of the kidneys are located deep to the 11th and 12th ribs and the inferior pole of the right kidney is approximately 1-2 cm above the iliac crest. The right kidney is positioned slightly more inferior than is the left kidney, likely due to the presence of the liver on the right. The kidneys are well protected from trauma due to this favorable anatomic position. The kidneys are tethered in place by layers of fascia and fat. Immediately external to the renal capsule is perinephric fat, which extends into the renal sinus. Importantly, the kidney plus this fat are held in place by the renal fascia, which blends medially with the vascular sheaths of the renal vessels and extends inferiorly as the periureteric fascia. Renal fascia and the accompanying paranephric and perinephric fat play an important role in holding the kidneys in place high in the abdomen (some movement does occur with respiration and positional changes) and can also create an anatomic barrier to blood, urine or pus resulting from a urinary infection.

E. Aging changes in the urinary system

Most histopathology involving the kidney occurs in disease states; fortunately, it is quite common for kidneys to work well long into senescence. However, some changes are worth noting. With age, the total kidney volume decreases, and the number of functional nephrons decreases; this puts relatively more burden on remaining nephrons, which under normal circumstances have a large reserve capacity. Other systemic diseases, or pharmacological interventions, can however put similar demands on the kidney's reserve capacity, such that kidney failure might be secondary to another cause of mortality. The glomerular filtration rate and tubular function both slow significantly. Changes in the circulatory system, especially the decreased compliance of vessel walls affect the kidney's ability to adjust its production. Finally, the bladder loses its mechanical function to some degree, probably as a result of changes to each of its connective tissue (fibrosis), smooth muscle, and nervous components. It is common in men for the urethra to be constricted by an enlargement of the prostate, causing urine retention in the bladder and resultant urinary tract infections

Importance of NH4 in acid-base status

Most of H+ excretion is through NH4 (2/3). Cells in the PCT form NH4 and HCO3 via deamination of glutamine/aa. HCO3 is transported to the basolateral membrane into peritubular capillaries while NH4 enters the lumen of PCT using the Na/NH4 exchange. The thick ascending limb in the loop of Henle reabsorbed some NH4 replacing K on the Na/K/2CL cotransporter. NH4 is in equilibrium with a small concentration of NH3/H+ to accumulate in the medullary interstitium. The collecting duct secretes NH3, NH4, and H using many methods. Rh-glycoproteins (RhCg/RhBg) helps move NH3 across the basolateral/lum membranes into the tubular fluid so that Nh3 can react with H+ to form NH4 that is "ammonium trapping" in the urine. This causes the maintenence of a medullary intersititial/tubular lumen NH3 gradient to rive ammonia into the collecting duct and prevent it from reaching the cortex where it could go back into the blood. NH4 secretion accounds for 2/4 of acid excretion. Unlike phosphate system, it does not contribute to the titratable acids because the pK is high beyond the pH of the tubular lumen. Titrating acidified urine many only convert very little NH4 to NH3 because of this (98% stay protonated at normal pH of NH4+)

Role of aldosterone in extrarenal K regulation is not clear. Small elevation of pK cause increase in adrenal aldosterone to raise colonic/renal K excretion though. Addisons or aldosterone deficiency patients can get hyperkalemia. Adrenal tumor of aldosterone secretion can cause hypokalemia.

Nonhormonal factors (acid-base, exercise, pOSM) can alter pK but do not contribute regulation. In acidosis, increased H+ moves into the cell to be buffered, which causes K+ to move out and reaises pK. A drop in .1pH leads to an increase in .6mEq/L in pK (no real H/K exchanger). Mechanisms for this 1) release of protein-bound K with drop in pH and inhibition of Na/K ATPase. In alkalosis H+ moves out of the cell to buffer the alkali ECF so K moves into the cell and causes hypokalemia. The decreae in pK is alkalosis is less drastic than in acidosis. Metabolic acidosis or alkalosis is assoc. with mineal acids/bases (HCl, NaHCO3) to cause shifts in K+ whereas organic acids/bases (lactic/ketone) do not cause shifts in K+. In respiratory acid-base disturbances, less shift in K+ occurs. Exercise changes pK+ because K will be released from muscle during exercise to increase blood flow The change in pK is reversed after rest. If a person takes B-adrenergic recepter blockers, pK can rise enough to be lethal during exercise. Increase in osmolality of ECF such as in hyperglycemia lead to outward movement of K causing the cell to shrink, which increases the cellular K concentration and increases this chemical K efflux gradient. In hyposmolality, K will seep into the cell and expand it to increase gradient into cell.

Roles of Potassium

Potassium is cofactor for protein/glycogen synth, regulate muscle blood flow, and set cell membrane potential (because of high permeability to K+ MP infuenced by K gradient so changes in K can cause issues in excitation of nerve cells and muscle.

Problem 3 Aminoglycosides are very effective antibiotics but they can be toxic, particularly to proximal tubular cells which take up and store the antibiotics after they are filtered at the glumerulus. The induced acute tubular necrosis (ATN) can be severe enough to impair sodium reabsorption by some nephrons. A. Patients with aminoglycoside-induced ATN develop a slow rise in serum creatinine after several days of antibiotic treatment. How might the reduced NaCl reabsorption in the proximal tubule contribute to the decline in GFR?

Reduced salt reabsorption in the proximal tubule will increase its delivery to the macula densa. This will activate the tubuloglomerular feedback system: release of adenosine by macula densa tubular cells that constricts afferent arterioles and lowers the nephron filtration rate towards normal. If this compensatory decline in GFR did not occur, the reabsorptive capacity of the distal and collecting tubules might be overwhelmed, leading to potentially fatal sodium and water losses.

Introduction After considering the basic renal processes in the previous lecture, we shall now examine the regulation of renal blood flow and GFR, and how the kidneys maintain the balance between the rates of filtration and reabsorption (Glomerulo -Tubular balance).

Renal Hemodynamics The rate of blood flow into the glomerulus is an important determinant of the rate of filtration. Recall that the oncotic pressure inside the glomerular capillaries rises as a consequence of the large filtration that takes place in these very permeable capillaries. This oncotic pressure opposes the filtration of plasma. When the delivery of arterial blood (with an oncotic pressure that is lower than that inside the glomerular capillary) to the kidney increases it reduces the rate of rise of the oncotic pressure inside the glomerular capillaries. As a consequence, GFR rises. Compared with the normal rate of filtration, GFR increases only moderately with increasing RBF, but it decreases greatly with declining RBF. As mentioned in the previous lecture, the renal vasculature is unique in that it has two sets of arterioles in series, as well as two capillary beds. As you would predict, significant pressure drops occur in the arterioles (see Fig. 1). The degree of constriction of both the afferent and efferent arterioles influences the pressure and flow through the glomeruli. Compared with systemic capillaries the pressure in the glomerular capillaries is relatively high, whereas the pressure in the peritubular capillaries is relatively low

The macula dense is one part of the three-component juxtaglomerular apparatus

Renin-containing juxtaglomerular cells (JGC; granular cells) located in the media of the afferent glomerular arteriole represent the second component. These JCG are modified smooth muscle cells that synthesize, store, and release renin granules. Renin granules can be seen in slide 460. Although the vast majority of juxtaglomerular cells are located in the afferent arteriole, occasionally they are present in the wall of the efferent arteriole. Extraglomerular mesangial cells (also known as lacis cells) are the third component of the juxtaglomerular apparatus. These small cells occupy space bordered by the afferent and efferent arterioles, the macula densa, and the vascular pole of the renal corpuscle. You may not see all three components in the same plane of section. The distal tubule that extends beyond the macula densa drains into a small collecting tubule that connects to the collecting duct.

Blood supply

The blood that is filtered by the kidneys is the same blood that supplies the parenchyma of the organ itself. This blood arrives to the organ via the renal artery. Although it is not uncommon to observe early branching or doubling of this vessel, the textbook description is that the renal artery divides close to the renal hilum into five segmental arteries (superior/apical, anterosuperior, anteroinferior, inferior, and posterior). These arteries are functionally end arteries, therefore dividing the kidney into independent surgically resectable renal segments. The veins within the kidney anastomose freely and drain to the renal veins

In our tissue sections, you should distinguish the following tubules (only): proximal convoluted tubule thick descending tubule thin tubule thick ascending tubule distal tubule collecting duct

The connective tissue compartment between the tubules and capillaries of the medulla is called the renal medullary interstitium and it is absolutely key to the ability of the nephron to produce urine of the appropriate osmolarity.

The production of hypertonic urine

The interstitium is most of the tissue in the medulla than the cortex, % increases moving toward papilla. Hyperosmolarity of the interstitium controls the high osmolarity of urine. In the cortex, filtrate in the collecting ducts is hypotonic. W/ ADH (anti-diuretic hormone), the cells of the collecting duct are permeable to water. So, as the filtrate moves down the collecting duct, through the high osmolarity of the medullary interstitium, water is drawn out of the urine, creating urine with high osmolarity. The water that is drawn out is returned to the body via the vasa recta. The Ureter Move urine from the kidneys to the urinary bladder, entering the bladder from back of the urinary bladder. Surrounded by valves that prevent backflow. The lumen of the relaxed ureter has a stellate pattern and lined by 4-5 layer transitional epithelium. Lamina propria submocosa is loose collagenous/elastic layer. There is a thin muscularis mucosae between the lamina propria and the submucosa. An outermost muscular tunic, the muscularis, consists of two or three loosely arranged smooth muscle bundles separated by connective tissue. The inner layer is longitudinally arranged, the next layer circular, and the third, outermost coat, is again longitudinally arranged. Because the ureter is, like the kidney, a retroperitoneal organ, it has an adventitia and no serosa.

A. Microscopic Organization

The kidney is a compound tubular gland, the basic functional unit of which is the nephron. Examine Figure 3 carefully and observe the details of parts of the nephron and, importantly, the location of these parts in the kidney with respect to the cortex and medulla. You know the basic organization of the kidney and its vasculature—now you will place the nephron into that context. Understanding this diagram of the nephron is key to your understanding of the microscopic sections of the nephron.

Introduction to Renal Function

The kidneys perform several vital functions beyond their well-known role of removing waste. The most important role of the kidney, and the one that we will discuss in detail, is the regulation of the volume and composition of the extracellular fluid (the internal environment in the words of Claude Bernard). This regulatory role of the kidney has been emphasized by several physiologists, from Claude Bernard to Walter Cannon, who coined the term homeostasis to describe the set of mechanisms that maintain the constancy of the internal environment. The main functions of the kidneys can be summarized as: 1. To regulate the volume and the composition (concentration of inorganic ions, osmolality, acidity) of the extracellular fluid in order to maintain the conditions compatible with life. 2. To excrete metabolic waste products (urea, uric acid, creatinine, etc), end products of hemoglobin breakdown and foreign chemicals (drugs, food additives, pesticides). 3. To produce hormones and circulating factors: erythropoeitin, renin, 1,25- dihydroxyvitamin D3.

General Design

The larger arteries and veins travel together. The arteries enter at the hilum and divide into segmental branches whose further branches travel between the medullary pyramids to the cortex, then pass parallel to the kidney surface at the interface between the cortex and medulla; smaller radial branches supply the cortex; small arteriolar branches supply the capillary systems and veins drain the capillary beds. Veins return via the route out to the hilum following the path of the arteries. All blood within the second capillary bed must first past through a glomerulus. the arteries have a distinct smooth muscle media and internal elastic lamina and the veins are thin-walled; in fact, they are unusually thin-walled in the cortex.

The urinary bladder collects urine & stores. Holds 300-350 ml of urine. W/ urine, the rugae flatten and the wall becomes thinner.stretch allows store larger without rise in internal pressure.

The lining epithelium high degree of elasticity and impermeable to salts and water. Lumen has thick layer of protective glycoprotein for added protection. Relaxed epithelium: apical PM olds inward: distends: the apical membrane unfolds and flattens out. In the relaxed urinary bladder the mucosa is thick, irregular folds; the transitional epithelium is 6-8 cell layers thick w/ thin muscularis mucosae the muscularis is well-developed with many interlacing bundles of smooth muscle fibers but not sharply demarcated into layers. the peritoneal serosa has an elastic lamina within the submesothelial connective tissue. The serosal surface is limited to the superior surface of the urinary bladder (and part of the posterior surface in men); the rest of the surface, where it attaches to the pelvic wall, is an adventitia the relaxed urinary bladder has a thick wall and both the luminal mucosa and the serosal surface are highly folded. Distention of this wall can occur without significantly increasing the internal pressure.

Fig. 6 Antidiuresis. The important changes from diuresis (Fig 4) are: (l) higher medullary and papillary interstitial osmolality; (2) presence of ADH; (3) osmotic equilibration between the fluid in the collecting duct and the adjacent interstitium.

The mechanism of action of antidiuretic hormone on cell water permeability has been intensively studied. In brief, binding of ADH to a basolateral receptor activates adenylate cyclase via a guanine nucleotide binding protein, with formation of cyclic AMP and phosphorylation of lumenal membrane proteins that leads to incorporation of aquaporins (AQP2) into collecting duct lumenal membranes (Fig.7). In addition to its enhancement of cell water permeability, ADH influences urine osmolality and volume by stimulating thick ascending limb salt transport, depressing vasa recta blood flow (see below) and enhancing inner medullary collecting duct urea transport. Stimulation of active NaCl reabsorption in the thick ascending limb raises osmolality in the interstitium, and facilitates water reabsorption from the adjacent collecting ducts. Depression of vasa recta blood flow reduces washout of interstitial solute and protects osmotic stratification. Enhanced urea permeability of the terminal medullary collecting duct contributes importantly to interstitial osmolality, which reaches some 1200 mOsm/kg during antidiuresis, and is associated with enhanced water reabsorption.

Fig 4: Left: Tubuloglomerular feedback. An increase in GFR (1) increases NaCl delivery to the loop of Henle (2), which is sensed by the macula densa and converted into a signal (3) that increases the resistance of the afferent arteriole (4), and decreases GFR and RBF. Right: Relation of single nephron GFR to distal nephron (macula densa) perfusion rate in dogs. As the perfusion rate increases (via the insertion of a micropipette in the late proximal tubule), there is a progressive decline in GFR to a minimum of about half the initial level. (Adapted from Navar, L. Am J Physiol 234:F357, 1978)

The mechanism of autoregulation not only prevents excessive changes in GFR by the continual fluctuations in renal artery pressure that occur with normal daily activities, but it also protects the fragile glomerulus from an excessive distending pressure, and it helps maintain the distal nephron flow at a constant rate. Excess filtration of Na+ and water, for instance, could overwhelm the limited reabsorptive capacity of the distal nephron and result in the loss of extracellular fluid.

The relative size of a nephron depends on the location of its renal corpuscle. More specifically, the relative length of the loops of Henle differs: for the nephron whose corpuscle is located in the outer cortex, the loop of Henle is relatively short and does not penetrate deeply into the medulla; for the nephron whose corpuscle is located nearer the medulla, the loop on Henle descends to the medullary papilla. This arrangement helps explain the pyramidal shape of the medullary pyramid: it would not be pyramidal if all nephrons extended to the apex.

The morphology of the tubular part of the nephron differs in three regions that are color-coded in the figure. It is all a simple epithelium. The epithelium of the proximal convoluted tubule and the descending thick limb of the loop of Henle is specialized for absorption and the cells have very prominent brush border and basal striations. The thin limb of the loop of Henle, is a squamous epithelium. The ascending thick limb of the loop of Henle and the distal tubule is a cuboidal epithelium with modest basal striations and no brush border. The collecting duct is a simple epithelium populated by two cell types.

1. Renal Corpuscle

The numerous glomeruli, each partially encircled by a clear area, are a prominent feature of the renal cortex. Renal corpuscles are located only in the cortex. The renal corpuscle consists of two parts: the vascular tuft of capillaries, the glomerulus Bowman's capsule, the surrounding epithelial sac The glomerulus arises from an afferent arteriole that enters the corpuscle at its vascular pole, and branches into the glomerulus. An efferent arteriole, formed of the reuniting capillary plexus, exits at the same vascular pole. Bowman's capsule is composed of two simple cell layers: parietal and visceral, both of which are simple squamous epithelia. The parietal layer forms a smooth, spheroid external capsule, while the visceral layer is composed of podocytes. The space between the two layers is known as the urinary space or Bowman's space.

Bowman's capsule is composed of two simple cellular layers: parietal and visceral, both of which are squamous epithelia.

The parietal layer forms a smooth, spheroid external capsule, while the visceral layer is composed of podocytes. Some podocyte nuclei can be distinguished at the periphery of the glomerulus, based on the fact that their cell bodies and nuclei project directly into the urinary space. Look carefully at glomeruli in slide 451 and find where the plane of section reveals the unusual shape of the podocyte cell body (as illustrated in figures 5 and 6 in this syllabus section). Seek cells within the capillary tuft whose nuclei neither surround capillary lumens, nor protrude into Bowman's space. These nuclei quite likely belong to mesangial cells. The mesangial cell nuclei tend to be slightly larger than, and they have more euchromatin than, the other nuclei in this structure (i.e., the endothelial cell nuclei and podocyte nuclei). The mesangial cells are principally located in the core of the glomerulus. Be aware that there will be some cells in the renal corpuscle that you are able to identify with certainty, but many that you will not be able to identify. Aim to identify only those that fit the morphological criteria you seek.

The pedicels of two separate podocytes interdigitate to form adjustable filtration slits. A nonmembranous slit diaphragm spans these slits. The slit diaphragm consists of several extracellular and transmembrane proteins, including nephrin, that are critically involved in preventing the passage of albumin and other proteins into the urinary space. At the vascular pole, the visceral layer of Bowman's capsule reflects laterally and becomes the simple squamous epithelial cells of the parietal layer of Bowman's capsule. Thus the ultrafiltrate is contained within a space that exists between the two layers of the capsule.

The pedicels of two separate podocytes interdigitate to form adjustable filtration slits. A nonmembranous slit diaphragm spans these slits. The slit diaphragm consists of several extracellular and transmembrane proteins, including nephrin, that are critically involved in preventing the passage of albumin and other proteins into the urinary space. At the vascular pole, the visceral layer of Bowman's capsule reflects laterally and becomes the simple squamous epithelial cells of the parietal layer of Bowman's capsule. Thus the ultrafiltrate is contained within a space that exists between the two layers of the capsule.

Glomerular Filtration

The plasma filtered at the glomerulus has to cross a filtration barrier consisting of three layers: 1) the single-celled capillary endothelium with fenestrations (70 nm holes) which is the principal barrier to blood cells, 2) the basement membrane, a 300-350 nm-thick extracellular structure made up of negatively charged glycoproteins (collagen, proteoglycans, laminin, etc.) that restricts large, negatively charged solutes from crossing, and 3) the single-celled epithelial visceral layer of Bowman's capsule. These epithelial cells are called podocytes, and they possess a large number of extensions (pedicels), or foot processes, embedded in the basement membrane surrounding the glomerular capillaries. Between these foot processes there are gaps (filtration slits) covered with a thin diaphragm (made up of negatively charged proteins: nephrin, podocin, etc. on lipid rafts of podocytes) containing pores ca. 4 nm in size. Glycoproteins with negative charges cover the podocytes, filtration slits and diaphragms. Because of hindrance and electrostatic interactions, molecules of size >3.6 nm are rejected by the barrier; molecules of size < 1.0 nm are filtered, and among those with intermediate sizes, cationic molecules are filtered better than anionic species. Filtration of large molecules, like proteins, is restricted. A small amount of albumin may filter through, but it is partially degraded and reabsorbed, and normally none appears in the urine.

a. Proximal convoluted tubule and thick descending limb

The proximal convoluted tubule is the longest tubule in the cortex, therefore it is the most frequent profile found in cortical sections. Cells are cuboidal or pyramidal, acidophilic and have long tangled microvilli. Reabsorption of protein, ions (such as sodium and bicarbonate), and glucose occurs at the apex of the cells and the cytoplasm contains many lysosomes. Vertical stacks of mitochondria are found at the base of the cells, which is typical of cells involved in active transport. The basal plasma membrane has many basal infoldings, which serve to increase the surface area of this part of the cell. Although not particularly evident in the light microscope, the lateral membranes are also highly folded, making complex interdigitations with neighboring cells. The proximal straight tubule has similar cells that are somewhat shorter and contain many peroxisomes. The thick descending tubule has somewhat longer microvilli, but looks otherwise identical to the proximal tubules, and is distinguished by its location in the medulla

Creation of osmolality profiles

The strong dependence of renal function on kidney structure was long ago inferred from the close correlation in many species between the length of the thin loops of Henle and maximum concentration of the urine. A basis for this correlation was suggested by Kuhn and coworkers, who, recognizing the structural similarity of the nephron and its vascular supply to common engineering systems, theorized as to how countercurrent mechanisms could produce and maintain osmotic gradients necessary for concentration of the urine. In the face of considerable skepticism Kuhn et al. later presented experimental evidence for "osmotic stratification", with progressive increase of interstitial osmolality from the cortex to the papilla. Several important features of the countercurrent mechanism are now well established. Despite as yet incomplete understanding, they help to explain how lumenal and interstitial osmolality profiles are created, maintained and utilized in the regulation of urine osmolality. Creation of osmotic gradients occurs largely in the ascending limb of the loop of Henle, where salt is actively reabsorbed (transported from the lumen into the interstitial fluid) without transport of water. Osmotic equilibration between 1 The plasma osmolality can be estimated from two times the plasma sodium concentration, Posm ≈ 2 PNa , because sodium salts comprise most of the extracellular osmoles. Of course, with hyperglycemia this relation underestimates Posm. the interstitium and the adjacent descending limb of Henle (highly permeable to water but not to solute) results in an osmolality difference, with the osmolality of the interstitial fluid and that in the descending limb exceeding that of the fluid in the ascending limb by some 200 mOsm/kg at every level (Fig 3). The creation of a 200 mOsm/kg gradient between the lumen of the ascending limb and that of the adjacent descending limb at each level along its length is termed the "single effect" and leads to "countercurrent multiplication". The multiplication depends on countercurrent flow whereby tubular fluid moving down the descending limb passes fluid moving upward in the adjacent, parallel ascending limb. An oversimplified, "classical" model that has been used to provide a conceptual understanding of this mechanism is presented in Fig. 4. It illustrates the progressive increase in osmolality as tubular fluid flows downward toward the hairpin loop and progressive decrease in osmolality as it flows upward, after the turn, through the ascending limb.

Tubular Secretion

The two most important substances secreted by the tubules are H+ and K+. Secretion of H+ will be explained in the lecture on acid-base balance, and another lecture will discuss the regulation of body K+. Many other substances, mostly organic acids or bases, either end products of metabolism or exogenous (drugs), are avidly secreted by the tubules (see table below). Most of the secreted anions share common carriers (Organic Anion Transporters, OATs) located at the basolateral and lumenal membranes of the tubular cells. Competition among substrates can lead to a reduced rate of excretion and prolong the biological half-life of a drug (e.g., administration of hippurates together with penicillin extends the half life of the latter and its therapeutic effects). Organic cations are also secreted by broad specificity facilitated mechanisms (Organic Cation Transporters, OCTs). At the luminal membrane the cation is secreted in exchange for H+ . Most of the secretion of anions and cations takes place at the late proximal tubule

The renal tubule, continuous with Bowman's capsule, is a very narrow cylinder made up of a single layer of epithelial cells resting on a basement membrane.

The ultrafiltrate enters the tubule and collecting ducts where it is processed before flowing as urine into calyces, the pelvis of the kidneys, ureters and bladder. The two processes taking place at the tubules are reabsorption and secretion. Plasma solutes filtered at the glomerulus are handled differently by the tubules. The excretion rate of a given substance equals its filtration rate plus the rate of secretion minus the reabsorption rate (Fig 1).

D. Urethra

The urethra forms the exit of the urinary bladder, traversing the body wall in females, and traversing the prostate gland (prostatic urethra), body wall (membranous urethra), and penis (penile urethra) in males. Within the urethra, several different epithelial morphologies are possible. Proximal to the bladder, a transitional epithelium continues. Distally, the epithelium transforms into stratified squamous epithelium as it joins the integument. In between, the epithelium may be described as pseudostratified, or as stratified columnar in areas where superficial mucus cells are abundant. These cells, along with minor mucus glands, and in males the ejaculatory ducts, prostate and bulbourethral glands (discussed in the syllabus section on Male Reproductive System), empty into the urethra. The muscularis layer near the junction of the urethra with the bladder forms the internal urinary sphincter. Similar to the anatomy of the anal sphincter, the internal urinary sphincter is a continuation of the muscularis layer of the bladder, and the separate external urinary sphincter is composed of skeletal muscle, located much further from the lumen. These sphincters are clinically important since disruption of their function causes enuresis, a condition for which people often are motivated to seek medical help.

The proximal tubule arises from the urinary pole of the corpuscle

The urinary pole is typically located on the opposite side of the corpuscle from the vascular pole. Scan the cortex to find a plane of section that shows the urinary pole. The proximal convoluted tubule (and the descending straight [thick] portion of Henle's loop) consists of relatively short and wide columnar cells, with large, spherical nuclei, an eosinophilic cytoplasm, and obscure lateral borders. The most striking feature relates to this tubule's function as the site of maximal absorption of material from the filtrate: the cells typically have many vesicles and vacuoles in their apical cytoplasm and a prominent brush border of long tangled microvilli. Careful examination should also reveal small perpendicular striations in the basal cytoplasm of these cells; these striations are the basal infoldings that serve to increase the area of the basal surface of this cell. Thus there are surface specializations on both the apical and basal surfaces of these cells that are designed to facilitate their function of reclaiming material from the filtrate so that these substances can be returned to the blood. Proximal convoluted tubules are found only in the cortex. However, descending thick tubules in the medulla share a nearly identical morphology. In contrast to the proximal tubules, the distal tubule (in the cortex some of this part of the nephron is straight and some is convoluted, but there is no additional adjective necessary) and the ascending thick limb of Henle's loop are lined by cells that lack a brush border and apical cytoplasmic vacuoles. The cells range from being quite low cuboidal to cuboidal with the nucleus being somewhat flattened to round, respectively. These tubules have a slightly wider lumen than the proximal ones. Their profiles are considerably less frequent in the cortex than those of the proximal tubules; this numerical frequency reflects differences in the relative length of the nephrons' proximal and distal tubules

C. Ureters and Urinary Bladder

The urinary bladder and ureters are functionally important in preventing resorption of compounds from the urine, and also in promoting a one-way flow through the urinary system. The urinary bladder epithelium consists of transitional epithelium or urothelium, which usually appears as three distinct layers of cells. Immediately adjacent to the basement membrane is the basal layer. Some of these cells are stem cells that serve to periodically renew the epithelium. The superficial layer is termed the umbrella cell layer due to the characteristic shape of its cells when the bladder is relaxed. The umbrella cells form a first line of defense to any bacteria that manage to enter the lumen of the bladder by traveling retrograde from the urethra. The surface membrane of the umbrella cells is unique in that the inner and outer membrane leaflets are asymmetrical, and thus thicker than a typical cell membrane. Asymmetrical membranes are also seen lining intracellular vesicles in umbrella cells, suggesting that their function may be add to or subtract from the apical surface area as the bladder changes in volume. Often, umbrella cells are multinucleated, suggesting that membrane turnover is so rapid as to permit adjacent cells to occasionally join together. Umbrella cells form tight junctions with each other, impeding the passage of luminal contents past the epithelium. In between the basal and superficial layer is found the intermediate layer. Because these cells are connected to their neighbors predominantly by desmosome attachments, they can appear radially elongated in the relaxed bladder, yet flattened in the stretched bladder. Despite the layered appearance of the urothelium, intermediate layer cells also manage to contact the basal lamina via fine cytoplasmic projections. Thus, an intermediate cell may appear basally located in a stretched bladder. A similar, but thinner urothelium surrounds the lumens of the ureters. Because the ureters traverse the muscularis layer of the bladder at an angle, pressure in the bladder lumen is not normally transmitted to the ureter or kidney. This is a critical function of the ureter: if the nephron and/or glomerular lumens are chronically exposed to pressure, life-threatening pathologies can easily result. The muscularis layer of the bladder may be termed the muscularis propria. In gross anatomy, it is known as the detrusor muscle. Though commonly ignored in histology textbooks, a superficial muscularis mucosae layer also exists, which is incomplete and often scant. This layer is worth mentioning since it may become pathologically hyperplastic (such as after local trauma), and can complicate the interpretation of biopsy specimens. Also, pathologists can use the layer in order to stage the progression of epithelial cancers

Perineal Embryology

There is no natural outflow pathway for the intermediate mesoderm. Rather, the caudal end of the mesonephric duct must form a connection with the outside world. It does so by meeting the endoderm-derived hindgut in an embryonic orifice known as the cloaca. The cloaca (< "sewer") represents the uniting of future bladder with future ureter, and with a third embryonic structure, the allantois, which is a connection to extraembryonic structures through the umbilicus. (In birds and reptiles, the allantois serves as an early reservoir for nitrogenous wastes, a function that is eventually taken over by the kidney.) For the practice of medicine, the principle importance of the allantois is to realize its anatomical position. As the cavity regresses, it leaves behind a normally fibrous structure termed the urachus, extending between the bladder and the umbilicus. If the allantois fails to regress, the urachus may continue to include a patent channel connecting bladder and umbilicus. The cloaca (in humans and other placental mammals) resolves into separate urogenital and anorectal openings through the formation of the urorectal septum. The urogenital sinus which lies anterior thus receives the urethral, derived from anterior cloaca (endoderm) and reproductive orifices (upper vaginal derived from paramesonephric duct in females, or ejaculatory duct derived from mesonephric duct in males) and connects them to the labioscrotal swelling, which is an ectodermal feature. The perineum is the integumental region lying between anal and urogenital openings.

On the other hand, in the presence of high blood concentrations of ADH, the late distal tubules and collecting ducts become highly permeable to water and permit equilibration of the lumenal fluid with the increasingly hyperosmotic interstitium in the course of descent through the terminal nephron (Fig. 6)

Therefore, the level of plasma ADH determines the volume and concentration of the urine. When plasma ADH is low or absent, a large volume (about 10% of the GFR or 18 L) of diluted (50-60 mOsm) urine is excreted. In the presence of maximal plasma levels of ADH, a small volume (near 0.4 L) of concentrated (1,200 mOsm) urine is excreted. This volume of urine is known as the obligatory water loss (required for the excretion of waste products). It contributes to dehydration when a person is deprived of water intake. Normally, plasma ADH levels, and therefore urine volume and osmolality, are between these two extremes. We will discuss later in this lecture the mechanisms that regulate ADH secretion.

The arcuate arteries give rise to branches that are again (like the interlobars) radial in their orientation

These interlobular arteries travel between the medullary rays into the cortex. The interlobular arteries and veins travel together. The interlobular arteries give rise to the afferent arteriole that eventually gives rise to the glomerulus (which we will examine in more detail in the Discussion Session). Inspect the outer cortex of slide 455. The glomeruli appear very dark because their capillaries are filled with red blood cells. These glomeruli represent the first of the two capillary beds that constitute the portal system in the kidney. An efferent arteriole leaves the glomerulus and connects the two capillary systems. You are not responsible for distinguishing afferent from efferent arterioles in these slides. There are two options for the second capillary bed: which one arises from the efferent arteriole depends on the location of the glomerulus. Search the areas around the various tubule profiles in the cortex and observe that there is an extensive pattern of capillaries (recognized by blood contained within them) surrounding the tubules. The efferent arteriole arising from cortical glomeruli gives rise to a peritubular capillary plexus (peri-, around). Examine the region of the medulla where tubules are longitudinally sectioned. Observe that there are blood-filled capillaries that run parallel to these various tubules. These capillaries are part of the other second capillary bed, knows as the vasa recta (L. straight vessels). The efferent arteriole of glomeruli located near the medulla (known as juxtamedullary glomeruli) gives rise to the capillaries of the vasa recta (descending arteriolae rectae and ascending venae rectae—between which you are not expected to differentiate) that extend into the medulla in parallel with the straight portions of the nephron (the loop of Henle). Most of the vasa recta capillaries drain into the arcuate veins, whereas the capillaries of the peritubular capillary plexus drain into the interlobular veins. Examine slide 458. This slide is especially useful for identifying the components of the portal system that exists between the arterial and venous sides of the circulation. In this annotated VM slide, an arcuate artery and vein are labeled based on their orientation and location between the cortex and medulla. An interlobular artery and interlobular veins are labeled, again based on their orientation and location. Note that the arteries have appropriately thick muscular walls, but the veins tend to have little more than the endothelium of their walls. Blood is not perfused from the capillary beds in this section, and so the capillaries here look like solid pink patches, sometimes encircled with recognizable endothelium. Look around the tubules in the cortex and medulla to get a sense of the extensive peritubular capillary plexus and vasa recta, respectively. Since the radially oriented tubules of the medulla are transversely sectioned in this tissue section, you can expect that the vessels of the vasa recta, which run parallel to the tubules in the medulla, are also sectioned transversely.

Renal Clearance

To estimate the rate at which a particular substance is removed by the kidneys from the blood we calculate its renal clearance. The concept of clearance can be better understood with the following example. Consider a patient who has a urine flow rate of 1.2 ml/min and urea concentrations of 17 mg/ml in the urine and 0.3 mg/ml in plasma. The amount of urea excreted by the kidneys every minute is simply: Eurea = Uurea ∙ V̇ = 17 ∙ 1.2= 20.4 mg/min What volume of plasma has to be supplied to the kidneys every minute to deliver this amount of urea? Since every ml of plasma contains 0.3 mg of urea, the volume of plasma required is: 20.4 (mg/min) / 0.3 (mg/ml) = 68 ml/ min Hence 68 ml/min represents the volume of plasma that would have been cleared of urea in a minute and it is called the clearance of urea.

Autoregulation of the GFR is mediated by more than the myogenic response, as both angiotensin II (when the renal perfusion pressure is reduced) and tubuloglomerular feedback, TGF (especially when renal perfusion pressure is increased, see later) play important roles. Recall than when the renal perfusion pressure is substantially reduced the renin-angiotensin system is activated and results in both local and systemic generation of AII. This hormone induces a preferential increase in efferent arteriolar resistance, and as a consequence it prevents the fall in PGC and GFR. Consequently, infusion of an AII antagonist or ACE inhibitor leads to a less effective maintenance of GFR (Fig. 3). Fig. 3: Role of AII in autoregulation. Effects of reducing renal artery pressure on GFR in control dogs and in dogs which received intrarenal infusion of an AII antagonist. Note that AII helps to maintain GFR when the perfusing pressure is substantially reduced. Adapted from Hall, Guyton et al. Am. J. Physiol. 233(5):F366, 1977

Tubuloglomerular feedback refers to the changes in GFR that can be induced by changes in the flow rate of the fluid perfusing the macula densa cells at the end of the ascending loop of Henle. An increase in tubular flow consequent to increased GFR is sensed by the macula densa cells that secrete an afferent arteriole vasoconstrictor resulting in reduction of filtration (Fig. 4). There is controversy and uncertainty on the mechanism of sensing by the macula densa and the nature of the vasoconstrictor. It is believed that the macula densa senses an increase in the concentration of NaCl in the fluid delivered to the distal nephron1 , probably as the rate of the luminal NKCC2 transporter that reabsorbs these ions is enhanced. This in turn alters either cell volume and/or intracellular Ca++ levels that stimulate the secretion of either adenosine or ATP. Adenosine causes vasoconstriction by interacting with an afferent arteriolar smooth muscle cell receptor (A1) that is different from the one (A2) that leads to vasodilation in other parts of the CVS. The sensitivity of the TGF mechanism, that is, how strongly it responds to an increase in the tubular flow rate at the macula densa, is modulated by the status of the ECF volume. Volume contraction (and the associated elevation in plasma AII levels) increases the sensitivity of the TGF, whereas volume expansion (and elevated NO and PGI2) decreases it. Although the mechanism of the modulation is not clear, its usefulness is. For instance, in a volume contraction it is crucial that any spontaneous increase in GFR be dealt with swiftly to prevent any potentially life-threatening loss of fluid. Therefore, a vigorous response is appropriate. On the other hand, in a situation of volume expansion the increase in GFR could be beneficial as it tends to return the ECF volume to normal. 1 The concentration of Na+ and Clin the tubular fluid reaching the macula densa is dependent on the rate of flow along the ascending loop of Henle. This tubular segment is water impermeable but it actively reabsorbs NaCl, so that the concentration of salt (and the osmolality) of the tubular fluid decreases as it moves up the ascending limb. An increased flow rate will decrease the time available to reabsorb the salt and cause higher Na+ and Clconcentrations in the tubular fluid reaching the macula densa.

Urothelium (Transitional epithelium)

Urothelial cells have a sensory function - mechanosensation, chemosensation, trigger activity in subjacent nerves by releasing mediators like NO and ATP • 3 layers 1.Umbrella cell layer (tight junctions between cells, plaques' forming an asymmetrical unit membrane on outer surface (uroplakin protein), common host site for UTIs 2.Intermediate layer 3.Basal cell layer (attached to basement membrane & stem cells)

Tubular Fluid/ Plasma (TF/P) ratio

Using micropuncture techniques, it is possible to obtain samples of luminal fluid from different tubular segments and analyze their composition. Since inulin is neither absorbed nor secreted, any increase in its concentration in a sample of tubular fluid above that in the plasma (or filtrate) can only be due to water reabsorption proximal to the site of sampling in the tubule. For instance, if the concentration of inulin in a sample of tubular fluid (TFINU) taken from a specific site is 4 times its plasma concentration (TFINU/PINU =4), it indicates that 3/4 of the filtered water has been reabsorbed in the segment of tubule proximal to the sampling site. The urine/plasma inulin ratio, UINU / PINU, is an indication of the fraction of filtered water that is reabsorbed throughout the tubular system in the course of formation of the urine. For example, a UINU / PINU = 100 indicates that 99% of the filtered water has been reabsorbed by the tubular system and only 1% is excreted in the urine.

Distal flow also impacts ADH. Because ADH increases water reabsorption in the late distal tubule and collecting duct; it reduces distake flow. But this does not decrease K secretion because ADH also stimulates Na reabsorption and K secretion in principal cells thus counteracting reduced distal flow. Distal flow is an autoregulatory mechanism to balance K due to changes in ECF and water balance.

Volume Depletion: With low ECF there is high AII and high Aldo that causes K secretion due to Na reabsorption. High proximal absorption of water leads to lower distal flow so less K secretion. Antidiuresis: High ADH causes K secretion due to high distal flow that thus leads to low K secretion. Distal flow opposes the first effect and thus leads to no net change in K excretion.

Volume of ECF/ICF after 1.5 L water: Effects with: B.Infusion of hypertonic saline

Volume of the extracellular compartment increases while that of the intracellular compartment decreases. The net result is addition of NaCl in excess of H2O. NaCl remains in the extracellular fluid, increasing osmolality; water leaves the cells by osmosis until the osmolal concentration of the intracellular fluid rises to equal that of the extracellular fluid. Therefore, in the new steady state the extracellular compartment is expanded, the intracellular is contracted, and the osmolality of both is greater than normal.

C.Excessive loss of solute (e.g., excessive sweating, followed by replacement of water, but not salt).

Water moves into cells, resulting in cellular hydration and extracellular dehydration. The loss of NaCl in excess of H2O results in decreased osmolality of extracellular fluid. Consequently, H2O will shift from the extracellular into the intracellular space. The new steady state will show contraction of the extracellular compartment and expansion of the intracellular space, with both having an osmolality that is less than normal. [Loss of NaCl in excess of water is common in patients with adrenal insufficiency.]

The magnitude of tubular water reabsorption can be easily appreciated. Since, as mentioned earlier, the GFR, or volume of plasma filtered at the glomerulus per unit time, amounts to about 180 L/day and we excrete a volume of urine of roughly1.8 L/day, it follows that 99% of the filtered water must be reabsorbed by the tubules. We can use Eqs. [3] and [4] to calculate the percentage of the filtered load of any freely filtered substance that is reabsorbed. The Table below shows the values for several plasma components of interest.

We see that, because of the large amounts of material filtered, the useful plasma components are largely reabsorbed. Some, like glucose, are completely reabsorbed (under normal conditions) and thus are not regulated by the kidneys. Others, like water and sodium, are subject to physiological control, and the amount reabsorbed varies according to the body's needs. Waste products like urea are largely excreted, but some reabsorption takes place due to passive diffusion after the reabsorption of water concentrates urea in the luminal fluid.

Regulation of Proximal Tubule Sodium Reabsorption

When ECF Na+ content and volume are normal ("euvolemia"), approximately two-thirds of the filtered Na+ is reabsorbed in the proximal segment. Increases in the Na+ content and ECF volume tend to reduce the fraction of filtered Na+ reabsorbed proximally, while decreases in Na+ content and ECF volume have the opposite effect. Several regulatory mechanisms are involved in altering the "set point", i.e., the fraction of filtered Na+ reabsorbed proximally, in response to ECF changes. • Sympathetic nerves innervate the proximal tubule. Increased sympathetic stimulation (in response to ECF volume decreases) stimulates proximal Na+ reabsorption. • Angiotensin II stimulates proximal Na+ reabsorption. (The effects of sympathetic activity and AII in proximal Na+ reabsorption are, in part, mediated by stimulation of the Na+-H+ exchanger, NHE3). • Starling forces in peritubular capillaries influence Na+ reabsorption. When Na+ and, secondarily, water are retained in ECF/plasma, RPF and GFR increase but not in the same proportion, so that the filtration fraction is reduced leading to lower protein concentration and plasma oncotic pressure in the peritubular capillaries. Since peritubular capillary hydrostatic pressure also rises, the reabsorption of fluid is reduced, retarding sodium reabsorption by the tubules. Loss of Na+ and water from the plasma has the opposite effect, i.e., an increase in Starling forces tending to enhance Na+ reabsorption. • Nitric oxide (NO), stimulated by volume expansion, inhibits Na+ reabsorption at several tubular sites including the proximal tubules.

Parts of the kidney

cortex: darkly stained, millions of spheroid renal corpuscles, which are tufts of blood vessels encapsulated by the spheroid beginning of the nephron. medulla: extends from the cortex towards the interior of the kidney. It is highly ordered and packed with radially-oriented tubules and capillaries.series of cone-shaped structures, each known as a medullary (renal) pyramid whose tip is the renal papilla. The human kidney typically has 6 to 14 renal pyramids, with 8 being most common. juxtamedullary cortex: the region of cortex close to the medulla is termed juxtamedullary medullary ray: penetrate the cortex from the medulla and taper toward the renal capsule; they are prolongations of structures radially oriented within the medullary pyramid. renal column: cortex between the pyramids towards the renal sinus capsule: The kidney has a dense connective tissue envelope that lines the external surface of this organ. There is no surface mesothelium because the kidney is embedded in perirenal fat tissue. hilum (or hilus): Observe the indented hilum. This is where the renal blood vessels enter and leave. renal sinus: The region containing loose or adipose connective tissue in the interior of the kidney. The renal pelvis, ureter, and veins travel within the renal sinus before exiting at the hilus; major arteries enter at the hilum and travel to the cortex within the renal sinus.

c. Thick ascending limb and distal tubule

e. The distal part of the renal tubule consists of the thick ascending limb (in medulla), the distal straight tubules (in cortex) and the distal convoluted tubule (distal to the macula densa) before joining the collecting tubule. The cells of the thick ascending limb, the distal straight tubule and the distal convoluted tubule resemble one another in that they have few microvilli and the cytoplasm is fairly empty looking. Cells in the distal tubule and thick ascending limb pump ions out of the lumen of the tubule while retaining water, so the contents of the tubule are kept hypotonic with respect to blood. ATPase pumps along the lateral cell membranes accomplish the ion pumping. The pumps get their ATP from mitochondria packed into basal enfoldings of the cell membranes

e. The amount of fructose filtered per minute. f. The amount of fructose reabsorbed per minute. g. The percentage of the filtered urea which is reabsorbed. h. In addition to the data given above, the following renal clearances were determined simultaneously: Cxylose = 90 ml/min; Chistamine = l60 ml/min. Therefore the tubule cells have: A. Reabsorbed both histamine and xylose. B. Secreted both histamine and xylose. C. Reabsorbed histamine and secreted xylose. D. Reabsorbed xylose and secreted histamine. E. Neither secreted nor reabsorbed xylose or histamine.

h. Since the Cxylose is less than the GFR, despite the fact that xylose is a small sugar and therefore freely filtered, it must have been partially reabsorbed. Since the Chistamine is greater than the GFR, some histamine must have been secreted. The answer is therefore "D". Note that the data in no way rule out the possibility that in some regions of the nephron there may also be secretion of xylose and reabsorption of histamine. All that we can observe is the net effect.

Problem 1 Renal blood flow and glomerular filtration rate are regulated by the arteriolar resistances under the influence of neural and humoral factors. The arteriolar tone can be affected also by drugs used in the treatment of diverse disorders and it is important to understand the direction of changes in RBF and GFR elicited by these substances. A. What would be the effect on renal blood flow, glomerular capillary pressure and GFR of the administration of? i) A drug that proportionately constricts both afferent and efferent arterioles. ii) An angiotensin I converting enzyme inhibitor.

i) A drug that proportionately constricts both afferent and efferent arterioles. Although constriction of both arterioles has opposite effects on glomerular capillary pressure and the final effect of PGC is difficult to predict, the reduction in RBF increases the average protein concentration and plasma oncotic pressure inside the capillaries, which reduces the net filtration force. Vasoconstrictors also tend to constrict mesangial cells and podocytes reducing the filtration area. As a consequence GFR is reduced. ii) An angiotensin I converting enzyme inhibitor. Administration of a CEI will reduce the production of AII with subsequent dilation of the arterioles and increase of RBF. Since AII is a more effective constrictor of efferent arterioles ACE inhibitors tend to reduce glomerular capillary pressure, PGC. Some investigators believe that the progressive nature of much chronic renal disease does not result simply from persistent activity of the underlying immunologic or metabolic disorder (such as diabetes mellitus), but rather from the high pressure within the glomerular capillary that leads to thickening of the capillary wall (sclerosis) and diminished function. Consequently, physicians treat appropriate renal patients (particularly diabetic patients) with CEI, reasoning that it is a relatively nontoxic agent with few side effects and by decreasing efferent arterial resistance it might be useful. The treatment has the potential drawback of reducing GFR. Obviously, one would not wish to lower the glomerular filtration rate in a patient with sufficiently advanced renal disease. However, this can be easily monitored by measuring plasma creatinine and the drug discontinued before the patient is endangered. ACE inhibitors, as well as AII receptor antagonists, are frequently used clinically to treat hypertension, congestive heart failure and complications from diabetes.

Blood Flow: to, within, and from the kidney

interlobar arteries and veins: large blood vessels in the renal sinus on either side of the renal papilla(-ae). In a multi-lobed kidney such as in humans, interlobar vessels are oriented radially and course between (not within) the medullary pyramids. They are branches of the segmental arteries and veins. arcuate arteries and veins: The interlobar arteries give rise to smaller branches that arch along the interface between the medulla and cortex. Unlike the interlobar vessels the arcuate vessels are surrounded by cortex because they travel just beyond the interface between cortex and medulla and they are oriented relatively parallel to the kidney surface. interlobular arteries and veins: The arcuate arteries give rise to branches that are again (like the interlobars) radial in their orientation. These travel between the medullary rays into the cortex. The interlobular arteries and veins travel together. afferent arteriole: The interlobular arteries give rise to the afferent arteriole of the glomerulus. These glomeruli represent the first of the two capillary beds that constitute the portal system in the kidney. An efferent arteriole leaves the glomerulus and gives rise to the second capillary bed.

The major baroreceptors involved in the sensing of the Na+ content are:

• Receptors in the central great arterial vessels (carotid sinus, aortic arch) that respond to increased blood pressure secondary to expansion of the "effective circulating volume"(ECV) by reducing sympathetic nervous system traffic, while decreases in stretch at the great vessel receptors lead to increased sympathetic nervous system signaling. • The cardiac atria respond to increased stretch by releasing atrial natriuretic peptide (ANP). Conversely, secretion of ANP is reduced if atrial filling is decreased by reductions in plasma volume. Like the arterial receptors, the cardiopulmonary receptors also alter sympathetic efferent traffic; it increases with reduced stretch and decreases with increased stretch. • The afferent arterioles in the kidney are volume/stretch receptors. Decreased stretch stimulates secretion of renin by the juxtaglomerular apparatus, while increases in transmural pressure reduce renin secretion. As described, renin leads to the formation of AII, a potent vasoconstrictor that stimulates secretion of aldosterone, the principal mineralocorticoid hormone, from the adrenal cortex.

Nephrogenesis

• Terminal tips of the collecting duct induce the formation of nephrons from mesenchymal tissue (metanephric mesenchyme). • Nephrons form in a nephroblastic zone, underneath the kidney capsule Horseshoe kidney - lobes unite before cranial migration • Double ureter - mistimed branching • Kidneys come in many sizes and shapes • The arrangement of cortex/medulla/ calyx is common • Mammalian kidneys come in many sizes and shapes, but the [cortex / medulla / calyx] topology is always preserved.

lumen of the ureter

is lined by a transitional epithelium and turns to stratified squamous epithelium when the lumen is distended. The loosely organized smooth muscle of the muscularis is responsible for peristalsis that moves the urine from the pelvis of the kidney to the urinary bladder.

Intraglomerular mesangial cells control glomerular perfusion via

mesangial cells can contract like smooth muscle and make/respond to vasodilators/constrictors

Urinary Bladder

mucosa of an empty bladder is folded, has transitional epithelium, the muscularis propria forms the (smooth) detrusor muscle which contracts during urination, the bladder is large, with a thick wall.There is a muscularis mucosae layer in the bladder

Na I K I BUN I glu Cl I Bi I Cr 142 I 6 I 48 I 389 107 I 28 I 1.6 Hemoglobin a1c 10.0 Urinalysis revealed: 2+ protein 4+ glucose Now potasium went from 6 -> 7.1 Why does ACE inhibitors cause rise in potassium? If postassium binder given and potassium falls greatly, what causes it?

p K/potassium rises with ACE inhibitors because = low AII = low aldosterone = no K secretion Low AII = low PCT reabsorption = more distal flow = only adds a little bit of K secretion Switching from ACE inhibitor to fix potassium using sodium polystyrene sulfonate (cation exchange resin binds potassium in gut) + insulin. Now potassium drops to 3.2 (so the 6.0 was actually still with reduced K stores)

What is the disorder? NpH: 7.4 NHCO3: 24mEq/L PCO2: 40 mmHg

pH HCO3 PCO2 a. 7.34 pH, 15 HCO3, 29 PCO2 Super low HCO3, partially compensated pH/pCO2 is metabolic acidosis b. 7.49 pH, 35 HCO3, 48 PCO2 Super high HCO3, more basic pH, high PCO2 to compensate is metabolic alkalosis c. 7.47 pH, 14 HCO3, 20 PCO2 High pH, super low PCO2, low HCO3 Chronic Respiratory Alkalosis d. 7.34 pH, 31 HCO3, PCO2 60 Chronic Respiratory Acidosis e. 7.26 pH, 26 HCO3, PCO2 60 Acute Respiratory Acidosis f. 7.62 pH, 20 HCO3, 20 PCO2 Acute Respiratory Alkalosis g. 7.09 pH, 15 HCO3, 50 PCO2 Metabolic Alkalosis + Respiratory Acidosis h. 7.40 pH, 15 HCO3, 25 PCO3 Metabolic Acidosis + Respiratory Alkalosis

The kidneys receive about 20% of the cardiac output of 5 L/min, i.e., the renal blood flow (RBF) amounts to 1 L of blood/min, a very large flow considering the size of the kidneys.

s. Assuming an average hematocrit of 40%, this blood flow translates into a renal plasma flow (RPF) of 600 ml/min. Since about 20% of the incoming plasma is filtered at the glomerulus, the glomerular filtration rate (GFR) is near 125 ml/min (or 180 L/day). The remaining 80% of the plasma entering the glomeruli flows into the efferent arterioles. The filtrate contains all substances present in plasma, except proteins, at virtually the same concentration as in plasma. The filtration rate, as in the systemic circulation, is dependent on the Starling forces (the hydrostatic and oncotic pressures), which drive fluid from the lumen of the glomerular capillaries into Bowman's space: GFR = Kf [(PGC - PBS) -(πGC - πBS)]

The ureters and bladder layers like the gut. Bladder is specialized to strecth + be a barrier.

stellate lumen: transitional epithelium, loose lamina propria/submucosa • 3 poorly defined layers in the muscularis Is small.

b. The thin segment

t is a part of the tubule with a diameter of 20-40 microns that is lined by a squamous epithelium about 1-2 microns in height. The nuclei of the cells bulge into the lumen, and this characteristic, together with the thickness of the epithelium, enables us to distinguish thin segments from medullary blood vessels composed of endothelium. The cytoplasm of thin segment cells has little in the way of organelles, suggesting that ion transport in this part of the renal tubule is largely passive. All thin segments are located entirely within the medulla.

Urethra size

~20cm in males, prostatic ~4cm membranous ~1cm penile ~15cm ~3-5 cm in females Transitional epithelium thins to a stratified or pseudostratified epithelium, terminating distally in stratified squamous epithelium

Sodium Distribution and Balance

• Distribution: About 40% of total body sodium is in bone, not available for metabolic processes. - Available sodium: • ECF : 140 mEq/L 15 L = 2,100 mEq • ICF : 10 mEq/L 30 L = 300 mEq • Almost 90% of available body Na is in ECF • Intake: Average of 100 mEq/day. It can range from <1 mEq/day to ≅400 mEq/day. Regulatory mechanisms unclear. • Excretion: Renal, balancing ingestion. Negligible (< 10 mEq/day) stool and sweat losses. (Note: 1 Eq Na+ = 23 g Na+ = 58.4 g NaCl)

Renal Function & Clearance

• Excretion of metabolic waste products (urea, uric acid, creatinine) and foreign chemicals • Secretion of hormones (renin, erythropoeitin) and active vitamin D

Factors Altering Potassium Secretion Because BK chanels open due to bending of principal cell cillium

• Flow of Tubular Fluid : Increased flow enhances K secretion (this is why some diuretics lead to hypokalemia)

Extra-renal Potassium Homeostasis

∆ [K]plasma = K ingested / ECF volume ≅ 30 mEq / 15 L = 2 mEq/L

The Renin-Angiotensin-Aldosterone and ADH mechanisms for maintenance of arterial blood pressure and extracellular fluid volume

**Draw out map**

47. Which of the following statements is FALSE? A. Most body water is in the intracellular space. B. Usually more than half of body mass consists of water. C. The concentration of protein is higher in plasma than in cells. D. The concentration of equivalents is higher in cells than in plasma. E. Intravenous infusion of isotonic saline decreases plasma oncotic pressure.

. C. The concentration of protein is higher in plasma than in cells.

pothetical model of nephrin assembly to form the isoporous filter of the podocyte slit diaphragm

. (A) Schematic domain structure of nephrin. Ig repeats + disulfide bridges B. interdigitating association of four nephrin molecules in slit between two foot processes. Ig repeats 1-6 of a nephrin molecule of one foot process associate in an interdigitating fashion with Ig repeats 1-6 . Free cysteine may w/ nephrin or unknown molecule to connect to cytoskeleton.

2. Renal Tubules.

. The ultrafiltrate exits Bowman's capsule at the urinary pole, and enters the renal tubule. The following segments are part of the renal tubule (in order of appearance from the urinary pole): the proximal convoluted tubule (in cortex), the proximal straight tubule (in the cortex) the thick descending limb (in the medulla), the thin segment (part of the descending limb of Henle's loop for short loops and part of both descending and ascending limbs of longloops), the thick ascending limb (in the medulla), the distal straight tubule (in the cortex). The macula densa is a specialization of part of the wall of the distal tubule where the tubule is apposed to the vascular pole of the originating renal corpuscle. The distal tubule becomes convoluted distal tubule after the macula dense and leads to a collecting tubule, and eventually to collecting ducts that exit the cortex within medullary rays. Despite this prolific naming of segments, there are really only three basic tubular morphologies encountered. Considered together with the collecting tubule/duct, the vasculature, and the nondescript renal lymphatics, these account for every recognizable tubular profile in the kidney

Calculate the secreted H+ by renal tubules. H+ must be secreted to reabsorb HCO3, some H+ is secreted in different forms. GFR: 100L/day P HCO3: 24mM U Flow: 1L/day U NH4+: 50 mM U HCO3: 5 mM Titratable acid: 30 mM U pH: 6.8

1. Total H+ secreted includes: -amount to reabsorb bicarbonate -amount used to replace bicarb due to buffering fixed acid -H+ secreted to reabsorb bicarbonate = filtered load of bicarbonate - lost U bicarbonate HCO3 filtered load: GFR * U HCO3 = 100*24 = 2400mmoles/day HCO3 excreted: U HCO3 * U Flow = 5*1 = 5mmoles/day H+ secreted for reabsorption: 2400 - 5 = 2395 mmoles/day 2. Total H+ secreted to replace HCO3 lost in buffering acid and lost in urine is titratable acid + NH4 excreted Titratable Acid: TA * U Flow = 30mM * 1 = 30 mmoles/day NH4+: U NH4+ * U F = 50mM * 1 = 50 mmoles/day Total H secreted is 2395 + 80 = 2475 mmoles/day (total HCO3 added by kidneys into blood stream per day) Net Acid Excretion (NAE): U TA + U NH4 - Ex. Bicarb = 30 + 50 - 5 = 75 mmoles/day NAE must eqaual HCO4 lost every day in buffering fixed acid if there is proper acid base balance.

There are three locations where the ureter is constricted and where renal caliculi (kidney stones) may get caught and obstruct the ureter:

1. Ureteropelvic junction - where ureter joins renal pelvis 2. Where ureter crosses pelvic brim 3. Ureterovesicular junction - where ureter enters wall of bladder

Sodium Na and H2O excretion are affected by

1. Vasoconstriction of glomeruli arterioles by sym. nerves reduces GFR and reduces filtered Na load so less excretable. 2. Sym nerv and AII stimulate PCT to increase Na reabsorption. 3. Aldosterone increases Na reabsorption in distable tubule and CD. 4. Increased ECF (despite low ECV) so atrial myocytes release ANP so plasma ANP elevated. ANP Effects of ANP opposed by others that REDUCE sodium. 5. Increased AII stimulates ADH that reabsorbs water in distal tubule and CD. 6. Lower MAP and low firing of baroreceptors causes ADH secretion 7. Cardiopulmonary "volume" receptors will supresses ADH secretion, so even though ADH is high its rise is blunted. 8. High ADH and lower GFR and high water reabsorbtion at PCT secondary to salt causes less water to be excreted 9. AII stimulates thirst and increased water ingestion 10. Net effect is retention of salt and water. So eCF increases and venous congestion leads to edema formation. 11. Plasma renin, NE, Aldosterone, ANP, ADH elevated. Mild CHF: restoration of CO and BP possible (if expansion of PV enough to increase cardiac EDV/SV). Plasma levels of factors returm to normale.

Explain Collecting duct transport of HCO3

10% HCO3 is reabsorbed by Intercalated A-Type AE-1 Cell in CD Secreting of HCO3 in alkalosis occurs by Intercalated B-type cell (Pendrin, HCO3/Cl antiport)

11. Glomerular capillary hypertension is a predictor of glomerular sclerosis. Which of the following can lead to an increased glomerular capillary pressure? A. release of adenosine by macula densa cells B. slight increase in renin release by juxtaglomerular cells C. administration of ACE inhibitors D. activation of the tubulo-glomerular feedback mechanism E. increased afferent vasoconstriction

11. Answer B A. False. Adenosine constricts afferent arterioles decreasing PGC B. Correct. Renin release will increase Ang II which at low concentrations will preferentially constricts efferent arterioles increasing PGC C. False. ACE inhibitors will reduce Ang II levels and dilate efferent arterioles (see B) D. False. Tubulo-glomerular feedback activation results in the release of adenosine (see A) E. False. Afferent constriction reduces PGC

A new drug tested in animal studies was found to be freely filtered and a plasma concentration of 1mg/ml it was excreted at a rate of 20 mg/min. The creatinine clearance, simultaneously determined, was found to be 130 ml/min. You may conclude that: A. the drug is secreted by the renal tubules. B. the renal clearance of the drug is 20 ml/min. C. the drug's U/P ratio is larger than the creatinine U/P ratio. D. the filtered load of the drug is 20 mg/min. E. the concentration of the drug in the glomerular filtrate is 20 mg/ml

2. Answer: B A. False. The clearance of the substance is: excretion rate/plasma concentration = 20/1 = 10 ml/min, much lower than the GFR = Ccreat=130 ml/min. Therefore, this substance is reabsorbed. B. Correct. See A C. False. Since the substance is reabsorbed its U/P ratio must be lower than the creatinine U/P ratio. D. False. The filtered load is GFR x plasma concentration = 130 ml/min x 1mg/ml =130 mg/min E. False. Since the substance is freely filtered its concentration in the glomerular filtrate is the same as that in plasma: 1mg/ml.

Explain Proximal Tubule Reabsorption of HCO3

2/3 HCO3 reabsorbed by Na/H antiport in PCT. 1/3 HCO3 reabsorbed by ATP H gradient in PCT. Main PCT HCO3 reabsorbers: NA/K ATP, 1NA/3HCO3 Symporter, HCO3/CL Antiport (Cl comes in from blood and HCO3 exits to blood).

Regulation of Sodium Reabsorption by the Distal Segments

3% 25,200 mEq = 750 mEq Intake range = 1 - 400 mEq Sodium reabsorption 'fine tuned' by: - Aldosterone (stimulates) - ANP and other natriuretic factors (inhibit) - Local factors: NO, prostanoids, kinins (inhibit)

A 34 year-old man with a recent episode of colitis and weight loss is admitted to the hospital after taking a large amount of ibuprofen. The attending physician suspects renal afferent arteriolar constriction. Which of the following is most likely to increase with afferent vasoconstriction? A. plasma creatinine concentration B. glomerular capillary hydrostatic pressure C. renal plasma flow D. glomerular filtration rate E. filtered load of glucose

3. Answer: A A. Correct. In the steady state the amount of creatinine produced each day equals that excreted per day. The latter is the amount filtered per day (Pcreat GFR) since creatinine is eliminated only by filtration. Constriction of the afferent arteriole decreases PGC in the glomerulus and decreases GFR. When GFR falls, the amount of creatinine filtered falls below the amount produced and Pcreat increases. It rises until the filtered load (increased Pcreat x decreased GFR) once again matches the daily creatinine production. B. False. See A. C. False. For a given renal artery pressure, a rise in afferent arteriolar resistance would decrease renal blood flow and plasma flow. D. False. See A. E. False. The filtered load of glucose (FGlu = GFR x PGlu) would decrease with a decrease in GFR.

4. At the end of the proximal tubule, where 2/3 of the filtered water had been reabsorbed, which of the following would be LEAST LIKELY in a normal individual? A. The TF/P ratio for inulin would be 3. B. The TF/P ratio for Na+ would be 3. C. The osmolarity of the lumenal contents would be the same as that of the filtrate. D. The TF/P ratio for glucose would be zero. E. The TF/P ratio for PAH would be greater than that of inulin.

4. Answer: B A. True. If only 1/3 of the filtrate remained, inulin would have been concentrated 3 fold. B. False. The TF/P ratio for Na+ equals 1 everywhere in the proximal tubule because Na+ is reabsorbed isosmotically, and it leaves behind a smaller volume of a solution with the same Na+ concentration as the filtrate. C. True. Isosmotic reabsorption occurs in the proximal tubule. D. True. Normally, all filtered glucose and amino acids are completely reabsorbed in the proximal tubule. E. True. Since PAH is freely filtered and is also secreted, its TF/P ratio would exceed that of inulin

5. In a patient with plasma creatinine concentration of 1.0 mg/100 ml, urine creatinine concentration of 200 mg/100 ml and urine flow rate of 0.5 ml/min, which of the following statements is FALSE? (Assume negligible creatinine secretion): A. The GFR is 100 ml/min. B. The kidneys filter 1 mg of creatinine every minute. C. The kidneys reabsorb 99.5 % of the filtered water. D. The creatinine concentration in the plasma leaving the peritubular capillaries is 1 mg/100 ml. E. The kidneys excrete 1 mg/min of creatinine.

5. Answer: D A. True. GFR= Ucrea V /Pcrea = 200 0.5 /1 = 100 ml/min. B. True. The filtered load of creatinine is the same as its excretion rate, or 200 mg/100 ml x 0.5 ml/min = 1 mg/min. C. True. The Ucrea/Pcrea =200, therefore 199/200=0.995 or 99.5% of the filtered water is reabsorbed. D. False. Since water, but not creatinine, gets reabsorbed from the tubule, the concentration of creatinine in the plasma leaving the peritubular capillaries must be less than 1 mg/100 ml. E. True. See B.

Grid

6. Answer: B3 There are three glomeruli. The one covering AB-12 has no clear vascular pole. The one covering AB-45 has a vascular pole and distal tubule (in AB-5 but the distal tubule has no clear macula densa. B3, however, has a vascular pole (note the bridging arterioles) and a DT with a concentration of epithelial cells on the rightward wall towards the glomerulus. This is the clearest macula densa in the image. 7. Answer: B2 The accompanying interlobular artery is seen in the lower right corner of C1. Both have unstaining RBCs filling the lumen. 8. Answer: C4 A3 has a DT in the upper right corner B3 has two prominent DTs C4 has all PCTs

25. Renin secretion is increased by: A an increase in circulating epinephrine. B. hypertension. C. increased sodium ingestion. D. an increase in aldosterone output. E. an increase in K+ intake

A an increase in circulating epinephrine.

Problem 1 In a nephron micropuncture study it was found that in the afferent arteriole the mean protein concentration was 8.0 grams/100 ml plasma, the creatinine concentration 1mg/dL, the Na+ concentration 140 mEq/L, and the filtration fraction was 20%. A) Assuming no significant filtration of protein, what were the mean protein, creatinine and sodium concentrations in the efferent arterioles leading to the peritubular capillaries? B) Assuming also not significant secretion of creatinine, what would be the mean protein, creatinine and sodium concentrations in the venous plasma leaving the kidney?

A) Assuming no significant filtration of protein, what were the mean protein, creatinine and sodium concentrations in the efferent arterioles leading to the peritubular capillaries? Since 20% of the incoming plasma gets filtered, 80 ml of every 100 ml of plasma remains in the efferent arteriole, but with the same amount of protein: 8.0 grams/80 ml = 10.0 grams/100 ml. Creatinine and sodium are filtered at the same rate than water; therefore their concentrations in the efferent arteriole are the same than in the afferent arteriole. B) Assuming also not significant secretion of creatinine, what would be the mean protein, creatinine and sodium concentrations in the venous plasma leaving the kidney? Because practically all the water and sodium filtered gets reabsorbed by the tubules, the concentration of sodium in the venous plasma is still the same as that in the arterioles. Reabsorption of water will bring the concentration of protein in the venous plasma back to its original value in the afferent arteriole 8 g/100 ml. Since 20% of the creatinine entering the glomerulus is filtered and not reabsorbed (but most of the water is reabsorbed), the concentration of creatinine in the venous plasma must be 20% lower than in the afferent arteriole or 0.8 mg/dL.

Problem 3 A 53-year old man admitted for treatment of a small cell carcinoma, which is frequently associated with SIADH (syndrome of inappropriate ADH secretion), exhibits normal skin turgor and blood pressure in the supine, sitting and standing position. Initial blood and urine tests reveal the following: Na+: 118 mEq/LUrine osmolality: 916 mOsm/kg A. How would you characterize total body sodium and water contents in this patient (normal, reduced, increased)? How do you know? Can you estimate plasma osmolality?

A) Since this patient exhibits normal skin turgor and blood pressure in the supine, sitting and standing position we can assume that his ECF volume is normal. Without further information it would be reasonable to assume that body Na+ content was normal as well. However, in this patient the Na+ concentration is below normal. Hence, the total extracellular Na+ content (the product of volume x concentration) is below the normal. The reduced Na+ concentration lowers the body osmolarity, in this case, to about 236 mOsM (normal: 275-295 mosm/L), and because of water equilibration, the intracellular osmolarity is similarly reduced. (Body fluid osmolarity can be simply estimated by doubling the PNa, since Na+ and the accompanying anions (mainly Cland HCO3 - ) account for most of the ECF solutes.) The reduced osmolarity indicates increased water content, presumably because the patient is unable to excrete water in the face of a continuous secretion of ADH ('syndrome of inappropriate ADH secretion'). Water retention resulting in hyponatremia is generally associated with an inability to suppress the secretion of ADH, either because of its ectopic production (as in SIADH) or because of effective circulating volume depletion. In the latter case, when over 10% of the plasma volume is reduced, ADH secretion is elevated. Although the resulting water retention dilutes body fluids and reduces osmolarity, ADH secretion remains elevated. Maintaining blood pressure in this situation becomes a higher priority than maintaining a normal osmolarity.

A. Calculate the osmolar clearance if plasma osmolality is about 300 mosm/L

A) The osmolar clearance is: COSM = UOSM * V / POSM = (600 mosmoles/day)/(300 mosmoles/L) = 2 L/day since the numerator in the above equation represents the excretion rate of all solutes.

Peter G. is a 16-year-old high school student who plays forward at the basketball team. He noticed after practice that his ankles were swollen but did not worry much about it, thinking that it was a hot day at the gym and that he also had run a lot. However the swelling persisted on the next day and his father took him to see his pediatrician. On physical examination the physician noted edema of his extremities and ordered a number of tests. Some of the plasma and 24-hour urine values are shown below: Plasma Na+ 142 mEq/L (normal : 136-145 mEq/L) Plasma creatinine 1.6 mg/dL (normal : 0.6 - 1.2 mg/dL) Plasma albumin 1 g/dL (normal : 4.5 g/dL ) 24-hour Urine protein: 4 g (normal : < 150 mg) 24-hour Urine creatinine: 1.6 g (normal) 24-hour Urine volume: 1.6 L (normal) Peter was diagnosed with minimal change nephrotic syndrome (MCNS, or minimal change disease in the US), an idiopathic glomerular disease without significant histological changes under the optical microscope (hence the name). Electron microscope studies have revealed a loss of anionic surface charge at the basement membrane and podocytes, and fusion of pedicels with reduction of filtration slits. The disease has an excellent prognosis with steroid therapy

A) Why was Peter excreting large amounts of protein in his urine (proteinuria)? The loss of anionic surface charge from the basement membrane and podocytes hinders their ability to reject plasma proteins. The increased glomerular permeability to large anions leads to proteinuria (compare data for this patient and the normal urine protein in the Table provided). B) Explain the hypoalbuminemia and its relation to edema formation. Because of the high rate of protein filtration at the glomerulus, the plasma concentration of albumin, the main plasma protein decreases in this subject to about 1/5 of the normal value. The loss of plasma albumin leads to a reduced plasma oncotic pressure and low plasma oncotic pressure favors movement of fluid from the vascular space into the interstitium. [Note that the interstitial protein concentration will also decline over time, since less protein gets filtered from plasma and the lymphatics keeps removing protein. As a result, the transcapillary oncotic pressure difference does not decrease much, and edema is not apparent, until the plasma albumin gets below 1.5- 2 g/dL]. C) Is his GFR estimated from the creatinine clearance normal? What do you expect to be the effect of hypoalbuminemia on GFR? The creatinine clearance is calculated as: Ccreat= creatinine excretion rate/plasma creatinine= 1.6 (g/day)/1.6 (mg/dL) = 1,600 (mg/day)/16 (mg/L) = 100 L/day. This is a reduced rate of glomerular filtration. Normal GFR values for a young adult are 130-190 L/day. The rate of glomerular filtration is a function of both: the filtration permeability and the net filtration pressure. A decrease in plasma oncotic pressure favors filtration out of the capillaries by increasing the net filtration pressure. On the other hand, although in patients with nephrotic syndrome the glomerulus is more permeable to proteins because of the loss of anionic charges, the permeability to water and small solutes decreases as a consequence of the fusion of pedicels and the subsequent reduction in filtration slits. Usually these patients exhibit no change or a small reduction in GFR. D) Assuming that all the urinary protein is albumin, estimate how much the filtration barrier is restricting albumin compared with a freely filtered solute. If albumin were freely filtered its filtered load would be Palb x GFR, i.e., 10 g/L x 100 L/day = 1000 g/day (note the conversion from g/dL to g/L). The measured excretion rate of 4 g/day represents a fraction of 0.4%. Thus, even in a disease in which glomerular permeability to albumin appears clinically to be markedly increased, virtually the entire circulating albumin remains unfiltered. F) Over the next few weeks of treatment with corticosteroids the physician followed the status of Peter's renal function by measuring his plasma creatinine concentration. The weekly reported values were (in mg/dL): 1.8, 1.5, 1.2 and 0.9. What is happening to Peter's GFR? Since we can assume a constant rate of creatinine production, the clearance of creatinine, and hence the GFR, changes inversely to PCr. The GFR values at successive weeks are: 89, 107, 133 and 178. This suggests that Peter's GFR, after an initial decline, is recovering towards normal. G) Discuss the limitations of the plasma creatinine measurement as an index of the GFR. As mentioned earlier, secretion of creatinine in humans results in a creatinine clearance slightly higher than the GFR. This is partially offset by the fact that the test for plasma creatinine overestimates the true value of PCr. The small error has not prevented the use of PCr as a clinical tool to follow changes in renal function. However in renal patients, with lowered filtration, the discrepancy between the Ccr and GFR increases because less creatinine is now being cleared by filtration relative to secretion. Thus, the actual GFR may be even lower that the value estimated by the creatinine clearance. Using just the plasma creatinine as an index of renal function adds other uncertainties. The assumption here is that creatinine production does not change and thus an increased plasma concentration of creatinine reflects a decreased filtration. However, plasma creatinine can also be elevated as a result of cardiac damage or muscle trauma, or even severe exercise. On the other hand, plasma creatinine is often low in elderly patients with normal renal function because they have a smaller muscle mass.

Use the following information to answer questions 35 and 36 below: An experimental animal, with a constant plasma inulin concentration, was found to have TF/P ratios for inulin and Na+ of 4.0 and 1.0, respectively, at the end of the proximal tubule. 35. The percent of filtered water that has been reabsorbed up to this point in the tubule is:

A. 25 %. B. 50 %. C. 67 %. D. 75 %.* Cin = GFR Cl Na/In = 1/4 Free Water Reabs: 3/4 E. 80 %. 36. The percent of filtered Na+ that has been reabsorbed up to this point in the tubule is: A. 25 %. B. 50 %. C. 67 %. D. 75 %.Na and H2O same E. 80 %.

7. Which of the following will increase proximal tubule Na+ absorption? A. Constriction of efferent arterioles. B. Increased plasma ADH concentration. C. Proximal tubule NO production D. Decreased plasma glucose concentration. E. Decreased Angiotensin II.

A. Constriction of efferent arterioles

23. Which of the following transport processes is appropriately matched to a location in the nephron:

A. Na+-Cl+ cotransport: distal tubule B. H+-K+ ATPase: collecting duct C. Aquaporin 2: apical collecting duct D. Na+-K+-2 Cl- cotransport: thick ascending limb E. Epithelial Na+ channel: distal tubule

16. Which of the following substances is likely to increase renal blood flow? A. Prostaglandins B. Endothelin C. Angiotensin II D. Epinephrine E. Adenosine

A. Prostaglandins increase RBF Endothelin, AII, Epinephrine, and Adenosine do not increse RBF

Water diarrhea + ulcerative colitis leads to loss of K and Na. Weakness. Seminormal Na lower K. EC shrunk 25%. A. % Exchangable Na Lost? A. Can you tell what fraction of exchangeable Na+ was lost? B. Can you tell what fraction of body K+ was lost? C. Administer Na+, K+, and alkali. What factors might account for the [K+] not rising?

A. Since exchangable Na is in ECM, same as fluid volume lost (25%) B. Since K in intracellular and acidemia shifts K into ECF, hard to tell how must is lost. C. K doesnt rise right away because it gets translocated to cells and also due to partial correction of acidemia it goes back into ICM.

9. In an experimental study it was found that a drug increased the urine/plasma inulin concentration ratio from 99 to 100 and the urine flow rate from 1.0 to 1.2 ml/min. Which of the following statements is correct? A. The drug has increased the GFR. B. The drug has decreased the GFR. C. More information is required to determine the effect on the GFR. D. The drug has increased renal plasma flow. E. Both A and D are correct.

A. The drug has increased the GFR.

Experiment for changes in ECF Na content vs concentration. McCHange examined the physiologic responses to depletion of Na+ without loss of H2O (usually lost together) via salt-fre diet + sweat box (high Na concentration): weight loss = water lost. DIstilled water to original volume but salt not replaced. If body Na depeleted but water volume same, Na concentration falls. Confusing finding: PNa did not change for 5 days. A. Why? B. 5 Days then Na, PNa, and Posm fell finally; why?

A. pNa (140mEq/L) + HCO3/l anions make up 95% of pOSM (290 mOsm/Kg) so a change in pNa causes change in pOSM. pNa was low right after diluting + sweating and so pOSM should be low. Reduction of 1-2% of pOSM signals hypothalamic osmoreceptors to turn off ADH so the kidney can pee out lots of free water until pNa and pOSM are normal and ADH goes back to normal. pNa was stable because they were excreting the diluted water they drank daily. It shows that a decrease in ECF Na leads to ECF water loss and reduced ECF volume (and high ECF Na leads to water retention and high ECF volume). Na is a skeleton for the ECF volume. As long as Na loss are not critical, they thus cannot reduce Na concentration. B. Once weight stabilized in 5 days, it meant the water was not retained. This is bc a 10% or more reduction in ECF volume causes ADH secretion even though low pOSM would turn off ADH secretion. So severe ECF volume education overrides control of ADH so "no pee hormone" activates. Once 10% was lost, psyc mech released ADH and so now concentration of pNa and pOSM began to decrease . For example, when Na and water loss in diarrhea, vomiting, diuretics is severe; pNA is reduced and they are hyponatremic.All retentive Na mechanisms are also active, urinary Na is negligible. Potassium excretion can be maintained despite fall in GFR and tubular flow due to increased aldosterone secretion.

30. Na+ reabsorption in the proximal tubule would increase if: filtered load of glu increased dopamine decreased increase in symp tone PTCP decreased increase in GFR A. the filtered load of glucose increased. B. local dopamine levels increased. C. there were a decrease in sympathetic tone. D. peritubular capillary pressure increased. E. there were a decrease in GFR.

A. the filtered load of glucose increased.

ACE Affects than lower BP: Diation of systemic vessels, Aldosterone inhibition so more Na/H2O excretion, inhibition of Na reabsorption in PCT

ACE Affects that raise BP: Dilation of efferent kidney arterial

58. Antidiuretic hormone (ADH) induces all of the following EXCEPT: A. insertion of aquaporins into proximal tubule lumenal membranes B. incorporation of aquaporins into collecting duct lumenal membranes C. enhanced urea permeability of inner medullary collecting ducts D. stimulation of thick ascending limb salt reabsorption E. depression of vasa recta blood flow

ADH increases all acept PCt aquapore. Aqu CD, urea perm in inner medulary cd, thick asc limb salt reabsorp, depression of vasa recta blood flow.

Renin (JGA) -> Angiotensin (liver, pulmonary capillaries) -> ADH (pituitary)

ADH makes the collecting ducts permeable to water Osmotic gradient in the medulla interstitium pulls water out of the filtrate

Long Term Adaptations to restore electrolytes and water Restoration of Electrolytes and Water

AII and adrenergic stimulation promote reabsorption of Na followed by water in the PCT. AII and adrenergic increase aldosterone secretion via adrenal cortex. Aldosterone increase reabsorption of Na in the distal tubules and collecting ducts. If Na excretion is less than intake, Na content increases and causes water retention in ECV to restore PV. AII has a direct effect on neurons in the thirst center of the brain thus causing increase H2O intake.

Non-hormonal Factors Altering K Homeostasis

Acid-base balance Acidosis = hyperkalemia Alkolosis = hypokalemia Exercise : K released from muscles Hyperosmolality -> Cell shrinkage -> K efflux -> Hyperkalemia Hypoosmolality -> Cell swelling -> K influx -> Hypokalemia Cell lysis (e.g., burns, rhabdomyolysis) hyperkalemia

How does the kidney absorb filtered HCO3? How does in control the intake and production of acid/alkali?

Acids and alkali are only present from the diet in small amounts. Most carbonic acid (20k mmol) of CO2 is due to metabolism, it is in equilibrium with H2CO3 so it is a volatile acid. Respiratory excretion of CO2 (thus H2CO3) allows this acid load to be removed so CO2 does not add to the acid load of the body. Metabolism of meat makes 70mmole of nonvolatile fixed acid (net endogenous acid production) which must be compensated by renal mechanism due to limitations of buffer and pulmonary mechanisms. H+ in fixed acids reacts with acids and HCO3+ to makes fixed buffer, H2O, and CO2. Excretion of H+ from a fixed acid through respiration requires equal amounts of HCO3- buffer ion. Respiratory compensation of fixed acid also caused loss of base so its cannot correct acid-based disorders (we need to excrete H without HCO3).

13. Which of the following will increase glomerular filtration rate? A. Thickening of the glomerular capillaries' wall. B. Increased local secretion of adenosine. C. Expansion of the extracellular fluid. D. Ureteral blockade by kidney stones. E. Administration of a converting enzyme inhibitor.

Adenosine lowers gfr C. Expansion of the extracellular fluid. ACE inhibitor will lower GFR Kidney stones lower zgfr Thickening lowers GFR

Renal Vasculature -Glomerular capillary tuft

All blood in kidney passes renal corpuscle prior to leaving. Blood returns to body after 1st cap takes stuff out and 2nd cap puts stuff back in.

Other Factors Regulating GFR in Response to Changes in ECF Volume

Angiotensin II: produced when renin secretion is stimulated by low arteriolar pressure and increased sympathetic activity. • Atrial natriuretic peptide: released by atrial myocytes in response to dilation of the atria. Vasodilator; increases RBF and GFR • Nitric oxide (NO): produced locally by vascular endothelium in response to increased ECF volume. It dilates the renal vasculature.

A 44yo man with low serum Na CONCENTRATION 116 mEq/L What is true?. A. He has a low total exchangeable Na+. (1) B. He has a high total exchangeable Na+. C. His total exchangeable Na+ is normal. D. He has ingested a lot of water. E. One cannot say anything definitive about the state of his electrolyte and water balance without added information.

Answer: E Na CONCENTRATION can be assoc. w/ low, normal, or high Na but a Na BODY CONTENT can accurately predict it.

Distribution and Balance

Approximately 40% of total body Na+ is in bone, but it is not available for ordinary metabolic processes. Almost 90% of the available Na+ is in the ECF (see Fig. 1). Since Na+, together with the accompanying anions Cl- and HCO3 account for most of the extracellular osmolality, the amount of body Na+ is an important determinant of ECF volume, as discussed below. Although the typical sodium intake is about 100 mEq/day, it varies from < 1mEq/day, for persons on a low salt diet, to more than 400 mEq/day for individuals who like salty foods. There is not clear evidence for physiological mechanisms that regulate intake in humans. On the other hand the major excretory pathway for Na+, renal excretion, is tightly regulated to match the amount of Na+ ingested in the diet. Under normal conditions the Na+ stool and sweat losses are negligible (< 10 mEq/day). (Note: some texts use grams of Na+ or NaCl instead of Equivalents. 1 Eq Na+ = 23 g Na+ = 58.4 g NaCl).

Transcapillary & transcellular fluid movement - short term adaptation

Arterioles constrict causing the capillary hydrostatic pressure to reduce so there is less outward filtration of fluid and thus more BV retained in the capilaries post 30 min (also we some reabsorption of fluid from interstitium). Epinephrine increases glucose release to increase ECF osmolarity up 20mosm/L so water moves from ICF to ECF as well.

Renin Secretion

Arterioles: 1A Tubules: 1B

Introduction Osmoregulation

As already discussed, approximately 99% of the filtered water is reabsorbed by the renal tubules. Under normal euvolemic conditions about 2/3 is reabsorbed isosmotically at the proximal tubule, by a mechanism secondary to the reabsorption of salt (Standing Osmotic Gradient Hypothesis). The descending limb of Henle's loop is also highly permeable to water. The driving force for water reabsorption at this segment, and at the collecting duct, is the high osmolality of the medullary interstitium (see later). In contrast, the ascending limb and distal tubule are impermeable to water, and no water reabsorption takes place at these segments. The permeability to water of the collecting ducts is regulated by antidiuretic hormone (ADH), a peptide secreted by the posterior pituitary (neurohypophysis). High level of plasma ADH increases the water permeability of the collecting duct promoting water reabsorption and the excretion of a small volume of concentrated urine. On the other hand, with low or zero plasma ADH, the permeability to water of collecting duct is near zero, and there is no reabsorption of water. This leads to the excretion of a large volume of dilute urine.

Absorption and secretion

As already mentioned, in absorbing epithelia the primary transported ion is normally Na+, with anions and water following it. Transcellular movement of Na+ renders the lumen more negative than the blood side1 and this electrical potential drives the paracellular movement of anions from the lumen to the interstitial space, as shown in Fig 3, left. In secretory epithelia, such as the salivary glands or the pancreatic duct cells, the secretion of Cl- is the primary event dragging along Na+ and water (Fig. 3). At the basolateral membrane, a Na+-K+-2Cl- cotransport driven by the electrochemical Na+ gradient accumulates Cl- ions in the cell above its electrochemical potential. At the apical domain a Cl- channel allows secretion of Cl- into the lumen. As the secretion of Cl- increases the luminal potential becomes more negative with respect to the blood side, driving the paracellular secretion of Na+. Note that in both cases, secretion and absorption, the transcellular transport processes set up the electrical gradients that drive the paracellular transport.

Acid Base Disorders

Aspirin or other salicylates posioning causes mixed: resp alkalosis due to high acidity in resp centers + metabolic acidosis due to metabolic digestions

Mean % change in arterial BP and ADH in response to blood loss in dogs

At 25mg, BP falls and ADH rises

Renal Hemodynamic Control Mechanisms

Autoregulation via tubuloglomerular feedback maintains gFR and RBF during changes in renal arterial pressures. The neural and adrenergic systems have a rapid response to changes in systemic blood pressure. The renin-angiotensin system if activated will reduce RBF and GFR thus increasing the sensitivity of TGF to respond to body sodium via changes in hemodynamic function and sodium reabsorption. NO & prostaglandins determine the resting vascular tone via NO and counteract AII by preventing too much vasoconstriction (PG)

B. Explain how this patient develops hyponatremia.

B) Small cell (aka oat cell) carcinoma of the lung may secrete ADH. Unlike ADH secretion from the posterior pituitary, ectopic hormone secretion from the cancer cell is not feedbackregulated. As a result, blood levels of ADH can become extraordinarily high increasing water reabsorption and resulting in water retention, dilution of body fluids and reduced plasma Na+ concentration

B. What would be the free water clearances if the daily urine volumes are 2L, 4L, or 1L?

B) first scenario: a urine volume output of 2 L/day. In this case the free water clearance is zero: CH2O = V - COSM = 2- 2= 0 and the urine osmolality is 600 mosmoles/2 L =300 mOsm, same as that of plasma. second scenario: a urine volume output of 4 L/day. The free water clearance is 4 - 2 = 2L/day. Urine osmolality is now 600/4= 150 mOsm. The daily urine volume (4 L at 150 mOsm) can be thought of two virtual volumes: 2L at the same osmolality (300 mOsm) as that of plasma (the osmolar clearance) plus 2 L of pure water (the free water clearance). third scenario: a urine volume output of 1 L/day. The free water clearance is 1 - 2= -1 L. The negative value indicates a volume of water that is separated from solutes by the kidneys and returned to the circulation (rather than excreted). It also represents a volume of water that needs to be added to the urine (urine osmolality is now 600/1 = 600 mOsm) to make it isosmotic to plasma. The free-water clearance provides a way to quantify the ability of the kidneys to generate solutefree water (a volume of water that is free of all solutes). When ADH levels are low and the kidneys excrete a large volume of hypotonic (dilute) urine, this solute-free water is excreted from the body (CH2O positive). On the other hand, with high plasma ADH the kidneys reabsorb water avidly and they excrete a small volume of urine more concentrated than plasma, i.e., this solutefree water is returned to the systemic circulation and CH2O is negative.

45. Which of the following is FALSE? A. Angiotensin II in low concentration constricts efferent arterioles more than afferent arterioles. C. Depression of NaKCl2 transport out of the lumen of the thick ascending limb of the loop of Henle would depress both maximum diluting ability and maximum concentrating ability. D. Medullary interstitial osmolality is greater in the presence of ADH. E. Profuse sweating, without replacement of salt or water, will result in an increase in extracellular fluid osmolality

B. During excretion of a concentrated urine, lumenal fluid entering the distal tubule is isosmotic.

46. Which of the following statements is FALSE? A. Normal plasma H+ activity is about 40 nEq/L. B. Raising the pH of a solution of a weak acid in water will decrease the dissociation of the acid. C. In the range of near-normal pH an increase of pH by .01 unit corresponds to a decrease of H+ activity of 1 nanoequivalent/liter D. A H+ activity of 10 -5.2 equivalents/L corresponds to a pH of 5.2. E. Hyperventilation has no effect on the pK of the bicarbonate-carbonic acid system.

B. Raising the pH of a solution of a weak acid in water will decrease the dissociation of the acid.

53. In antidiuresis, all of the following occurs EXCEPT: A. The medullary interstitial osmolality is at its maximal value. B. The late collecting duct urea permeability is very low. C. Medullary blood flow decreases. D. The urine is hypertonic. E. The late distal tubule and collecting duct water permeability is high.

B. The late collecting duct urea permeability is very low.

29. Which of the following statements regarding the vasa recta is correct? Volume entering decensing is smaller than leaving ascneding Som of descending is lower than ascending Flow is decreased by ADH Abs salt and loose water in descending into inner medulla Medulalry interstitial osmolarity decreases with increase vasa recta flow rate A. The volume of fluid entering the descending limb is larger than that leaving the ascending limb. B. The osmolarity of fluid entering the descending limb is lower than that leaving the ascending limb. C. The flow is increased by antidiuretic hormone. D. They absorb water and lose salt in their descent to the inner medulla. E. The medullary interstitial osmolarity increases with increasing vasa recta flow rates.

B. The osmolarity of fluid entering the descending limb is lower than that leaving the ascending limb.

18. In chronic respiratory acidosis: A. The urine is alkaline B. The plasma bicarbonate concentration is elevated C. Respiratory compensation lowers plasma PCO2 D. Plasma K+ concentration is low E. The kidneys only partially reabsorb the filtered bicarbonate

B. The plasma bicarbonate concentration is elevated

59. A loss of almost 30% of an individual's blood volume might lead to all of the following EXCEPT: A. a decreased free water clearance B. a decreased PaO2 C. increased movement of interstitial fluid into plasma D. an increase in ECF osmolarity due to elevated glucose E. lactic acidosis

B. a decreased PaO2 Pa O2 should remain the same.

41. If the glomerular filtration rate remained unchanged, a state of antidiuresis would be associated with: A. a lower osmolarity of lumenal fluid in the proximal tubule B. a greater fraction of water reabsorption in the thin descending loop of Henle C. a greater osmolarity of lumenal fluid in at the end of the thick ascending loop of Henle D. a lower concentration of urea in the medullary interstitium E. a larger urine volume

B. a greater fraction of water reabsorption in the thin descending loop of Henle

38. Toward the end of World War II, Karl Beyer and his associates noted that the injection of PAH decreased the excretion of penicillin in the urine. What would you suggest was its mechanism of action? The PAH: A. prevents the reabsorption of penicillin. B. competes with penicillin for a carrier in one of the secretory mechanisms. C. increases the renal blood flow and hence the filtration of penicillin. D. competes with penicillin for a carrier in one of the reabsorptive mechanisms. E. promotes diuresis.

B. competes with penicillin for a carrier in one of the secretory mechanisms.

27. Prolonged lactic acidosis would: A. result in a compensatory increase in PCO2. C. increase the pH of urine. D. decrease the plasma K+ concentration. E. decrease the NH4 + concentration in urine.

B. decrease the daily filtered load of HCO3

55. Potassium excretion by the kidney would be increased by all of the following EXCEPT: K excreted more due to more Na, more flow, alkalosis, aldosterone, more K in diet A. increased delivery of Na+ to the distal tubule. B. decreased distal tubular flow. C. alkalosis. D. aldosterone. E. increased dietary intake of K+ .

B. decreased distal tubular flow.

3. The tubular portion of the nephron: in the medulla. The loop of Henle is the part of the nephron that extends into the medulla.

Be aware that the relative length of these loops differs: for the nephrons whose corpuscle is located in the outer cortex, the loop of Henle is relatively short and does not penetrate deeply into the medulla; for the nephrons whose corpuscle is located nearer the medulla, the loop on Henle descends to the medullary papilla. This arrangement helps explain the pyramidal shape of the medullary pyramid: it would not be pyramidal if all nephrons extended to the apex. The loop of Henle has three morphologically distinct portions, which are, in order: the descending thick portion, the thin limb, which includes the U-turn, and the ascending thick portion. The morphology of the descending thick and ascending thick portions have been described above.

Defects in Kidney Formation in the Embryo

Because a significant portion of kidney cortex is derived from differentiated local mesenchymal tissue, the lobes of the kidney, which are functionally distinct, Renal - 8 - 2 tend to grow anatomically together. You have previously learned to identify "cortical columns" which represent this anatomical union of two adjacent lobes. The paired kidneys migrate cranially during fetal life relative to their surrounding organs and vasculature (which themselves are also continuing to expand caudally). During this migration, the two kidneys must squeeze medially together in order to pass cranially through the fork formed by the branched common iliac arteries. Occasionally the kidneys become fused during this passage, usually at their caudal and medial aspect, forming a horseshoe kidney. A horseshoe kidney, connected across the midline, will fail to ascend past the inferior mesenteric artery, which travels medially and ventrally from the abdominal aorta, and will thus remain abnormally caudal, at about L3-L4. Kidneys which fail to ascend for other reasons remain within the pelvic girdle and are termed pelvic kidneys. Microscopically, the development of the various discrete regions of a functional nephron (proximal tubule, thin tubule, distal tubule, etc.) is heavily dependent on continued normal functioning of the nephron as a whole. Because the nephron is an enclosed epithelial space surrounded by a basement membrane, nephrons (and collecting ducts) that fail to connect or function normally are prone to degenerate into fluid-filled cysts. Thus, many molecular and genetic disorders of kidney development get classed together as polycystic kidney disease. Wilms' tumor, or nephroblastoma, is an important childhood cancer (greatest incidence at ~3 years of age) caused by genetic mutations in nephric precursor cells. Microscopically, Wilms' tumor tissue contains stromal and epithetlial elements that resemble the structures of the developing kidney. Students should appreciate that historically, the study of Wilms' tumor together with retinoblastoma led directly to the discovery of the class of genes known as tumor suppressor genes. Today, Wilms' tumor is treated with a combination of surgery, radiation and chemotherapy, and patients generally have a good prognosis.

Replenisihing depleted HCO3 stores

Bicarb buffering for fixed acids can deplete extracellular bicarb levels. Kidneys can restore bicarb by excreting H+. H+ excreted as free acid is minimal since the minimal urinary pH of 4.4 is only a H+ concentration 1000x more concentration than plasma. At this level, daily removal of 70mEq fixed acid load with free unbuffered acid would need 1,750 liters of urine. H+ that is secreted into urine reacts with urinary buffers so H+ in required amounts can by excreted in absense of very low pH or very high urine volume. The main buffers are phosphate (in lumenal fluid) and ammonia (from aa glutamine in proximal tubule cells). Buffering H+ by phosphate: HPO4 (HPO42/H2PO4=4) but when max acidification in tubular fluid occurs H2PO4 is prefered (HPO4/H2PO4 = .01). Phosphate adds to the TA of urine (TA is total contribution of buffers via back-titration with alkali to the pH of the glomerular filtrate which is 7.40) Titratable acid accounts for 1/3 of acid excretion in the urine. Increase in TA excretion in acidosis is minimal except for endogenous disease where buffer production is too high (diabetes ketoacids)

Maintenance of interstitial osmolality profile

Blood flow through the medulla tends to dissipate the interstitial osmolality gradients. However, the tendency for dissipation of osmotic stratification by diffusion and/or bulk flow in efferent blood vessels is counteracted by passive countercurrent exchange of solutes and water between the closely apposed descending and ascending vasa recta (Fig. 8). Recall that the vasa recta form straight loops reaching the inner medulla. Because of the high permeability of these capillaries to water and small solutes, the plasma in the descending vasa recta gets concentrated as it flows into the medulla. However, while flowing through the ascending portion, where solute concentration in the interstitium decreases progressively at higher levels in the medulla, the plasma is diluted as solutes leave and water enters the vasa recta. This passive countercurrent exchange thereby minimizes, although does not abolish, solute washout.

Cells of the glomerulus 3 cells: 1. Endo cell of fenestrated caps 2.Podocytes of Bowman's capsule 3. Mesangial cells in area between these two layers (resident contractile phagocytic pericytes)

Bowman outer simple squamous parietal layer + inner visceral alyer. Podocytes of visceral layer= shape w/ processes for fine linear pedicels aline w/ abluminal surface of the glomerular capillaries.

Body Responses to Acid Load

Buffering ECF has 43% bicarbonate buffers (sec/min) and ICF is 57% proteins/phosphates buffer (hrs) Respiratory compensation due to carotid bodies, chemoreceptors ventilation. H + HCO3 H2CO3 H2O + CO2 Renal response is HCO3 reabsorption and formation of new HCO3 as well as secretion of H. NAE = Uta + Unh4 - Uhco3 * Uflow

Besides preserving the integrity of the epithelium, the tight junctions form a boundary that separates the luminal and basolateral domains of the cell membrane, preventing the diffusion of integral proteins from one domain to the other.

By targeting different transporters to either membrane domain the epithelium is capable of carrying out absorption (net transport from the lumen to the blood side) or secretion (net transport in the opposite direction) of solutes and water. The cell polarity confers epithelia the important function of vectorial transport. Fig. 2A shows how an epithelium accomplishes Na+ absorption, i.e., the net transport of Na+ from the lumen to the interstitial/capillary space, by locating the Na+ pumps exclusively at the basolateral membrane and Na+ channels at the luminal membrane. As we will see later, Na+ absorption is accompanied by Cl- , the most abundant counterion, and by water. This is the usual arrangement in absorptive epithelia: Na+ absorption is the primary mechanism that drives the movement of salt and fluid. Since the apical and basolateral domains have different complement of ion channels they exhibit different ion permeabilities and, consequently, different membrane voltages. This creates a difference in voltage across the epithelium, between the lumen and the interstitial side (see Fig. 2B). The magnitude and polarity of the transepithelial voltage are determined by the specific transporters in the two membrane domains, as well as by the permeability characteristics of the tight junction.

C. What would you anticipate would be the effects of furosemide (a loop diuretic) on the: • interstitial osmolality • luminal osmolality • maximal concentrating ability • maximal diluting ability • osmolar clearance

C) Inhibition of solute reabsorption at the TAL will result in decreased interstitial osmolality. In parallel, the luminal fluid osmolality at the top of the TAL will increase. These effects will result in the inhibition of both, the maximal diluting and concentrating abilities. The osmolar clearance will increase as a consequence of the inhibition of solute reabsorption. Although diuretics cause an increase in urine output, it is important to distinguish this diuresis from that which occurs following the ingestion of large volumes of water. In the latter case the urine is primarily comprised of water, and solute excretion is not increased. In contrast, diuretics result in the enhanced excretion of both solute and water.

C. Although water handling is impaired in SIADH, the volume regulatory pathways (such as the RAA system) are intact. What effect will the initial water retention have on volume regulation and urinary sodium excretion?

C) The addition of water to the body does not usually cause a permanent increase in ECF volume (and blood pressure). First, because most of the water retained (about 2/3) goes into the ICF. Second, the initial ECF expansion decreases renin release and increases that of ANP. These changes will lead to enhanced Na+ and water excretion that will tend to return the ECF volume toward normal. Thus, part of the initial hyponatremia seen in SIADH is due to a decrease in total body Na+ .

42. Normal plasma values of various acid-base parameters are as follows: pH = 7.40 [H+] = 40 x 10-9 Eq/L [HCO3] = 24 mEq/L PCO2 = 40 mm Hg Which of the following statements is correct? A. Metabolic acidosis is characterized by a high PCO2 and a high HCO3B. If plasma pH = 7.43, plasma [H+ ] = 43 x 10-9 Eq/L. C. A patient with plasma [H+] = 33 x 10-9 Eq/L, [HCO3 = 14 mEq/L, PCO2 = 20 mm Hg is suffering from respiratory alkalosis. D. If plasma [HCO3 is normal, plasma pH is directly proportional to plasma PCO2. E. A patient with plasma pH = 7.40, [HCO ] = 15 m Eq/L, PCO2 = 25 mm Hg is suffering from mixed metabolic alkalosis plus respiratory acidosis.

C. A patient with plasma [H+] = 33 x 10-9 Eq/L, [HCO3 = 14 mEq/L, PCO2 = 20 mm Hg is suffering from respiratory alkalosis.

56. Which of the following statements about sodium handling by the kidneys is FALSE? Na = + 20M/day, PCT reasb increase Na reabsorp of frac filt if ECF is volume depelered, ANP causes meducllary abs or Na, Na reabsorp uses 90% of O2, diuretics increase renal clearance of Na A. Sodium reabsorption by the renal tubules amounts to more than 20 Moles/day. B. Proximal tubular sodium reabsorption, as a fraction of the filtered load, is increased following ECF volume depletion. C. In the presence of ANP the medullary-collecting duct secretes sodium. D. Sodium reabsorption utilizes more than 90% of the O2 consumed by the kidneys. E. Most diuretics will increase the renal clearance of sodium.

C. In the presence of ANP the medullary-collecting duct secretes sodium.

17. The main barrier preventing filtration of proteins at the glomeruli is formed by: A. Fenestrations in endothelial cells. B. Mesangial cells. C. Negatively charged basement membrane and filtration slits. D. The negative electrical potential in Bowman's space. E. The parietal layer of Bowman's capsule.

C. Negatively charged basement membrane and filtration slits.

15. A few days after doubling the usual salt intake of a person: A. The kidneys will excrete enough sodium to maintain the original total body sodium content. B. The plasma sodium concentration will be significantly higher than normal. C. The kidneys will excrete an amount of sodium equal to the amount ingested. D. The volume of the intracellular fluid increases. E. Serum creatinine concentration increases.

C. The kidneys will excrete an amount of sodium equal to the amount ingested.

24. An increase in aldosterone production occurs in response to: A. ingestion of sodium chloride. B. an increase in blood volume. C. an increased intake of potassium. D. atrial natriuretic peptide. E. administration of an angiotensin converting enzyme (ACE) inhibitor.

C. an increased intake of potassium.

The ECV depends on the ECF, PV and CO, but it can dissociate from these parameters

CHF: Low CO causes ECV drop Water Immersion: High CO without impacting ECF causes ECV rise Hypovolemia:CO, ECF, PV low leads to ECV low

For any substance X we can calculate its clearance, CX, as the excretion rate of X divided by its plasma concentration:

CX = UX ∙ V̇/ PX [5] As indicated above, it has the units of a volume per unit time and it represents the volume of plasma that is cleared of X by the kidneys per unit time. Note that the clearance is a calculated, not an actual volume/time, since most substances are only partially removed in a single pass through the kidneys.

Circulatory shock symptoms

Circulatory shock: acute inability to perfuse tissues in body -Pale/cold/clammy skin due to vasoconstriction + active sweat glands -thready pulse due to tachycardia and low stroke volume -MAP normal or reduced but consistently low pulse pressure -Rapid/shallow breathing -Reduced urine output (oliguria)

60 year-old man with alcoholic cirrhosis presents with increasing abdominal distention. Normal temp, high pulse, normal BP, high RR, normal O2. Jaundiced/malnourished.Liver shrunken and abdomen distenced. 3+ pitting edema. Low Na, BUN, Cr.

Cirrhosis is scarring of the liver. Portal vein nutrient rich blood from GI/spleen to liver for processing. Scarring around portal vein constricts it leading to portal hypertension. Back up of liver blood can lead to hypovolemia and lower cardiac output. Low effective blood volume (EBV) and cardiac output (CO) activates renin-angiotensin system + secretion of ADH. RAII + ADH cause vasoconstriction so even lower perfusion. Chronically lower renal blood flow also causes renal failure. Portal hypertension can cause hyponatremia due to sodium stores being less secreted. Activating RAA, Sym nervous system, and ADH causes decrease of ECV. Lower ECV cause sodium and water to be retained, but more water than NA. Thus, ADH secretion rises leading to more water retention and hyponatremia.

Roles of Potassium are..

Cofactor for enzymes: regulate muscle blood flow, cell growth, maintain cell volume, nerve/muscle excitment Vm ≅ 61 log Kout/Kin Changes in K-out -> paralysis and arrhythmias

Cells of the collecting duct

Collecting duct has principal cells (ion exchange, basal infolding, clear cytoplasm, ADH responsive, activate aquaporin transporters) and intercalated cells (acid/base balance, dark cytoplasm, mito/golgi/polyrib, H+ transporters, HCO3 transporters

The tubular portion of the nephron: in the medulla. Loop of henle is part of nephron reaching medulla makes concentration gradient in the medulla of the kidney with 3 different parts: the descending thick portion, the thin limb,, and the ascending thick portion

Compare epithelial lining of the the descending thick and the ascending thick tubules can be compared. This field is in the outer medulla, near the cortex, where both ascending and descending thick limbs of the loop of Henle occur. A collecting duct also traverses this field. Two cell populations are apparent in the collecting duct: the more numerous paler-staining principal cells and the interspersed intercalated cells that have darker cytoplasm. VR here.

Fig. 4 Events following hemorrhage

Compensatory responses increase reduced blood volume and cardiac output but do not restore them. Arterial pressure is maintained. *Write out map*

The clearance of inulin measures the GFR

Consider a substance that is freely filtered, is neither reabsorbed nor secreted, and is not metabolized by the kidneys. Inulin, an exogenous 5.2 kDa plant polysaccharide is such a substance. All inulin molecules filtered by the glomerulus are excreted in the urine. That is: FINU = EINU, and from Eqs. 3 and 4: PINU·GFR = UINU· 𝑉𝑉̇ and solving for GFR: GFR = (Uinu*V)/Pinu = Cinu according to the definition of clearance (Eq. 5). That is, the clearance of a substance that is removed from the blood only by filtration, and that it is not reabsorbed, is equal to the GFR, since this is exactly the volume of plasma that is cleared of the substance (per unit time). Note that Eq. 6 provides a method to determine GFR, since the variables in this equation can be measured once a steady state is reached during infusion of inulin.

In cortex: See glomeruli: DCT confined here but no loops of henle In medulla: not distal convoluted tubules, loops of hene confined here Must identify: proximal convoluted tubule (cortex) thick descending tubule (medulla) thin limb of Henle's loop (medulla) thick ascending tubule (medulla) distal tubule (cortex) collecting duct (both?)

Cortex: 1. Glomeruli 2. PCT CORTEX ONLY - short/wide columnar cell, large nuclei, eosinophilic cytoplasm, brush border microvilli, vesicles vacuoles, striations of basal infolding for surface area filtrate. (descending thick tubules in medulla look like PCT) 3. Distal tubule/ascending thick limb: lack brush bord and vacules, low cuboidal nucleus, wider lumen. 4. CT

5. Normal values of plasma acid-base parameters are approximately: pH= 7.40; [HCO3 - ] = 24 mEq/L; PCO2 = 40 mmHg. Which of the following sets of values indicates respiratory alkalosis in a patient with normal renal function? pH [HCO3 - ] PCO2 (mEq/L) (mm Hg) A. 7.49 35 48 B. 7.34 15 29 C. 7.34 31 60 D. 7.47 14 20 E. 7.09 15 60

D. 7.47 14 20

22. In an individual with a plasma and urine osmolality of 300 mOsm/L, if the urine flow rate is 33 ml/min the osmolar clearance (Cosm) and the free-water clearance (CH2O) will be: A. Cosm = 33 ml/min, CH20 >0 B. Cosm >33 ml/min, CH20 =0 C. Cosm <33 ml/min, CH20 <0 D. Cosm =33 ml/min, CH20 =0 E. Cosm = 33 ml/min, CH20 <0

D. Cosm =33 ml/min, CH20 =0

50. Consider the exchange of solute and water in the vasa recta. Which of the following statements is FALSE? A. Plasma solute concentration at the end of the vasa recta is greater than that at the beginning of the vasa recta. B. Reduction of vasa recta blood flow decreases washout of interstitial solute. C. Water moves from the descending vasa recta to the ascending vasa recta. D. Vasa recta blood flow exceeds cortical blood flow E. Blood flow leaving the vasa recta is greater than that entering the vasa recta..

D. Vasa recta blood flow exceeds cortical blood flow

28. A rise in plasma creatinine concentration might result from: decrease resistance of efferent arteriole decrease in PV decrese in creatinine clearance decrease in Kf decrease in Pgc A. an increase in the resistance of the efferent arteriole. B. an increase in plasma volume. C. an increase in creatinine clearance. D. a decrease in the glomerular filtration coefficient (Kf). E. an increase in the glomerular capillary pressure.

D. a decrease in the glomerular filtration coefficient (Kf).

32. An acute increase in the NaCl delivery to the macula densa promotes: A. constriction of the afferent arteriole and dilation of the efferent arteriole. B. constriction of both the afferent and efferent arterioles. C. constriction of only the efferent arteriole. D. constriction of only the afferent arteriole. E. dilation of the afferent arteriole and constriction of the efferent arteriole

D. constriction of only the afferent arteriole.

44. All of the following are involved in sodium reabsorption by the proximal tubules EXCEPT: A. sodium-3 HCO3 - cotransporter B. sodium-glucosecotransporter C. sodium-hydrogen exchanger E. sodium-amino acid cotransporter

D. paracellular diffusion

The thin limb of Henle's loop is ONLY IN THE MEDULLA. It is squamous and bulging nuclei, looks like capillaries but some thicker call and closer nuclei.

Deeper medulla, only the thin limb of the loop of Henle and the ascending thick limb of the loop extend. Capillaires are vasa recta see by blood cells Thin limbs TL smaller diameter than vasa recta. CD have tallest epithelia in the medulla.

Problem 2 Narrowing of the renal artery (renal artery stenosis) is associated with a reduction in arterial pressure distal to the obstruction. Despite the fall in the perfusing pressure of the glomeruli, the GFR can initially be maintained. A. Explain a possible mechanism for the maintenance of the GFR in this setting.

Despite the fall in pressure perfusing the glomeruli, the GFR can initially be maintained by autoregulation: myogenic response by the afferent arteriole, and the production of angiotensin II that, by constricting preferentially the efferent arteriole prevents the glomerular capillary pressure from declining with the occurrence of hypotension.

Case: 56y/o women thirst + urination 3months no hist/meds. Normal temp, pulse, higher bp (160/90), normal rr, normal O2. High BMI, edema, sensation lower, high glucose 401 mg/dl -> diabetes mellitus. ACE inhibitors: Lower BP + protect kidney via lisinopril.

Differentialas: ADH deficiency - DI neurogenic or nephrogenic Osmotic diuresis: Glucose in urine in D.M via osmotic gradient. Pregnancy -> pressure on bladder. Structural or funcitonal reabsorption kidney ability. Loop diuretic (inhibit Na-K-Cl in ascending limb loop of Henle so renal medulla less concentrated so less water reabsorption and more pee). Caffeine (block PCT adenosine, water/sodium excretion, inhibits ADH).

What is the relationship between flow rate and K secretion at the distal nephron? Area in graph between vertical lines is the flow rate under physiological conditions.

Distal flow effects K secretion shows why aldosterone does not distrub K balance when the hormone is secreted to correct ECF volume. When ECF volume is decreased, the renin-angiotensin-aldosterone system stimulates aldosterone to increase distal reabsorption of Na and thus secretion of K (if thats it, we would get hypokalemai). THis does not occur due to decreased ECF volume on distal flow. Decreased ECF volume -> plasma AII is elevated and causes water reabsorption that slows distal flow. Lower distal flow prevents K wash out so gradient fo K secretion is lower. If aldosterone is chronically admiistered it will still lead to hypokalemai.

Sodium Regulation (or how we maintain ECF volume and BP)

Distribution and Balance • Response to Changes in Body Sodium. Receptors • Regulation of Filtered Sodium • Regulation of Reabsorption - Proximal Tubule - Thick Ascending Limb - Distal Segments • The Renin-Angiotensin-Aldosterone System (RAAS) • Natriuretic and Local Factors • Comparison of Volume Regulation and Osmoregulation

Papillary Ducts (Ducts of Bellini)

Ducts of Bellini is the collecting ducts that open into the minor calyx passing through the cribrosa.

Concluding remarks

During euvolemia (i.e., with a normal ECV), as long as variations in the dietary intake of NaCl are minor, control of Na+ excretion is achieved by altering the reabsorption at the collecting ducts, principally by aldosterone (through the renin-angiotensin-system) and ANP (Fig. 4). The appropriate renal responses result from subtle changes in levels of these hormones and require negligible neural and hormonal influences on proximal tubular reabsorption. Recall that during euvolemia several mechanisms (autoregulation, glomerulotubular balance, load dependency of Na+ reabsorption by Henle's loop) converge to deliver a constant fraction of the filtered Na+ to the distal nephron. On the other hand, when significant changes in NaCl intake/ECF volume occur, additional factors are called into play, involving GFR and proximal tubule and TAL reabsorption, which restore the balance between excretion and intake. Other less well-understood mechanisms participate also in the regulation of the ECF volume. This is illustrated by the phenomenon of aldosterone escape. Subjects given aldosterone, or patients with an aldosterone-secreting adrenal tumor, retain fluid for only a few days and then undergo a spontaneous diuresis that returns the volume (and arterial pressure) toward normal. Experiments in which a renal artery clamp is used to control renal pressure independently of systemic arterial pressure have shown that the rise in the renal perfusion pressure (rather than in systemic pressure) is the major factor triggering the natriuresis (termed pressure natriuresis). Several mechanisms are involved in this response that seems to function as a backup system that prevents inordinate rises in blood pressure when the humoral regulation of Na+ excretion is malfunctioning.

12. Which of the following statements is correct? A. Most K+ ingested following a meal is excreted promptly in the urine. B. Small elevations in plasma [K+] cause a prompt decrease in adrenal aldosterone secretion. C. A 1 mM increase in extracellular K+ concentration will have the same effect on membrane potential as a 1 mM decrease in intracellular K+ concentration. D. Vomiting of gastric contents will increase plasma [K+. E. Increases in plasma K+ concentration lead to an increased activation of membrane Na-K pumps.

E. Increases in plasma K+ concentration lead to an increased activation of membrane Na-K pumps.

2. Following marked blood loss one would expect to find: A. A decrease in sympathetic nervous system activity. B. A decrease in plasma angiotensin II concentration. C. A decrease in plasma renin concentration. D. An increase in renal vascular nitric oxide production. E. None of the above.

E. None of the above.

51. Which of the following statements is FALSE? A. Each afferent arteriole is associated with a single glomerulus. B. Several nephrons drain into a single collecting duct. C. Each kidney contains about one million nephrons. D. Most nephrons are cortical. E. Only juxtamedullary nephrons contain the juxtaglomerular apparatus

E. Only juxtamedullary nephrons contain the juxtaglomerular apparatus

14. If the filtered load of glucose exceeds its tubular transport maximum: A. The clearance of glucose is larger than the clearance of inulin. B. Glucose is secreted by the tubules. C. Glucose concentration in Bowman's space fluid is higher than that in plasma. D. The clearance of glucose is zero. E. The distal tubular fluid contains glucose

E. The distal tubular fluid contains glucose

40. Isosmotic water reabsorption at the proximal tubules implies: A. the osmolality of the tubular fluid remains constant along the proximal tubule. B. the concentration of sodium along the proximal tubule remains constant. C. solutes and water are reabsorbed at the same fractional rate by the proximal tubule. D. the fluid reabsorbed from the proximal tubule has the same osmolality as plasma. E. all of the above.

E. all of the above.

The kidneys are two organs weighting about 160 g/each that lie retroperitoneally in the back of the abdominal wall.

Each kidney contains about 1 million nephrons, which are the basic functional units. The nephron consists of the renal corpuscle and the tubule. The renal corpuscle contains the glomerulus, a tuft of very short capillaries, and a fluid filled capsule that surrounds it (Bowman's capsule). As blood from the afferent arteriole enters the glomerulus, a fraction of it is filtered into Bowman's space and forms a fluid free of cells and proteins (the ultrafiltrate). The remaining blood leaves the glomerulus through the efferent arteriole. The special arrangement of blood running from capillaries into efferent arterioles (instead of into a vein) is of great importance in regulating the pressure and flow through the glomeruli, as will be discussed later.

The Filtration Barrier Secretion of hormones (renin, erythropoeitin) and active vitamin D

Endothelium has 70 nm fenestrations Basement membrane has collagen and negatively charged glycoproteins Podocytes (visceral epithelial layer) have Interdigitating pedicels forming Filtration slits covered with diaphragms (nephrin, podocin) containing pores of variable sizes (4-14 nm) Contractions of mesangial cells and podocytes regulate filtration permeability

Properties of epithelia

Epithelia are layers of cells usually found lining the luminal surface of hollow organs (intestines, renal tubules, alveoli, etc.) The basal side of the epithelium rests on a basement membrane attached to the underlying connective tissue. The cells forming the epithelium are bound together by various types of adhering junctions. Gap junctions provide low-resistance electrical connections between cells, such that the epithelium behaves functionally as a syncytium. Tight junctions are formed by continuous rows of transmembrane proteins (claudins and others) at the intercellular spaces, near the luminal border. Attachment of claudins from adjoining cells form the junctions (see Fig. 1).

. The Ureterr

Examine a transverse section through the human ureter, slide 465. Observe the stellate pattern of the lumen, which is lined by transitional epithelium that is 4-5 cell layers in thickness. The lamina propria-submucosa is a loose collagenous and elastic layer. There is a very thin muscularis mucosae between the lamina propria and the submucosa. An outermost muscular tunic, the muscularis, consists of two or three loosely arranged and ill-defined layers of smooth muscle bundles separated by connective tissue. The inner layer is longitudinally arranged, the next layer circularly, and the third, outermost coat, is again longitudinally arranged. A third coat is present only in the distal portion of the ureter and is not present in this slide. Because the ureter is, like the kidney, a retroperitoneal organ, it has an adventitia and no serosa.

Effector Mechanisms NA

Excreted Na = Filtered Na - Reabsorbed Na PNa GFR ≅ 140 mEq/L 180 L/day = 25,200 mEq/day The kidney filters ECF Na more than 10 times per day • Only a small fraction of filtered Na is excreted to eliminate dietary intake • Large changes in excretion can result from small percentage changes in either filtered load or reabsorption Tubular reabsorption: • Proximal segments (PT & TAL) : Large capacity for reabsorption and ability to alter transport in response to load ('load-dependent") • Distal segments (DT & CD) : Limited capacity for reabsorption but able to 'fine tune' reabsorption to achieve balance (excretion=intake)

Functional Organization: three levels renal lobe: med pyramid + cortex + 1/2 renal column renal lobule: 1 med ray intralobular in position nephron: the smallest functional unit of the kidney.

Figure 4: An adult kidney has 8 to 18 cone-shaped lobes. At the tip of each pyramid urine flows into a minor calyx lined with transitional epithelium

Blood in kidney

Figure 7: Simplified diagram of the blood vessels in three lobes of the kidney. Naming is with respect to the lobes and lobules, and location. Differences in the second capillary bed associated with a subcapsular glomerulus PTCP (left lobe), a regular glomerulus (middle lobe), and a juxtamedullary glomerulus VR (right lobe) are illustrated.

HCO3 is reabsorbed mainly in the PCT (80%), ThickAL (10%), DCT (6%), CCD (4%) so there is near 0% secreted in IMCD.

Filtered load of HCO3: 4300 mEq/day Reabsorption at PCT, TAL, & DCT are similar but with different transporter isoforms Little change in tubular fluid pH in proximal nephron CD uses different transporters so pH can drop to as low as 4.4 by removing HCO3- base.

The concept of renal clearance is also useful to determine how the kidneys handle different substances.

For example, if a substance X is freely filtered and then partially reabsorbed, the amount of X excreted must be less than the filtered load, i.e. EX < FX. That is UX · V ̇ < PX · GFR or UX · V̇ / PX < GFR and CX < CINU [7] We can then state that if the clearance of a freely filtered substance is less than the clearance of inulin (GFR) that substance is reabsorbed. Similarly, it can be shown that, if the clearance of a substance is higher than the clearance of inulin, that substance is secreted

Renal Compensatory MechanismsEmb

Fortunately, the healthy adult kidney has a profound overcapacity to maintain fluid homeostasis and thereby support life. It is therefore possible for loss of function to go unnoticed until a high percentage of normal functioning has been lost. Kidney function in the adult can be lost due to chronic kidney disease, (a common sequelae of aging), or surgical removal of a kidney (due to disease, or from a healthy organ donor). Since antiquity, it has been known that removal of a kidney causes a compensatory increase in size in the remaining kidney, and in fact many disease processes also trigger such a compensatory increase in size. The earliest evidence of compensatory function is a decrease in resistance in unaffected glomeruli, mediated by normal functioning of the glomerulus and juxtaglomerular apparatus, their associated hormones and local factors. Subsequently, the glomerular capillaries increase in length, and glomerular interstitial space increases in volume, increasing the volume of the remaining functional glomeruli. Remaining functioning nephric tubules grow in both length and diameter, with the greater increases seen proximally. Importantly, glomeruli do not significantly increase in number. Glomeruli that are irreversibly lost by a disease process decrease permanently the total number of available glomeruli. Thus, it is important to ask what mechanisms exist that can reverse damage to glomeruli and tubules. The nephron is interesting in that while stem cells exist in place throughout the epithelial tissues, no obvious difference in their morphology or function has yet been discovered. It is possible that many or all cells of the nephron have a latent capacity to revert to a stem-like state. In addition, it has been shown that the bone marrow can serve a source of stem cells that incorporate into nephric epithelia. Despite such potential, the natural repair by stem cells seems to be a process more suitable to the repair of normal cellular senescence than to the repair of pathological damage

Efferent Vasculature

Glomerular capillary loops merge to make efferent arteriole which becomes 1) peritubular capillary plexus in the cortex/dips into medulla or 2) vasa recta: capillary looks in medulla that follow henles loop. Veins draining mirror arteries supplying.

Explain the renal regulation of acid-base balance

Glomerular filtration makes filtrate with alot to be reabsorbed. Preserving acid-base balance requires reabsorption of 4300 mEq of filtered HCO3 daily (24mEq/L) * (180 L/day). The kidney must also create 70 mEq of HCO3 to replace the HCO3 destroyed due to fixed acids that form CO2 and H2O. Both reabsorption of bicarbonate and forming new bicarbonate via kidneys requires secretion of H+ into the urine. H+ is secreted by 2 methods: Ion exchange or primary active H+ transport. Na/H and Na/NH4 exchange through the NHE3 transporter in PCT depends on the basolateral Na/K ATPase so there is low intracellular Na that allows Na to come into the cell and H/NH4 to leave via the lumen. NH4 is formed by local metbol of amino acids. Active H transportoccurs in the distal tubule and collecting duct helps lower the pH beyond possible because HCO3 is not reabsorbed in this area (only in the PCT/TAL area)

Acid-Base Balance Venous blood is acidic, Arterial blood is basic.

H ion & pH controlled for: Body acid poduction, Renal bicarb transport, PCT reabsorption, Collecting duct bicarb transport, Formation of new bicarb, titratable acid, ammon-genesis, renal resp. to acidosis, and acid-based disorders.

Formation of new Bicarbonate can be made through ammonium production 2/3 of excretion of acid is through ammonium

HCO3 is reabsorbed in the PCT and H is secreted Gradient is formed from NH4 and NH3 release in TAL Rh glycoproteins in the CD secrete NH3 ammonia that joins with H that is pumped out.

Receptor firing rates (A) and cardiovascular responses (B) in response to graded hemorrhage in the dog.

HR/Venous tone rise with drop in Blood Pressure/Venous Pressure lower with drop in BP CP barareceptors more sensitive to drop than arterial BP.

Renal dialysis

Hemodialysis - Blood passes along a filtration membrane which draws out waste via an osmotic gradient • Peritoneal dialysis - Fluid is added to the peritoneal space via a catheter, then drained after a few hours - Uses the peritoneal wall as a natural semipermeable membrane

Bladder The bladderis a hollow muscular organ that serves as a reservoir for urine. When empty, the adult bladder lies behind the pubic symphysis in the pelvis. In infants and children it projects more superiorly. As the bladder fills its superior aspect rises above the symphysis. In this state the bladder can readily be palpated or percussed through the abdominal wall Trigone Triangular region at base of bladder created by ureters and urethra Clinical relevance: Urinary tract infections are more common in women because bacteria has a shorter distance to travel. Prostatic benign hyperplasia (enlarged prostate) - very common among older men. Narrowing of urethra can cause frequent or urgent need to urinate, increased frequency of urination at night (nocturia), difficulty starting urination, weak urine stream

Hemodialysis : Blood removed through artery in arm, dialyzed, then returned to vein in arm. Typically, an arteriovenous (AV) fistula or an AV graft will be surgically created to act as the access point for dialysis. Typically performed in a clinical setting ~3 times per week Peritoneal dialysis: A sterile fluid is introduced into the abdomen through a permanent tube that is placed in the peritoneal cavity. The fluid circulates through abdomen to draw impurities from surrounding blood vessels in the peritoneum, which is then drained from the body. Performed ~5 times per day at home.

Hormonal involvement

Hormones protect CV from blood loss and increase sympathetic activity causes adrenal medulla to secrete Epinephrine/NE to contribute (E does not vasodilate B-2R if A-1R NE also fired). Sympathetic activation of kidney decreases renal perfusion which leads to release of renin that forms the vasoconstrictor AII. Decreased nerve activity from cardiopulmonary receptors/baroreceptors removes inhibitons on ADH release so now ADH a vasoconstrictor at high concentrations is released + increse in BV. ADH also causes NO-mediated dilation of cerebral/coronary vessels.

Functional Development of the Kidney • When does nephrogenesis happen? • Did branching morphogenesis proceed correctly? • How many nephrons are present? • How many nephrons might be added? • Is there renal injury?

How many nephrons are present? How many nephrons might be added? Wilms' Tumor Functional Development • nephroblastoma • likely stem cell origin • onset in early childhood • very rare (500/yr in US), but higher propensity among Africans • usually treatable with nephrectomy

Explain how new bicarbonate is formed from excretion of hydrogen with urinary buffers because excreting hydrogen buffers leads to reabsorption of HCO3 into blood.

Hydrogen buffers include HPO4, cretainine which are titratable acids that contribute 1/3 of acid excretion. It is prompt but limited. HCO3/Cl reabsorbed every time H is pumped into tubular fluid.

Regulation of Potassium Secretion Hyperkalemia + Aldosterone

Hyperkalemia - Stimulates Na-K pump activity - Increases luminal K permeability - Induces aldosterone secretion • Aldosterone: - Stimulates Na-K pump activity and luminal Na and K permeability - Activation of transport proteins (mins-hr) - Synthesis of transporters (hrs-day)

How does respiratory acidosis look on the davenport diagrahm?

Hypoventilation causes increase pCO2 (therefore HCO3) to point B (higher HCO3 but slightly lower pH). Renal compensation will increase the pH (more HCO3) toward D to increase pH while remaining on the pCO2 Isopleth. pH can return to normal, the patient does not have a acid-base balance because there is still high pCO2 and bicarbonate. To fix this balance, the respiratory disorder causing hypoventilaton must be changed. Respiratory acidosis is chronic (D) because of closer-to-normal pH and higher bicarbonate. Respiratory acidosis is acute because of lower pH not yet compensated.

Distribution of Body Potassium ICF Vol: 28L [K+]: 130-150 mEq/L ~3,500 mEq 98% ECF 14L 4-5 mEq/L ~65 mEq 2%

ICF K distribution: • Muscle ≅ 2,600 mEq • Bone ≅ 300 mEq • RBCs ≅ 300 mEq • Liver ≅ 300 mEq Plasma K: • Normal 3.5 - 5 mEq/L • Hypokalemia < 3.5 mEq/L • Hyperkalemia > 5 mEq/L

Short term responses to shock

In 10% mild hemoorhage: 500ml blood loss & immediate reduction in venous return but no drop in MAP due to rapid compensatory mechanisms (detected by non-myelinated aferent cardiopulmnary receptors due to reduced cardiac filling). Less discharge from CP receptors removes activation of the cardio-inhibitory center and depressor area and reduces vagal tone while increasing sympathetic activity. Leading to higher HR, contractility, TPR, which decreases rate for blood to seep out of arteries, and venoconstriction to lower venous compliance so more blood to heard. Higher TPR,HR,contract,venomotor tone mainitain the MAP but CO is not restored. Carotid sinus/aortic arch baroreceptors reflex: detect normal MAP but discharge rate is reduced because of smaller pulse pressure assoc. w/ reduced stroke volume. Rely info consistant with low-pressure receptors. Cardiopulmonary and baraoreceptors make the same responses. Selective constriction in skin, splanchnic, muscle, and kidney raise resisitance along with venomotor tone/cardiac response to reistribute blood from vein and heart ot arteries and keep pressure high enough to perfuse the coronary and cerebral vessels (no vasoconstriction here)

Other cells within the corpuscle:

In addition to endothelial cells and podocytes, the corpuscle contains mesangial cells. These replace connective tissue cells supporting the glomerular structures. Some of them form a pad of tissue (the extraglomerular mesangium; the cells are called lacis cells) in the region of the vascular pole where the afferent and efferent arterioles meet. Intraglomerular mesangial cells are contractile, and can alter blood flow through the glomerulus; they are also phagocytic, responsible for "cleaning" and repairing the endothelial/podocyte basement membrane so as to keep the filtration process efficient.

The urethra

In both males and females the lining epithelium begins as transitional like the bladder but becomes stratified squamous. Male urthera is terminal duct for both the male reproductive system and the urinary system. The urethra in the female is short, measuring only 3 to 5 cm in length. It terminates just posterior to the clitoris.

Autoregulation

In spite of changes in renal perfusion pressure that take place, for example, with changes in posture or diet, the renal blood flow (RBF), particularly glomerular blood flow, and the GFR are maintained relatively constant between renal arterial pressures of 80 and 180 mm Hg (see Fig. 2). This is accomplished by intrinsic mechanisms (they occur in a denervated, perfused kidney) that alter arteriolar resistances. One component is the myogenic response of the afferent arteriole smooth muscle that contracts when stretched and relaxes on lowering the distending pressure. This mechanism is mediated, as in other systemic arterioles, by stretch sensitive cation channels. The mechanism adjusts the resistance of the afferent arteriole according to the distending pressure and results in near constant flow rate and glomerular hydrostatic pressure. The efferent arterioles do not seem to respond directly to changes in stretch and therefore do not contribute to the myogenic response. Fig. 2: Autoregulation of the renal blood flow (RBF) and glomerular filtration rate (GFR) as the renal perfusion pressure is varied. The ability to maintain renal hemodynamics become impaired at pressures below 70 mm Hg.

Vasculature Bladder • Arterial supply: superior and inferior vesical arteries + vaginal artery in females • Venous drainage: to prostatic / vesical plexus of veins which drain to internal iliac v

Innervation • Parasympathetic innervation [via the pelvic splanchnic nerves, S2-4] - stimulates contraction of detrusor muscle, relaxes internal urethral sphincter • Sympathetic innervation relaxes detrusor muscle, constricts internal urethral sphincter

Renal cortical interstitium

Interstitial tissue is 10% of the cortex, has collagen type I & III, fibronectin, less between PCTs and capillaries Cortical interstitial cells include fibroblast-like EPO cells, lymphocyte-like APCs

Renal medullary interstitium

Interstitial tissue more abundant in medulla (40%) than cortex, matrix is high osmolarity. Interstitial cells are fibroblast-like cells with prominent lipid droplets, synthesize prostaglandin, change appearance with diuretic state, lymphocyte-like cells, pericytes

Medullary interstitium Renal intersititum = loose connective tissue aroundnephron. Small tight packed in cortex but more prominent in medulla (medullary inter resp for osm of urine). Fx in renal water reabsorption due to build up of high hyperosmotic pressure to draws water out of the thin descending limb of the loop of Henle and CD.

Interstitium is Ct surrounding tubules/blood vessels. More visible in the medulla than in the cortex. Hyperosmotic & draws water from filtrate to collecting ducts to reabsorb.

General. The nephron is a complex, tubular, unbranched, and precisely tortuous structure

It begins in the cortex with the spheroid two-layered Bowman's capsule that is composed of two distinct cellular layers encapsulating a tuft of fenestrated capillaries. The next, tubular, portion of the nephron meanders around its origin in the cortex, and then it descends into the medulla and forms a loop, known as the loop of Henle. The tubule returns to the cortex where it contacts its own Bowman's capsule. This distal part of the nephron ultimately flows into a collecting tubule that drains into a larger radially-oriented collecting duct. The collecting ducts join with other collecting ducts and, as a bundle, descend through the medulla and eventually empty into the lumen of the minor calyx. Since the tubules of the nephron intermingle extensively with each other and are frequently very convoluted, it is impossible to follow the morphological changes within a single nephron in any one plane of section. Therefore, you will search for examples of these various components of the nephron using specific morphology and location to guide you. Identification of the microscopic components of the kidney: overview. Return to slide 455. Review the circular solid-looking profiles scattered in the cortex: the glomeruli. They are the tufts of capillaries that mark the beginning of the nephron. Review the prominent medullary rays that extend radially between the medulla and the cortex. These are bundles of collecting ducts and some straight tubules of the nephron.

The juxtaglomerular apparatus reg fx of nephron.At vascular pole of the renal corpuscle & regulates renal blood flow/glomerular filtration rate. Ascending thick limb of Henle in cortex is called distal tubule (no histo change). DT to vascular pole of own renal corpuscle to go to afferent arteriole where in wall of DT near AA is columnar cells with closely nuclei, called the macula densa.

JGA: 1. MD: tall cell in DT JGC: Smooth muscle + renin (tunica media of AA) EGMC (lacis): in interstitial space near AA/EA/MD/VP

What is the normal filtered load of K. What are the tubular sites of K reabsorption/secretion.

K balance requires 90 mEq K excreted into urine daily (renal excretion is regulated by pK and aldosterone). Potassium is freely filtered at the glomerulus at 600-900 mEq/day. Most of filtered K is reabsorbed at the proximal tubule PCT and thick ascending loop of Henle TAL. K is reabsorbed by the PCT via difusion through the intercellular pathway (tight jx) due to reabsorption of water that leads to higher concentration of K. The late proximal tubule has a lumen with positive transepithelial potential so it also now favors paracellular K reabsorption. 80% of filtered K is reabsorbed at the PCT. The transcellular pathway is NOT used for K reabsorption. Under normal condition, K movement from lumen into cell is against the electrochemical gradient due to negative cellular potential is not enough to oppose the K outward driving gradient. Reabsorption at the TAL involves cotransport of Na/K/2Cl (inhibited by loop diuretics) at the lumenal cell membrane and K/Cl cotransport + K channels at the basolateral membrane. Some K entering the cell can leak back into lumen via apical cell. Net reabsorption of K in TAL is 10% of filtered load. Unlike PCT/TAL that can only reabsorb K, DCT/CD can reabsorb or secrete regulated by hormones and plasma K. BC 90% of K reabsorbed at PCT/TAL, chanes in excretion rate of K is all responsible on the distal nephron.

GFR = Kf [(PGC - PBS) -(πGC - πBS)]

Kf is the filtration coefficient, which is a function of the permeability of the glomeruli and their surface area. The surface area is reduced by contractions of the podocytes and the supporting mesangial cells under the influence of angiotensin II or epinephrine. The glomerular capillaries have a Kf about 100 times larger than that of other capillaries, underlying the enormous difference between the filtration rate in the glomerulus and that of the arterial end of all other systemic capillaries (about 20 L/day). The pressure in the glomerular capillary (PGC) is opposed by the pressure in Bowman's capsule (PBS) and the oncotic pressure in the glomerular capillary (πGC). The oncotic pressure in Bowman's space (πBS) is about zero and can be ignored. If we assume that Kf is constant at a particular time, GFR is proportional to (PGC- PBS) - πGC = ΔP-π, which sometimes is referred to as the ultrafiltration pressure, PUF.

Differences in tubular sites of action of K wasting and K sparing diuretics.

Loop diuretics/osmostic/thiazide will increase the rate of tubular flow in the distal nephron to ehnance K secretion. It is an unwanted side affect so K must be supplemented. K-sparing diuretics like sprionolactone, triamterene, and amiloride avoid this because spironolactone is a inhibitor of aldosterone and the other two inhibit lumenal Na distal channels so less K secretion occurs.

The kidney can control efficiency via altering glomerular perfusion, dynamics of filtrate modification, and responding/making hormones. The Juxtaglomerular apparatus is at the vascular pole.

Macula densa is near the afferent arteriole, it monitors NaCl concentration in the DCT to regulate renin secretion. JxG cells have renin granules made from tunica media of arterioles. Mesangial cells eat debris, basment membrane support, and secrete glomerular injury molecules acting like pericytes.

5. Collecting Ducts.

Make up the medullary ray of the cortex in the kidney. Epithelial cell are more pale, cuboidal-columnar + round nucleus, domed apical surface, distinct cell boundaries, principal numerous cells, inercalated darker cells. Travel from cortex to medulla, height of CD cell increases closser to renal papilla. Apex of RP = union of CD leads to formation of Papillary ducts of Bellini = 200-300 um wide, uine from this to calyx capping the papilla.Renal papilla with lumen of the minor calyx, lined with transitional epithelium, at the bottom of the image. The largest tubules in this field are the collecting ducts. Diameter of CD increases closer to lumen of the calyx. 2nd tubule is ascending thick limb of the loop of Henle. VR seen bc blood cells. High % is interstitium.

How does metabolic acidosis look on the Davenport diagram?

Metabolic acidosis (caused by diabetes/diarerhea) is caused by a fall in HCO3 which is then quickly recovered by respiratory hyperventilation to reduce pCO2 and increase pH back near normal. The patient cannot reach F 7.4 pH because the increased ventilation due to low pH causes pCO2 to fall and cause rise in CSF pH (basic) that prevents further rise in pH. Fixed acid combines with HCO3 to cause it to leave as CO2/H2O. The kidney can also excrete acid to restore HCO3 lost in a few days

How to diagnose

Mixed metabolic alkalosis/respiratory acidosis: cardiopulmonary arrest (lactic acidosis (low pH), cessation of respiration (low pCO2) Mixed metabolic acidosis/respiratory alkalosis: pH is normal but +HCO3 -PCO2 (aspirin overdose) bc uncoupling ox phos by salicyclic acid and hyperventilation by centers impacted by salicyclic acid High altitude respiratory alkalosis leads to hypoxemia -> hyperventilation -> lower pCO2. Hypoxemia also causes anaerobic metabolism/acidopsis against respiratory alkalosis to normal pH Acidic pH (Metabolic if HCO3/pCO2 low; Respiratory if HCO3/pCO2 high with low pH) HCO3 -24 Metabolic Acidosis pCO2 -40 Compensation via hyperventilation pCO2 +40 Respiratory Acidosis HCO3 +24 Compensation via renal HCO3 absorb. Basic pH (Metabolic if HCO3/pCO2 high; Respiratory if HCO3/pCO2 low with high pH). HCO3 +24 Metabolic Alkalosis pCO2 +40 Hypoventilation Compensation pCO2 -40 Respiratory Alkalosis HCO3 -24 Compensation via renal HCO3 excretion

11. Which of the following would cause a decrease in K+ secretion by the distal tubule? A. Inceased delivery of non-reabsorbable anions to the distal nephron. B. Osmotic diuresis due to increase blood glucose. C. Inorganic metabolic acidosis D. Blockage of the distal tubule Na+-Cl- cotransporter. E. Increased plasma K+ concentration.

More anions = more secretion More BG/diuresis = more secretion Alkalosis = more K secretion (opp H) Acidosis = H secretion so more K reabsorption Block Na/Cl = more secretion Increased pK = more secretion

Standing osmotic gradient hypothesis for fluid absorption by epithelia

More sodium pumps are located at the lateral membranes, near the apical end. • The effective osmotic pressure across cell membranes and junctions is larger than across basal membrane and capillaries, leading to net water movement from lumen.

Innervation Kidney

Motor: The autonomic nervous system affects the pacemaker cells in the collecting system. These cells initiate peristaltic contractions in the muscular wall of ureter: Parasympathetic system activates the peristalsis (vagus n) Sympathetic system regulates vascular tone (sympathetic trunk) Sensory: Pain response is triggered by stretching of the urinary collecting system, including stretching of the ureter. Stretching of the ureter causes visceral pain that is accompanied by a reflex-like activation of the subcostal nerve, genitofemoral nerve and ilioinguinal nerve

Renal Handling of Potassium

Normal diet: PCT reabsorbs and DCT secretes High K diet: PCT eh and DCT super secretes Low K: PCT eh and DCT reabsorbs!

What do you consider to be an appropriate treatment to correct the hyponatremia?

Normovolemic/asymptomatic hyponatremic: Free water restriction + IV V2 receptor antagonists. Low pOSM -> swelling of cells = danger of neuro death. Acute hyponatremia more dangerous than chronic due to chronic adaption for loss of intracellular osmolyte that reduces volume of brain cells.Plasma Na concentration returned to normal w/ infusion of hypertonic NaCl 3% saline w/ 513 mEq/L Na to get to concentration at rate of 1mEq/L per hour. Needs careful monitoring.

Continuous operation of this countercurrent multiplier leads to a progressive rise of interstitial osmolality from 300 mOsm at the cortex to as much as 1200 mOsm at the papillary region.

On the other hand, the osmolality in the lumen of the cortical portion of the ascending limb is about 100 mOsm. As we shall see later, the establishment of these gradients permits either dilution or concentration of the urine, depending on the level of plasma ADH. Although our discussion and the model in Fig. 3 imply active transepithelial transport of salt along the entire length of the ascending loop of Henle, active transport has only been demonstrated in the thick ascending limb. Countercurrent multiplication apparently involves the thin limb as well, however, as suggested below. The osmolality in the inner medulla is due not only to salt but also to urea (see Urea recycling below), which may account for up to 50% of the solute there. The presence of urea underlies another proposed, more recent model that may help to explain the medullary solute gradient. The model depends on the thick ascending limb actively reabsorbing salt in the manner described above, but it assumes that passive processes occur in the thin loops. The model is based on two assumptions: 1) the descending limb is permeable to water, but not to salt or to urea to any extent, and 2) the ascending limb is permeable to salt and to some extent urea, but not to water. It is argued that as the fluid moves through the descending limb water moves into the hyperosmotic interstitium. The solute concentration in the tubule lumen (mainly salt) increases until its osmolality balances that of the interstitium, which is due to salt ions and urea. The result is a luminal fluid with the same osmolality but a higher salt concentration than that in the medullary interstitium. When the fluid in the descending limb turns at the tip of the loop and moves into the ascending limb, which is permeable to salt, salt moves into the inner medulla down its concentration gradient. This process adds salt to the innermost regions of the medulla and is considered to play a role in maintaining the high osmolarity there. It should be cautioned that while these models are useful in trying to understand the creation and maintenance of the medullary osmotic gradient, further research is necessary before details of this process can be fully understood.

A. The Nephron

Overall structure: Each nephron consists of a renal corpuscle and its associated tubules. The renal corpuscle consists of glomerular capillaries enclosed by Bowman's capsule. The urinary space between the capillaries and Bowman's capsule contains the ultrafiltrate, which is the first step in the process of urine formation. Bowman's capsule turns into a renal tubule at the urinary pole of the corpuscle. The renal tubule has many different subdivisions, each with its own type of cells, but the overall purpose of the tubule is to transform the filtrate into urine with the correct composition of water, ions, and organic molecules for the state of the organism. [So, the filtrate is what is within the tubules of the nephron, after the corpuscle, and urine is what enters the minor calyx and travels through the rest of the system.] Each renal tubule forms a loop (called the loop of Henle) that passes radially down into the medulla and then back in juxtaposition with the renal corpuscle from which it originated. The proximity of the loop to the capillaries in the medulla enables the proper reabsorption/secretion of ions and water.

Ultrafiltrate exits urinary space and is changes by renal tubule epithelia, cortical tubules, loop of henle, and collecting duct.

PCT/PST/TcDT: large molecule reabsorption, active reabsorption TiDL/TiAL: passive water reabsoprtion TcAL/DST/DCT: active salt transport CT/CD: hormone based water reabsorption

Sensory Innervation of Kidneys & Ureters

Pain associated with these organs can be caused by: Stretching the collecting system, Direct injury, Irritation of surrounding tissues. , Pain fibers of kidneys and ureters are primarily preganglionic sympathetic nerves that reach spinal cord levels T11 to L2. Ureteric pain is caused by stretching the ureter, which causes visceral pain that is accompanied by a reflex-like activation of the subcostal, ilioinguinal, and genitofemoral nerves. Irritation of the renal capsule or other tissue around the kidney directly irritates the local body wall nerves (T11- L1) Clinical test: Costovertebral angle tenderness (CVAT) Tenderness upon percussion of the back at the junction of the 12th rib and the vertebral column, i.e. over the kidney. CVAT indicates irritation of the renal capsule (as in pyelonephritis, an infection of the renal parenchyma) or other inflammation of the tissue around the kidney.

Layers from outermost to innermost:

Peritoneum anteriorly and transversalis fascia posteriorly • Paranephric fat - extraperitoneal fat of the lumbar region; most obvious posterior to the kidney • Renal fascia -layer of connective tissue that encapsulates kidneys and adrenal glands along with the perinephric fat o Gerota's fascia: anterior layer o Zuckerkandl fascia: posterior layer (note that many references refer to both layers as Gerota's fascia) o Layer extends inferiorly along the ureter: periureteric fascia • Perinephric fat • Renal capsule - tough fibrous layer surrounding kidney • Renal cortex • Renal medulla

Comparison Between Titration of Phosphate and Ammonium

Phosphate is a better buffer bc pH is within tubular fluid range

Renal corpuscle has a glomerulus inside paretal layer of Bowman's capsule. • The glomerulus has 3 layers: thinnest is a filtration barrier Each kidney is about 10 x 4 x 2 cm in size. Renal A & V bring blood Minor -> Major Calyx -> Ureter

Podocytes make the visceral layer of Bowman's capsule. Podocyte foot pedicels interdigitate to make the glomerulus basement membrane. 2 epithelia make 3 parts of the glomerulus: Two epithelia create three compartments in the glomerulus: vascular (capillaries cont w aff/eff art), Connective tissue (mesangial, base mem cont w/ cotrical intersititum), urinary space (cont w lumen of PCT). Maintain basal lamina via phagocytosis + production of collagen + reg cell prolif/immunne. Filter by + charge, small size. Capillary endothelium fenestrated, basement membrane shared by endothelium and podocyte is thicker. Mesangial cell phagocytose debri from BM. Adjacent pedicel filt slit + diaphragm make up transmembrane protein nephrin.

Diseases

Polyuria - more pee DI: No ADH (no no pee hormone) so more pee, dehydration, thrist Cholescystectomy - remove gb BUN: plasma nitrogen from waste product urea. Increased plasma BUN/Creatinine = reduced GFR. Urea reabsorption enhanced by ADH, BUN increased more than plasma creatinine during hypovolemia (when ADH is secreted) Urinalysis: Appearance, Concentration, protein, glucose, cells, bacteria, etc. Gluycosuria: Sugar in urine (usually none) Proteinuria: Protein in urine (usually few) Proteinuria: Presence of excess proteins in the urine Splanchnic circulation: Circulation from celiac trunk, IMA, SMA

Sodium Reabsorption by the TAL

Reabsorption by the TAL is load dependent. • Regulatory mechanisms: - ADH stimulates sodium reabsorption - Prostanoids and NO, released locally when ECF sodium/volume is elevated, decrease reabsorption

Afferent Vasculature

Renal artery bring blood from aorta to kidney and makes 1) segmental arteries feeding unqiue segment then 2) interlobar artey traveling inbetween lobes of medulla then 3) arcuate arteries which are parallel to kidney at cortico-medullary border then 4) interlobular which travel radially toward cortex surface then 5) afferent arterioles that supply the glomerular capillary in the renal corpuscle.

Innervation

Renal pain fibers are primarily preganglionic sympathetic nerves that reach spinal cord levels T-11 to L-2. Spinal transmission of renal pain signals occurs primarily through the ascending spinothalamic tracts. Manifests as flank pain. Costovertebral angle tenderness (CVAT) is a clinical sign in which the patient reports tenderness when the practitioner percusses (taps) the back over the location of the kidney. CVAT indicates irritation of the renal capsule (e.g. as in pyelonephritis) or other inflammation of the tissue around the kidney as the percussion shifts and disturbs the inflamed tissue.

Davenport diagrom shows acid-base disturbances. It is based on: pH = 6.1 + log(HCO3)/.03*pCO2 Y Axis: HCO3 X Axis: H+ Curved Line: pCO2 (different partial pressures/isopleths) Straight Line: CO2 titration curves - changes in HCO3/H+ if the partial pressure of CO2 changes. Shaded Hexagon: Normal values Values of HCO3 and pH along each curved line (isobar) are calculated from the Henderson-Hasselbach equation with pCO2 held constant. Davenport response to acid load (protein/diarrhea)

Respiratory acidosis/alkalosis is due to changes in curved line pCO2. Hyperventilation at high altitude: Respiratory Alkalosis Hypoventilation w/ nerve damage: Respiratory Acidosis Metabolic acidosis/alkalosis due to the removal of non-volatile base/acid from the body. Diarrhea/Diabetes: Loss of HCO3: Metabolic acidosis Vomiting/Hypovolemia/Hypokalemia/Diuretic Use: Loss of H or Excess HCO3+: Metabolic alkalosis Changes in pCO3 for metabolic disorders are a compensatory response.

4. Juxtaglomerular Apparatus.

Return to the cortex to view the distal part of the nephron. When the ascending thick limb of Henle's loop enters the cortex it is renamed the distal tubule. The histology does not change between the ascending thick limb and the distal tubule. The distal tubule travels to the vascular pole of its own renal corpuscle where it contacts the afferent arteriole. At this site of contact there is a specialization in the wall of the distal tubule: an elliptical clustering of columnar cells with closely packed nuclei, called the macula densa (L. dense spot). Scan this slide to identify renal corpuscles sectioned near the vascular pole, and then inspect them to see whether there is a macula densa in the portion of a distal tubule included in that plane of section.

Normal K daily is 100mEq. 5-10 leaves via feces and less via sweat. 90 mEq K absorbed by GI into skeletal muscle and live through uptake using the Na-K ATPase.

Rise in plasma K after meal activates the Na-K ATPase pump. Epinephrine and insulin also stimulate the Na-K ATPase pump (increased pump turnover rate after activation of cytosolic kinase PKA PKC ERK1/MAP2. Insulin post-meal rises due to plasma glucose so insulin is the most important post-meal K-uptake hormone. Ex. If B adrenergic blocker, plasma K elevation after load is larger because Epinehprine cannot bind to B2 to enhance K uptake. Opposite Ex. If E or B2 agonist given, K reduces a lot. Insulin-deficient diabetics has higher plasma K after meal. Hyperkalemia can by treated by insulin or glucose to release insulin (or both to prevent hypoglycemia).

Control of Renin Secretion

Secretion of renin is stimulated by Increased renal sympathetic activity Reduced NACl in the fluid perfusing the macula densa Reduced perfusion pressure at afferent arterioles (intrarenal baroreceptor)

The Three Kidneys

Segmental kidneys grow sequentially in three regions, cranial to caudal, of the intermediate mesoderm. • The metanephric kidney takes advantage of branching morphogenesis to grow an elaborate duct system & form • Evolution is recapitulated in the sequential formation of kidneys in three separate region. • The pronephric and mesonephric kidney regress during embryonic life. • The mesonephric duct forms alongside all three regions. • Pronephric glomeruli drain to the coelomic cavity • Mesonephric glomeruli include a Bowman's capsule • Metanephric kidney forms lobes & lobules through branching of a common ureteric bud

B. Renal interstitium and functional compartments of the kidney

Since the various basal lamina present in the kidney play a large role in restricting the movement of cells and macromolecules, another useful way to understand the organization of the kidney is to consider the functional compartments that are delineated by various epithelia and their basal laminae. Basal laminae in the kidney surround the vasculature, the nephron, and the epithelia of the collecting ducts, calices, and ureter. The connective tissue compartment is external to all of these, being further subdivided only by gross anatomical considerations. The connective tissue compartment may be divided into hilar, cortical, and medullary regions. Hilar connective tissue serves mainly in a supporting role for the renal lobes. It often contains an abundance of adipocytes. Cortical and medullary connective tissue are physiologically important in that they form a buffer between resorbed materials and the blood. Because of the anatomical isolation of connective tissue in the medullary pyramid, and the active resorption of salt by thick ascending tubules, medullary interstitial tissue becomes increasingly hypertonic as one moves towards the base of the medullary pyramid. This tissue thus provides the substrate for the physiological countercurrent exchange system of the kidney. Connective tissue forms a greater percentage of the total tissue within a renal lobe as one moves deeper. Interstitial cells, a modified form of fibroblast cells, are present in both cortical and medullary compartments. Cortical interstitial cells release erythropoietin, and medullary interstitial cells release prostaglandins - two major hormones that influence blood pressure. The glomerulus involves vasculature, metanephric tissue and cortical connective tissue intertwined. With the compartmental topology above in mind, it should be apparent that a substance crossing the filtration barrier of the glomerulus is effectively crossing the connective tissue compartment, with that crossing being made highly efficient by the local absence of Types I and III collagen at this interface.

Hormonal Regulation of K Homeostasis

Slight increases in plasma K after a meal stimulates K uptake through the Na/K-ATPase • Insulin and epinephrine up-regulate K uptake by increasing the Na/K-ATPase turnover rate. - Since insulin is also released after a meal it is the most important hormone promoting K uptake (insulin deficiency leads to hyperkalemia) - Administration of β antagonists may lead to hyperkalemia after ingestion of K or its release from muscles during exercise • Treatment for hyperkalemia - Insulin (w/dextrose) - β agonists • Aldosterone role in extra-renal K homeostasis is uncertain

Correlation of metabolic acidosis with severity of chronic kidney disease.

Slower GFR/chronic kidney disease = higher chance of metabolic acidosis

Sodium Reabsorption in the Loop of Henle

Sodium movement in the thin ascending limb of Henle's loop is relatively small and is important largely in urinary concentration and dilution (see later in the Osmoregulation lecture). The thick ascending limb (TAL) responds to increases or decreases in delivery of tubular fluid Na+ by immediate changes in Na+ reabsorption in the corresponding direction ("load dependent"). Regulatory mechanisms for TAL Na+ reabsorption are not fully understood. Three recognized mechanisms are: • Antidiuretic hormone (ADH), which stimulates Na+ reabsorption by the TAL in addition to its main regulatory role in water reabsorption by the distal nephron (see Osmoregulation lecture). Secretion of ADH is stimulated by small increases of plasma osmolality and by large (≥10%) reductions in ECF volume. • Prostanoids and Nitric Oxide (NO), released locally by tubular cells and by medullary interstitial cells when ECF Na+/volume increases, inhibit Na+ reabsorption by the TAL.

Explain mechanisms that lead to decompensated shock, collapse, and death

Studies on dogs show that if BP doesnt drop below 50mmHg, recovery is compensated. If it drops below 45 mm Hg,it is decompensated or progressive post transfusion.

2. The tubular portion of the nephron: in the cortex

Surrounding the glomeruli in the cortex, are many, many profiles of tubules. There are only three morphologically distinct types of small tubules in the cortex—proximal convoluted tubules, distal tubules, and collecting tubules—and they appear in that order of frequency. The collecting tubule is a short segment that connects the distal tubule of the nephron to the radially oriented collecting duct. The different morphologies of the epithelial cells comprising each tubule reflect the very specific functions of these tubules. Use location, morphology, and numerical frequency to identify these tubules of the cortex.

Regulation of Proximal Tubular Sodium Reabsorption The fraction of filtered sodium reabsorbed by the proximal tubules can change from 50% (ECF expansion) to 80 % (ECF contraction), under the influence of:

Sympathetic activity: Increased sympathetic activity stimulates proximal sodium reabsorption. • Angiotensin II: Stimulates proximal sodium reabsorption. • Nitric Oxide (NO): Produced by tubular cells, inhibits sodium reabsorption. • Starling Forces: Expansion of the ECF decreases the filtration fraction lessening the increase in oncotic pressure in peritubular capillaries. Since peritubular capillary hydrostatic pressure also rises, the reabsorption of fluid is reduced, retarding sodium reabsorption by the tubules.

Secretion of renin by juxtaglomerular cells involves the exocytosis of vesicles containing the enzyme, a process activated by protein kinase A, itself stimulated by cAMP. The adenylate cyclase producing cAMP in juxtaglomerular cells is inhibited by intracellular Ca2+ and upregulated by activation of several Gprotein coupled receptors (adrenergic and prostanoids). There are at least three distinct inputs to the juxtaglomerular cells that increase renin secretion in response to ECF Na+/volume depletion:

Sympathetic nerve activity: The renal sympathetic nerves directly innervate the juxtaglomerular cells. Binding of NE to β1-AR leads to increased AC activity and subsequent increased release of renin. • Perfusion pressure: The afferent arterioles behave as baroreceptors ("intrarenal baroreceptors"). When blood pressure in the afferent arteriole is elevated intracellular Ca2+ levels increase and inhibit AC and the release of renin. On the other hand when the perfusing pressure decreases, renin secretion is stimulated. • Delivery of NaCl to the macula densa: A decrease in the Na+ and/or Cl- concentration in the fluid flowing through the end of the ascending loop of Henle, the macula densa, increases the secretion of renin. Although not all the details of this process are known, it is likely that the local release of PGs by the macula densa cells is responsible for the stimulation of renin secretion Conversely, a decreased sympathetic nerve activity, increased renal perfusion pressure or enhanced delivery of NaCl to the macula densa, inhibit the secretion of renin by the juxtaglomerular cells.

Other adaptations for shock

Synthesis of albumin/plasma proteins in liver restore mass of circulating proteins in 3-7 days. Lost RBC are restored in 4 weeks due to renal release of erythropoietin that increases secretion due to low O2. False security in maintaining BP due to these adaptive responses means more clinical signs must be used to see if blood transfusion is needed.

Normal Values

T: 97.8-99.1 Pulse: 60-100 BP: 90/60-120/80 BMI: 18.5-24.9 Glu: <140 mg/dL post meal Na: 140 K: 4 BUN: 7-20 Glu fast: 70-100 Cl: 96-106 Bi: 20-29 Cr: .6-1.1 (higher in men) Hba1c: 4-5.6 6.5+ DM

Utilization of osmolality profiles

The above described mechanisms set the stage for excretion of either dilute or concentrated urine. As usual, water tends to flow from regions of high to regions of low water concentration (i.e., from regions of low to regions of high osmolality), but the rate of flow depends on the permeability of cell membranes to water. In the absence of ADH the distal tubules and collecting ducts are impermeable to water. The lumenal fluid flowing from Henle's loop is diluted (120 mOsm/kg), but water is unable to cross the tubules, and continuing salt reabsorption in the distal tubules and collecting ducts reduces the lumenal fluid osmolality even further. The osmolality of the voided urine may be as low as 50 mOsm5 (Fig. 5).

Bladder

The bladder is a subperitoneal organ that rises toward and even above the pelvic brim as it fills. Note the following anatomical features in Figure 3: Apex, fundus, trigone, ureteric orifices, internal urethral orifice, internal urethral sphincter, external urethral sphincter

Transport structures through which urine passes

The blood is filtered at the glomerulus, producing an ultrafiltrate. The ultrafiltrate becomes filtrate as it is modified in the nephron and the collecting ducts. Urine is released from the collecting ducts into the lumen of the minor calyx. minor calyx: Each renal papilla is capped by a funnel-shaped structure that collects the urine and is attached at its 'rim' to the parenchyma of the medullary pyramid. In a tissue section, you typically see a crescent lumen around the tip of the medullary pyramid and bordered by a C-shaped dense-appearing wall. This epithelium-lined dense connective tissue structure is a minor calyx. major calyx: several of the funnel-shaped minor calyces unite with their neighbors to form two or three larger chambers. renal pelvis: the major calyces fuse to form the funnel-shaped renal pelvis, which traverses the hilum, tapers, and becomes the ureter. ureter; the renal pelvis narrows to the tube of constant diameter that leads from the renal pelvis to the urinary bladder. One ureter from each kidney carries urine to the urinary bladder.

The thin limb of Henle's loop occurs only in the medulla.

The component cells are squamous with nuclei that project slightly into the lumen. At first glance many thin loops resemble a capillary, but the thin limb has a somewhat thicker epithelial wall and more closely spaced nuclei. In this tissue section, many of the capillaries (vasa recta) have blood cells within them and so are clearly not thin limbs of the loops. Move to a deeper region of medulla and attempt to identify the ascending thick limb and the thin limb of the loop of Henle, the capillaries (with blood cells), and the third tubule that populates the deep medulla, the collecting ducts, to be examined later. The collecting ducts have the tallest epithelia in the medulla.

Tubular Reabsorption and Secretion

The composition of the urine entering the renal pelvis is quite different from that of the glomerular filtrate, reflecting the operation of the other two basic renal processes: reabsorption and secretion by the renal tubules. Reabsorption refers to the movement of substances from the tubular lumen into the peritubular capillaries and secretion is the movement in the opposite direction. The proximal tubule is where most of the reabsorption of solutes and water takes place. The loop of Henle is responsible for the mechanism of concentration and dilution of the urine, while the distal tubule and the collecting duct are the site of action of certain hormones and constitute the "fine-tuning" portion of the tubule, where the final composition of the urine is achieved.

The nephron is a complex, tubular, unbranched, and precisely tortuous structure.

The first tubular portion of the nephron meanders around its origin in the cortex, and then it descends into the medulla and forms a loop, known as the loop of Henle. The tubule returns to the cortex where it contacts its own Bowman's capsule. This distal part of the nephron ultimately flows into the larger radially-oriented collecting duct. The collecting ducts join with other collecting ducts and, as a bundle, descend through the medulla and eventually empty into the lumen of the minor calyx. Since the tubules of the nephron intermingle extensively with each other and are frequently very convoluted, it is impossible to follow the morphological changes within a single nephron in any one plane of section. Therefore, you will find examples of these various components of the nephron using specific morphology and location to guide you.

Flow of HCO3 is from CO2/H2O to H2CO3 CO2 + H2O ↔ H2CO3 ↔ H+ + HCO3 ↕ Buffers H-Buffers

The increase in pCO2: Equal number of H+/HCO3, thus without buffer the increase in HCO3 = H+. So if pCO3 is doubled = 40nM more of H+ and HCO3 (which is much more drastic for H+ than for HCO3 24mM). BUT when there are non-bicarb buffers, pCO2 rises and causes a rise in H+/HCO3 but the buffers drain the H+ and pull the reaction to the right to make even more HCO3 up to the point they are nm->mm. Now with mM HCO3 but nM of H+, the slope of the CO2 titration curve is influenced by the buffering capacity by being more steep.

3. Juxtaglomerular Apparatus.

The juxtaglomerular apparatus consists of the cells of the macula densa and the juxtaglomerular cells. The macula densa is thought to respond to the composition of the fluid in the distal tubule by interacting with the juxtaglomerular cells that are in contact with the incoming blood in the afferent arteriole. The juxtaglomerular cells of the afferent arteriole are distinguishable because of their PAS-positive granules; they contain renin, which is important for the regulation of blood pressure, a topic that you will cover in the physiology lectures. Both the macula densa and the juxtaglomerular cells are thought to interact with mesangial cells (also called lacis cells) in the extraglomerular mesangium in their vicinity but the details are unknown.

Embryo of the kidney

The kidneys are formed out of the mesodermal layer of the embryo, in the intermediate mesoderm region alongside and just lateral to the caudal section of the neural tube (future spinal cord). Kidney organogenesis is complicated by the fact that paired kidneys form in three separate times & places during embryonic and fetal life. These kidneys recapitulate an evolutionary progression of increasing efficacy, twice using the Renal - 8 - 1 strategy of deleting the previously formed organ and starting over anew. So, functional temporary kidneys form in two regions, pronephros and mesonephros before the final adult kidney arises from the metanephros. Pronephros, mesonephros and metanephros describe three progressively more caudal territories of intermediate mesoderm differentiation. As in the rest of the body, differentiation of the intermediate mesoderm proceeds in a cranial to caudal order. As a reminder, the human embryo forms sequentially from ~39 somites, the caudal 33 of which eventually form the vertebrae. (The first-formed "occipital" somites contribute to more cranial structures.) The pronephros forms primarily from somites 9-13. The mesonephros encompasses somites 9-26, and the metanephros somites 23-28. At the time of the pronephros' existence, only between 9-23 somites have developed. Formation of the mesonephros begins from the 18 somite stage, and ends around 9 weeks PF. The metanephros begins forming at 4 weeks, and finishes in early childhood. (These numbers should not be memorized; they are presented merely to give you a feel of the spatial and temporal development of these three regions.) The pronephric kidney never contains more than a handful of functioning glomeruli. These glomeruli are not encapsulated by a Bowman's capsule, but rather reside directly under the parietal coelomic (peritoneal) mesothelium. Thus, ultrafiltrate enters the coelomic (peritoneal) space. Also in the pronephros, ciliated ducts develop to drain the coelomic space. These ducts coalesce among themselves, but in humans never form a proper drainage pathway. The pronephros resembles the functioning kidneys found in fish and in coelomate invertebrates. Later, and more caudally, a mesonephric kidney develops (meso > middle). This region develops proper, encapsulated glomeruli, connected via a rudimentary nephric tubules to the mesonephric region of the mesonephric duct. Against the coelomic cavity in the mesoneprhic region is located the gonadal ridge, which generates ovary and testis. Upon degeneration in a male, the nephric tubules become associated with the testis as the distal end of the reproductive duct (within the future epididymis). In a female, both glomeuli and nephric tubules lose function. The adult kidney forms in the most caudal region of the intermediate mesoderm, termed the metanephros. The kidney in this region is distinguished by elaborate branching of the tubule arising from the mesonephric duct, the ureteric bud. Rounds of branching and differentiation occur via a coordinated process of molecular signaling that is evolutionarily conserved across numerous organs (including lungs, mammary glands, salivary glands and pancreas), and termed branching morphogenesis. The ureteric bud will become the ureter and the highly-branched collecting ducts in each lobe of the kidney. At the distal end of each branch of collecting duct, (the future collecting tubules), the surrounding mesenchymal tissue is induced to condense, forming a metanephric cap. Each cap develops into a separate S-shaped tubular epithelial structure. One end of the "S" will connect to the collecting duct. The first curve will greatly elongate, becoming the nephron. The second curve will surround around a local capillary. As the capillary elaborates into the glomerular capillary tuft, the apposed cells of the tubule differentiate to form the podocytes. Branching morphogenesis allows the total number of functional nephrons in the metanephric kidney to reach approximately one million per kidney. (Interestingly, this number may vary over ½ an order of magnitude among individuals with no immediately apparent functional consequence.) For comparison, one source estimates approximately 7 functional units per pronephros, and 70 per mesonephros.

B. Patients who have greater than a 75% narrowing of one or both renal arteries are often hypertensive. Provide an explanation for the increased systemic blood pressure.

The low perfusion pressure distal to the stenosis leads to an increased synthesis and secretion of renin from the affected kidney(s) elevating systemic levels of AII and blood pressure. The higher systemic pressure normalizes the pressure in the renal artery downstream from the constriction.

Embryology of the Cloaca

The mesonephric duct must form a connection to the outside world - it does so by connecting to the rectum, forming the embryonic cloaca • Urinary bladder is derived from allantois mesonephric duct joins gut in a common "sewer" (cloaca) • bladder forms from embryonic structure allantois, extending to umbilical cord • remnant structure, urachus (usually fibrous) connects bladder to umbilicus in the adult • urorectal septum separates urogenital sinus from rectum Hypospadias

Receptors Sodium

The regulation of body Na+ is achieved by receptors (sensors) that detect changes in ECF volume secondary to changes in ECF Na content and by effector mechanisms that regulate the renal excretion of Na+ Changes in the amount of salt ingested are reflected in the volume of the ECF, because the thirst and ADH systems maintain body fluid osmolality within a narrow range. Thus, an increase in salt ingestion increases the osmolality of the ECF (and ICF because of osmotic equilibration) and leads to stimulation of thirst and the release of ADH. The increased fluid intake and urinary retention restore body osmolality but increase ECF volume. At the new steady state, the extra ingestion of salt has resulted in the addition of an isosmotic solution to the extracellular fluid. This can be seen in Fig. 2, where body weight increases (reflecting ECF volume gain) over a period of 5 days after an individual daily Na+ ingestion is doubled. Importantly, the excretion of Na+ varies in parallel with the change in ECF volume. In the new steady state Na+ ingestion and excretion are equal again. (balance), but during the adjustment period the individual is in positive balance (ingestion exceeds excretion) with ensuing increases in total body Na+, ECF volume and body weight. After reducing the ingestion to the original levels the person is in negative balance (excretion exceeds ingestion) for a few days and the total body Na+, ECF volume and body weight return to normal. Because under normal physiological conditions, the ECF Na+ concentration is maintained constant by the ADH and thirst mechanisms, the monitoring of the body Na+ content is achieved by the baroreceptors that sense changes in pressure (or stretch) in the arterial system. The pressure of the blood perfusing the baroreceptors, or the degree of filling in the arterial system is called the effective circulating volume (ECV). It is an unmeasured volume that reflects the extent of tissue perfusion. Under normal conditions it changes in parallel with the ECF volume, but in some cases it may be independent of it. An example that we discussed in the CVS section is that of the patient with CHF. In this case the ECV is decreased because the reduced cardiac output lowers the pressure at the baroreceptors. The decrease in pressure induces compensatory salt and water retention by the kidneys and leads to expansion of the ECF volume. The net result is that the ECV is reduced but the plasma and ECF volumes are augmented. There are other examples of dissociation between ECV and the ECF volume in certain pathologies, but it also happens under physiological conditions. For example, if a subject is immersed to the neck into warm water the hydrostatic pressure of the water acting on the lower extremities causes a redistribution of the blood from the legs to the chest. This leads to an increased venous return, cardiac output and ECV, although the ECF and plasma volumes remain unaltered.

Urethra

The urethra is the passageway for urine from the bladder to outside the body; in men it is also a passageway for semen. Female urethra: approximately 4cm long and opens into the vestibule of the vulva, anterior to the vaginal opening and posterior to the glans of the clitoris Male urethra: approximately 20cm long divided into prostatic, membranous, and spongy (penile) parts. Opening is in the glans of the penis.

The emphasis of this lecture will be on the kidney, with the goal of making the histological organization of structures within the kidney understandable and easy to remember.

The urinary system consists of the following organs and their functions: 1. kidneys (2 per person) -remove wastes (mostly nitrogen-containing compounds such as urea and creatinine) from the blood. -help maintain the correct osmolarity of the blood through selective reabsorption and secretion of ions. -produce and release renin, an enzymatic component of the renin-angiotensin mechanism that influences blood pressure. 2. urinary bladder -stores urine until it can be voided. 3. ureters and urethra -ureters (1 per kidney) conduct urine from kidney to bladder. -urethra (1 per person) conducts urine from bladder to outside world Nephrons are the individual functional units of the kidney that filter blood and produce urine and they are discussed in greater detail below.

Figure 10: This is a view of renal cortex from a tissue section in which the cells and nuclei of the nephron tubules are all stained a golden-brown color, their lumens white,and the blood cells in the capillaries are stained bright red.

There is relatively little interstitial tissue in the cortex, and the capillaries form a very dense plexus around the tubules. These capillaries pick up glucose, amino acids, bicarbonate, sodium, chloride, phosphate and water extracted from the filtrate by the proximal and distal tubules of the nephrons. (This light micrograph image shows a field that is entirely proximal and distal tubules of the nephrons.)

C. What do you think would be an appropriate treatment for these patients and what would be the main concern associated with the treatment?

These patients are often treated with antihypertensive therapy to lower the systemic blood pressure. As a result the renal perfusing pressure distal to the stenosis, including glomerular pressure, will fall to a level below normal. As indicated in the answer to question A , autoregulation plays an important role in maintaining glomerular pressure and GFR, a response that can be partially impaired by diminishing the production of angiotensin II with an ACE inhibitor (see Figure). About one-half of such patients given an ACE inhibitor will have a decline in GFR with danger of severe renal failure. Role of AII in autoregulation. Effect of reducing renal artery pressure on GFR in control dogs and in dogs which received intrarenal infusion of angiotensin II antagonist. To lower the renal artery pressure an adjustable clamp was placed on the abdominal aorta just above the left renal artery. Note that AII helps to maintain GFR in spite of the fall in perfusing pressure. Adapted from Hall, Guyton et al. Am. J. Physiol. 233(5):F366, 1977. Other antihypertensive medications are less likely to produce this problem, since they do not interfere with autoregulation. But if the perfusion pressure is markedly reduced the ability of autoregulation to protect the GFR is impaired (see figure above for RAP below 85 mm Hg). Thus any antihypertensive agent can produce acute renal failure when there are severe renovascular lesions.

The body responds to acid-base through buffers and pulmonary CO2 excretion.

They can prevent acute changes in H+ but cannut eliminate non-volatile fixed acids from the body to restore buffer supplies. Only the kidney can restore these.

Note in Fig.2 that the relatively high hydrostatic pressure at the afferent end of the glomerular capillaries changes only slightly at the efferent end because of the low resistance to flow in these capillaries.

This is in contrast to other capillaries in the body, where there is a progressive lowering of hydrostatic pressure towards the venous end with little change in the oncotic pressure, a situation resulting in reabsorption at the venous end of the systemic capillaries. In the glomerular capillaries, the large filtration of water and retention of proteins lead to a significant, progressive increase of oncotic pressure (πGC) as the fluid moves from the afferent to the efferent end. If πGC equals (PGC - PBS) at any point along the capillary, filtration ceases (filtration equilibration) as occurs in some animals. Although some investigators believe that this also happens in humans, most believe that the forces do not equalize and filtration continues at the efferent end of the glomerulus. Reabsorption does not occur along the glomerular capillaries.

Renal Lobules

This tissue preparation illustrates a single kidney lobe, which is functionally organized into lobules. Observe in the cortex regular light-staining radial striations. These lighter-stained medullary rays, which traverse the cortex from the medulla and taper toward the renal capsule, are prolongations of structures from the base of the renal pyramid. Each medullary ray is the center of a renal lobule. Thus, each medullary ray isintralobular in position, i.e., within each lobule. Before you leave this VM slide, compare the histology of the cortex and the medulla. The cortex uniquely contains spheroid renal corpuscles amongst a variety of multicellular tubules sectioned in various planes, in contrast to the medulla that is composed of highly ordered multicellular tubular structures surrounded by the renal interstitium of loose connective tissue. In regions where the medullary pyramid is horizontally sectioned the parenchymal components all appear as multicellular circular profiles

Epithelial cells are polarized

Tight junctions: • Band of specialized proteins (claudins) that bound together epithelial cells • Preserve the integrity of the epithelium • Form boundary between apical and basolateral domains of plasma membrane maintaining asymmetrical distribution of membrane proteins • Impermeable to macromolecules but with variable permeability to ions and water

Ultrafiltrate contains water, glucose, ions, amino acids, small molecules, urea BUT NOT CELLS LAGE MOLC ALBUMIN

Ultrafiltrate: filtered prior to modification in the urinary space/bowmans capsule- passes filtration barrier Filtrate: filtered substance during modification - in collecting ducts/tubules Urine = filtered substance after modification enters minor calyx/urter/bladder

Manifests as pain radiating in flanks, groin, and genital area

Ureteric colic (renal colic): Classic presentation: acute, colicky flank pain radiating to the groin; "Worst pain of (the patient's) life" often accompanied by nausea and vomiting. Pain is caused by obstruction of the urinary tract by calculi. Frequent emergency in medical practice. Between 5-12% of the population will have a urinary tract stone during their lifetime, and recurrence rates approach 50%.

Blood Supply

Ureters take their arterial supply / venous drainage from vessels they pass. Ureteric vessels pass longitudinally within the periurethral adventitia so it is important to protect/maintain this layer during surgeries. Abdominal ureteric vessels are on the medial aspect of the ureter and pelvic ureter vessels are on the lateral aspect: Abdominal ureter supplied by renal and gonadal vessels Middle ureter supplied by common iliac and gonadal vessels Distal ureter supplied by branches from common and internal iliac

Factors regulating RBF and GFR

Vasoconstrictors include sympathetic vs that act via a1-receptors to lower RBF/GFR, AII via renin/+symp/low arteriolar pressure. Vasodilators: NO can decide resting vasculare tone/PGE2/PGI prevent excessive vasoconstriction from sympathetic/AII. ANP will increase RBF and GFR.

Glomerulo-Tubular Balance

We have seen how the mechanisms of autoregulation help maintain near constancy of RBF and GFR in the face of changes in renal perfusion pressure. However, despite autoregulation, fluctuations in GFR do occur and could markedly influence the filtered load of Na+ . An increased filtration of salt and water must be compensated by their increased reabsorption to avoid severe salt and water depletion. This is achieved by a process termed glomerulo-tubular balance (G-T balance); when sodium balance is normal, Na+ and water reabsorption increase in parallel with an increase in GFR and the Na+ load. Thus, a near constant fraction of the filtered water and salt (about 2/3) is reabsorbed at the proximal tubules. G-T balance is achieved by two mechanisms. One involves Starling forces at the peritubular capillaries. For example, an increase in GFR, at constant RBF, increases the concentration of proteins and the oncotic pressure in the peritubular capillaries, and leads to an increased reabsorption of water and Na+ from the interstitial fluid. Another factor enhancing Na+ and water reabsorption in the proximal tubules following an increase in GFR is the augmented filtered load of glucose, amino acids and other organic solutes that are cotransported with Na+ at the proximal tubule. Since the cotransport mechanisms are normally operating below maximal capacity, an increased delivery of the organic solutes results in an enhanced reabsorption of Na+ and fluid. These two mechanisms of glomerulo-tubular balance ensure a constant fraction of reabsorption of Na+ and water at the proximal tubule and minimize changes in body salt and water that may result from alterations in GFR. Glomerulo-tubular balance takes place at the proximal tubule, loop of Henle and the distal tubule, although since the bulk of the filtrate is reabsorbed at the proximal tubule and loop of Henle it is here that it is more apparent. Together with autoregulation, which keeps the GFR relatively constant, G-T balance prevents the fluid delivery to exceed the limited reabsorptive capability of the collecting tubules, where the small changes in excretion of water and electrolytes are made to balance the changes in intake. The relationship between glomerular filtration and tubular reabsorption is not fixed, and it may be reset at a different level when there are changes in the effective circulating volume (see lecture on Sodium Regulation).

Regenerative Potential of the Adult Kidney

When the kidney is injured, functioning nephrons grow in size but not in number. (Hypertrophy without hyperplasia.) • Cells with stem potential are scattered throughout tubules. They are not obviously discernible. • Interestingly, bone marrow has potential to repopulate nephric stem cells in lab animals.

Renal Response to Acidosis Hydrogen secretion is caused by AII, aldosterone, and hypokalemia.

When there is acidosis: The kidneys will reabsorb all filtered HCO3, increase excretion of Hydrogen/TA, and increase production of ammonium. This occurs through changes in tubular cell pH, secretion of endothelin/cortisol/PTH, which impact NHE3, HATPase, NBCe1, RhCg, and synthesis of ammoniagensis enzymes. Acidosis has high TA/NH4+ Renal failure has low TA/NH4+

Glomerulo-Tubular Balance

With Na balance, Na and water reabsoprtion rises parallel to GFR/filtered load Na. Fs = Ps * GFR. An increase in GFR brings more organic solutes to the PCT and therefore enhances reabsorption of Na and H2O. Peritubular starling forces causes from an increase in GFR will increase the filtration fraction concentrate so there is more protein in the plasma of the peritibular capillaries. The increases GFR and protein causes rise in oncotic pressure so more fluid is reabsorbed into the capillaries. The Glomerular-Tubular balance has a constant fraction of filtered Na and H2O reabsorbed from PCT even when GFR varies.

B. ADH reduces blood flow through the vasa recta, perhaps by constricting smooth muscle cells proximal to the vasa recta in the juxtamedullary region. a) What physiological benefit would you predict for a reduction of medullary flow during antidiuresis? b) In the problem above assume that the GFR represents 20% of the renal plasma flow (RPF) and the vasa recta flow is 5% of the RPF. Calculate the difference between flow into and out of the vasa recta as the capillaries reabsorb fluid from the medullary interstitium during antidiuresis.

a) The vasa recta perfuse the medulla and carry away the fluid reabsorbed from descending limbs and collecting ducts. A decrease in flow reduces the tendency for washout of the concentrated medullary interstitium. It helps maintain a steep osmotic gradient for water reabsorption. Additionally, a decrease in vasa recta pressure from upstream constriction improves the Starling forces favoring reabsorption. b) With a filtration fraction of 20% and a GFR of 150 ml/min, the RPF would be 150/0.2 = 750 ml/min. The vasa recta inflow would be 5% of the RPF or 37.5 ml/min. Volume absorbed in the dLH (37.5ml/min) and medullary CD (3.2 ml/min) adds 40.7 ml to the outflow. Thus the outflow from the vasa recta would be 78.2 ml/min, a bit more than twice the inflow rate. Note that, in spite of containing more water, because of interstitial solute washout the outgoing vasa recta plasma is more concentrated than that of the incoming vasa recta.

33. Which of the following substances is likely to decrease renal blood flow? A. prostaglandins B. endothelin C. nitric oxide D. bradykinin E. histamine

endothelin is likley to decrease RBF


Kaugnay na mga set ng pag-aaral

Radiology: Chapter 7 Dental X-ray Film and Processing Methods

View Set